由买买提看人间百态

boards

本页内容为未名空间相应帖子的节选和存档,一周内的贴子最多显示50字,超过一周显示500字 访问原贴
Parenting版 - 忍不住上来问问,我娃的数学是啥水平(6岁)?
相关主题
陶天才论天才 (转载)我招,我是猪
Explaining Your Math: Unnecessary at Best, Encumbering at W大学录取率
是不是任我儿子继续迷象棋学校早晚要教的东西,早早学会了又如何?
9岁, 如何报名考amc10amc 10 的A, B卷有啥区别
有明天考AMC 8的吗?来点美好的时光
[转载] 为什么说usamo 简单有些小学四年级的数学题不太会做
二年级女儿的report转载: 从首个IMO季军谈起 by 付云皓
觉得美帝数学进度慢的进来看一下新加坡数学对小学生帮助大吗?
相关话题的讨论汇总
话题: 数学话题: asperger话题: aime话题: 普通话题: 小娃
进入Parenting版参与讨论
1 (共1页)
c********5
发帖数: 207
1
娃6岁了,一直都是普通娃,有些方面和同龄人比,还delay一年甚至更多。最近回国,
大家都说他数学有天分,可以多培养培养。当妈的我第一次有了飘飘然滴赶脚。上来一
是想问问,是不是大家忽悠我呢。二是想问问,下一步数学该如何推(无论是不是有天
赋)。真心讨论娃的问题,不喜欢,请右上角点X退出。万分谢谢。
1.我原本想给娃讲讲高斯定理,娃一看到我出的题,马上的反应是:
1+2+3+4+5+6+7+8+9+10
=10+1+9+2+8+3+7+4+6+5
=55
(这个学校里倒是常常要求凑10,也可能有人问过他这道题)
2.
8*8=9*9-9-8=64
家里面肯定没这么教过,我就是不确定学校里是不是教过。如果是他自己总结出来的,
我觉得还是挺厉害的。他知道9*9=81,可惜的是72-8算成了66,加减法基础根本不扎实
,乘除法这些花哨的东西倒是学会了。
3. 12*12=12*10+12*2=144
这个是妥妥的乘法交换律要。这个我倒是不确定是不是有人教过他。
n*******l
发帖数: 2911
2
六岁,乘除法无师自通,肯定是天才啊。

【在 c********5 的大作中提到】
: 娃6岁了,一直都是普通娃,有些方面和同龄人比,还delay一年甚至更多。最近回国,
: 大家都说他数学有天分,可以多培养培养。当妈的我第一次有了飘飘然滴赶脚。上来一
: 是想问问,是不是大家忽悠我呢。二是想问问,下一步数学该如何推(无论是不是有天
: 赋)。真心讨论娃的问题,不喜欢,请右上角点X退出。万分谢谢。
: 1.我原本想给娃讲讲高斯定理,娃一看到我出的题,马上的反应是:
: 1+2+3+4+5+6+7+8+9+10
: =10+1+9+2+8+3+7+4+6+5
: =55
: (这个学校里倒是常常要求凑10,也可能有人问过他这道题)
: 2.

t******l
发帖数: 10908
3
dcbang 有人喊你上葩坛灌数学。

:娃6岁了,一直都是普通娃,有些方面和同龄人比,还delay一年甚至更多。最近回国
,大家都说他数学有天分,可以多培养培养。当妈的我第一次有了飘飘然滴赶脚。上来
一是想问问,是不是大家忽悠我呢。二是想问问,下一步数学该如何推(无论是不是有
天赋)。真心讨论娃的问题,不喜欢,请右上角点X退出。万分谢谢。
Q***k
发帖数: 1419
4
如果没人教过,肯定是天才了

【在 c********5 的大作中提到】
: 娃6岁了,一直都是普通娃,有些方面和同龄人比,还delay一年甚至更多。最近回国,
: 大家都说他数学有天分,可以多培养培养。当妈的我第一次有了飘飘然滴赶脚。上来一
: 是想问问,是不是大家忽悠我呢。二是想问问,下一步数学该如何推(无论是不是有天
: 赋)。真心讨论娃的问题,不喜欢,请右上角点X退出。万分谢谢。
: 1.我原本想给娃讲讲高斯定理,娃一看到我出的题,马上的反应是:
: 1+2+3+4+5+6+7+8+9+10
: =10+1+9+2+8+3+7+4+6+5
: =55
: (这个学校里倒是常常要求凑10,也可能有人问过他这道题)
: 2.

a*****g
发帖数: 19398
5
不是机器人,就很厉害

【在 c********5 的大作中提到】
: 娃6岁了,一直都是普通娃,有些方面和同龄人比,还delay一年甚至更多。最近回国,
: 大家都说他数学有天分,可以多培养培养。当妈的我第一次有了飘飘然滴赶脚。上来一
: 是想问问,是不是大家忽悠我呢。二是想问问,下一步数学该如何推(无论是不是有天
: 赋)。真心讨论娃的问题,不喜欢,请右上角点X退出。万分谢谢。
: 1.我原本想给娃讲讲高斯定理,娃一看到我出的题,马上的反应是:
: 1+2+3+4+5+6+7+8+9+10
: =10+1+9+2+8+3+7+4+6+5
: =55
: (这个学校里倒是常常要求凑10,也可能有人问过他这道题)
: 2.

Z**********1
发帖数: 1447
6
他为什么知道9*9=81?
d***a
发帖数: 13752
7
呵呵,和我小时候差不多。:)
开开玩笑,这小孩挺好的。
s**********y
发帖数: 509
8
很好哈。 参考意见 见 数学教育, 一家之言

【在 c********5 的大作中提到】
: 娃6岁了,一直都是普通娃,有些方面和同龄人比,还delay一年甚至更多。最近回国,
: 大家都说他数学有天分,可以多培养培养。当妈的我第一次有了飘飘然滴赶脚。上来一
: 是想问问,是不是大家忽悠我呢。二是想问问,下一步数学该如何推(无论是不是有天
: 赋)。真心讨论娃的问题,不喜欢,请右上角点X退出。万分谢谢。
: 1.我原本想给娃讲讲高斯定理,娃一看到我出的题,马上的反应是:
: 1+2+3+4+5+6+7+8+9+10
: =10+1+9+2+8+3+7+4+6+5
: =55
: (这个学校里倒是常常要求凑10,也可能有人问过他这道题)
: 2.

t******l
发帖数: 10908
9
楼主娃非常聪明。。。这种情况下最好去测一下智商,如果是 gifted math kid 就找
gifted class。。。隔壁楼提到了轻微的 hyperlexia,对于万一有轻微的
hypernumeracy 的情况,测过智商也就知道怎么教娃。
我过去目测看猪跑,祖国一般程度的大路货的重点高中的数理校队里,十年一见的娃娃
,基本也不会有楼主娃娃这么先进,甚至有可能差的很远。。。所以我觉得楼主描述的
情况,八九不离十是超常娃娃。。。而超常娃娃就可能可以解释娃娃为啥知道 9*9=81
,而父母不知道娃已经知道,最大的可能超常娃娃某一次 glance 过后就记住了。

:他为什么知道9*9=81?
f*******e
发帖数: 3433
10
给6岁小娃讲高斯定理。。。 坑吧。9*9都知道,8*8还需要用9*9算。怎么不用更简单
的8+8+8+8+8+8+8+8? 或者10*8-2*8?

【在 c********5 的大作中提到】
: 娃6岁了,一直都是普通娃,有些方面和同龄人比,还delay一年甚至更多。最近回国,
: 大家都说他数学有天分,可以多培养培养。当妈的我第一次有了飘飘然滴赶脚。上来一
: 是想问问,是不是大家忽悠我呢。二是想问问,下一步数学该如何推(无论是不是有天
: 赋)。真心讨论娃的问题,不喜欢,请右上角点X退出。万分谢谢。
: 1.我原本想给娃讲讲高斯定理,娃一看到我出的题,马上的反应是:
: 1+2+3+4+5+6+7+8+9+10
: =10+1+9+2+8+3+7+4+6+5
: =55
: (这个学校里倒是常常要求凑10,也可能有人问过他这道题)
: 2.

相关主题
[转载] 为什么说usamo 简单我招,我是猪
二年级女儿的report大学录取率
觉得美帝数学进度慢的进来看一下学校早晚要教的东西,早早学会了又如何?
进入Parenting版参与讨论
s**********y
发帖数: 509
11
9* 9 - 9 - 8 能够不是从头算而得到 8 * 8 对三年级娃已属不易。能猜到这一关
系, 就像潮老大说的, 绝对超常哈。

【在 f*******e 的大作中提到】
: 给6岁小娃讲高斯定理。。。 坑吧。9*9都知道,8*8还需要用9*9算。怎么不用更简单
: 的8+8+8+8+8+8+8+8? 或者10*8-2*8?

s*******e
发帖数: 226
12
鎴戝濞宀侊紝涔熺煡閬撹繖浜涖濂藉儚鏄湅浜嗕粈涔堜功鍙楀埌鍚彂銆傛垜
缁欎粬鍑洪浮鍏斿悓绗奸棶棰橈紝鍙鏁板瓧涓嶅お澶э紝鑷繁涔熻兘鎺版壇娓呮
銆備笉瑙夊緱闇鐗瑰埆寮曞锛屽挨鍏舵槸鏁板銆傛病浜嬬粰浠栧嚭鐐硅叮鍛
抽锛岃浠栦繚鎸佸叴瓒c
l******t
发帖数: 55733
13
楼主知道什么是交换绿什么事分配绿吗
a****l
发帖数: 8211
14
要小心自闭症。自闭症的孩子往往在某方面有特殊表现。如果别的方面发展不均衡的话
,要提早去医院检查一下,及早干预。祝好运!

【在 c********5 的大作中提到】
: 娃6岁了,一直都是普通娃,有些方面和同龄人比,还delay一年甚至更多。最近回国,
: 大家都说他数学有天分,可以多培养培养。当妈的我第一次有了飘飘然滴赶脚。上来一
: 是想问问,是不是大家忽悠我呢。二是想问问,下一步数学该如何推(无论是不是有天
: 赋)。真心讨论娃的问题,不喜欢,请右上角点X退出。万分谢谢。
: 1.我原本想给娃讲讲高斯定理,娃一看到我出的题,马上的反应是:
: 1+2+3+4+5+6+7+8+9+10
: =10+1+9+2+8+3+7+4+6+5
: =55
: (这个学校里倒是常常要求凑10,也可能有人问过他这道题)
: 2.

B****n
发帖数: 11290
15
關鍵不知道他學過些什麼 如果只是會作最基本的加減乘法
以下這些算法都是自己想出來的 那確實很天才

【在 c********5 的大作中提到】
: 娃6岁了,一直都是普通娃,有些方面和同龄人比,还delay一年甚至更多。最近回国,
: 大家都说他数学有天分,可以多培养培养。当妈的我第一次有了飘飘然滴赶脚。上来一
: 是想问问,是不是大家忽悠我呢。二是想问问,下一步数学该如何推(无论是不是有天
: 赋)。真心讨论娃的问题,不喜欢,请右上角点X退出。万分谢谢。
: 1.我原本想给娃讲讲高斯定理,娃一看到我出的题,马上的反应是:
: 1+2+3+4+5+6+7+8+9+10
: =10+1+9+2+8+3+7+4+6+5
: =55
: (这个学校里倒是常常要求凑10,也可能有人问过他这道题)
: 2.

c********5
发帖数: 207
16
我特意问了儿子好几次,都说是我告诉他的。
我估计应该是他会简单的乘法后(2*4,3*5这类的),我想试一试他,问他9*9等多少
。他没背过乘法口诀,问9*9就是扳手指头数9,18,27,36。。。。(或者是100-10-9
)然后我告诉他我小时候是背乘法口诀的,直接就能说出来是81,然后他就记住了,但
8*8也问过几次了,都要重新算,而且最近都算错了。今天说等于88了。

【在 Z**********1 的大作中提到】
: 他为什么知道9*9=81?
c********5
发帖数: 207
17
我只能知道我和老公教没教过。其他的没办法知道。发帖也想问问是不是美国学校就是
这样的算法,那就没啥可得瑟的了。
娃上prek的时候,我尝试教10以内加法都要气吐血了。后来因为忙,我就放弃了。prek
毕业时候,学校里应该最高是14以内的加减法。后来刚上k时候,有个中国小女孩会100
以内加减,你说我有啥可得瑟的呢。
后来学校的gifted and talented考试,数学应该是满分,我认为的阅读反倒没过,我
当时还不敢相信。翻转呀
所以学校里有限的天才班和为期一个月的中文after school我都不知道数学教了什么,
还有和其他人在一起都学了什么
有一天,有个亲戚和儿子在一个房间玩数学游戏,我和其他亲戚聊天,他们两搞了能有
1个多小时,我就不知道说什么了。把娃搞得对数学异常兴奋。

【在 Q***k 的大作中提到】
: 如果没人教过,肯定是天才了
s**********y
发帖数: 509
18
高斯定理更您这说的好像不是一回事, 小声提醒一句。

-9

【在 c********5 的大作中提到】
: 我特意问了儿子好几次,都说是我告诉他的。
: 我估计应该是他会简单的乘法后(2*4,3*5这类的),我想试一试他,问他9*9等多少
: 。他没背过乘法口诀,问9*9就是扳手指头数9,18,27,36。。。。(或者是100-10-9
: )然后我告诉他我小时候是背乘法口诀的,直接就能说出来是81,然后他就记住了,但
: 8*8也问过几次了,都要重新算,而且最近都算错了。今天说等于88了。

c********5
发帖数: 207
19
sorry,我当时感到一丝丝滴不对。谢谢提醒

【在 l******t 的大作中提到】
: 楼主知道什么是交换绿什么事分配绿吗
t******l
发帖数: 10908
20
美帝公立学校的数学教育的确比较水。。。我给我家大娃四年级开始加点料加到现在七
年级,效果也不是太好,但比没有强。。。所以我现在给 K 班小娃也开始适当加点数
学概念方面的料,总比没有强。。。所以我觉得有条件的话,确实可以在家里给娃的数
学适当加料。

:我只能知道我和老公教没教过。其他的没办法知道。发帖也想问问是不是美国学校就
是这样的算法,那就没啥可得瑟的了。
相关主题
amc 10 的A, B卷有啥区别转载: 从首个IMO季军谈起 by 付云皓
来点美好的时光新加坡数学对小学生帮助大吗?
有些小学四年级的数学题不太会做WISC-V 智商测试
进入Parenting版参与讨论
t******l
发帖数: 10908
21
我觉得只要有心教娃适当加料,具体数学术语技术不是问题,特别是现在网上资源狠丰
富。。。美帝小学初中数学的术语啥的,查一下 http://www.mathsisfun.com 就行。。。初中高中可以查 http://www.intmath.com,或者 http://www.AoPS.com。。。

:sorry,我当时感到一丝丝滴不对。谢谢提醒
t******l
发帖数: 10908
22
sum of arithmetic sequence

:sorry,我当时感到一丝丝滴不对。谢谢提醒
f*******e
发帖数: 3433
23
怪不得,pre-k就推这么狠了。我们这K才要求5以内加减法。算100以内加减反而不难,
我家5岁半竖式加减也差不多会了,刚开始教他make10的解法, 反而你给的2和3难(如
果没人教过的话;从你写的看,你教过和2类似的了)。我觉得对这个年纪小孩,最重要
的是基本功,而不是加减乘除还没学精就学分配律什么的。与其现在教深的东西,不如
把学校要讲的先教会,再教深的,否则路基不稳楼再高也不行。

prek
100

【在 c********5 的大作中提到】
: 我只能知道我和老公教没教过。其他的没办法知道。发帖也想问问是不是美国学校就是
: 这样的算法,那就没啥可得瑟的了。
: 娃上prek的时候,我尝试教10以内加法都要气吐血了。后来因为忙,我就放弃了。prek
: 毕业时候,学校里应该最高是14以内的加减法。后来刚上k时候,有个中国小女孩会100
: 以内加减,你说我有啥可得瑟的呢。
: 后来学校的gifted and talented考试,数学应该是满分,我认为的阅读反倒没过,我
: 当时还不敢相信。翻转呀
: 所以学校里有限的天才班和为期一个月的中文after school我都不知道数学教了什么,
: 还有和其他人在一起都学了什么
: 有一天,有个亲戚和儿子在一个房间玩数学游戏,我和其他亲戚聊天,他们两搞了能有

t******l
发帖数: 10908
24
我同意你说的要“打好娃版数学基础”。。。但我不同意“打好公立学校/课后班里所
教的基础”,因为我觉得这种属于重复基建、浪费资源。。。当然也不可能完全避免重
复基建,但我觉得重复基建是不得已而为之,因为马可夫模型所需要的必须的底线程度
的循序渐进(海空直降也不能太远,否则可能 a bridge too far)。。。但如果把重
复基建作为目标,那我就觉得不值得。

:怪不得,pre-k就推这么狠了。我们这K才要求5以内加减法。算100以内加减反而不难
,我家5岁半竖式加减也差不多会了,刚开始教他make10的解法, 反而你给的2和3难(如
:果没人教过的话;从你写的看,你教过和2类似的了)。我觉得对这个年纪小孩,最重
要的是基本功,而不是加减乘除还没学精就学分配律什么的。与其现在教深的东西,不
如把学校要讲的先教会,再教深的,否则路基不稳楼再高也不行。
f*******e
发帖数: 3433
25
我没说要重复基建啊。学校教的如果小孩都掌握了就往上教,如果没掌握就要补课。家
长教难的之前至少与应该确认小孩基本功已经扎实了再教,循序渐进。但LZ小孩的情况
显然小孩很多基本东西都没学,就往上建高楼了,教学过程过于注重技巧却忽略了理解
的重要性。这个年纪小孩,很多靠记忆。LZ教他9*9=10*10-10-9,小孩当然会套
8*8=9*9-9-8,但却不懂公式的意义是什么。这样教出的小孩看似天才,但实际上是
囫囵吞枣,拔苗助长的结果。
如果分配律不是家长教的,那可能小孩学区亚裔居多或者是中文after school教的,
否则不会kindergarten天才班就学分配率了。而且这家长说的一些我不大相信。
而且把交换律记成高斯定理了,有点匪夷所思,如果这么基础的都记错,家长
就应该放手让学校教。如果chinese afterschool只照高楼不顾根基,就换个
afterschool.

(如

【在 t******l 的大作中提到】
: 我同意你说的要“打好娃版数学基础”。。。但我不同意“打好公立学校/课后班里所
: 教的基础”,因为我觉得这种属于重复基建、浪费资源。。。当然也不可能完全避免重
: 复基建,但我觉得重复基建是不得已而为之,因为马可夫模型所需要的必须的底线程度
: 的循序渐进(海空直降也不能太远,否则可能 a bridge too far)。。。但如果把重
: 复基建作为目标,那我就觉得不值得。
:
: :怪不得,pre-k就推这么狠了。我们这K才要求5以内加减法。算100以内加减反而不难
: ,我家5岁半竖式加减也差不多会了,刚开始教他make10的解法, 反而你给的2和3难(如
: :果没人教过的话;从你写的看,你教过和2类似的了)。我觉得对这个年纪小孩,最重
: 要的是基本功,而不是加减乘除还没学精就学分配律什么的。与其现在教深的东西,不

r*g
发帖数: 3159
26
其实也没那么可怕。
不理解,但是对记规则,利用规则感到舒适,能推的动,那也是一种思维方法。
搞不好这么做,小孩将来是代数思维方式,对抽象的东西越学越快,反而是好事。

【在 f*******e 的大作中提到】
: 我没说要重复基建啊。学校教的如果小孩都掌握了就往上教,如果没掌握就要补课。家
: 长教难的之前至少与应该确认小孩基本功已经扎实了再教,循序渐进。但LZ小孩的情况
: 显然小孩很多基本东西都没学,就往上建高楼了,教学过程过于注重技巧却忽略了理解
: 的重要性。这个年纪小孩,很多靠记忆。LZ教他9*9=10*10-10-9,小孩当然会套
: 8*8=9*9-9-8,但却不懂公式的意义是什么。这样教出的小孩看似天才,但实际上是
: 囫囵吞枣,拔苗助长的结果。
: 如果分配律不是家长教的,那可能小孩学区亚裔居多或者是中文after school教的,
: 否则不会kindergarten天才班就学分配率了。而且这家长说的一些我不大相信。
: 而且把交换律记成高斯定理了,有点匪夷所思,如果这么基础的都记错,家长
: 就应该放手让学校教。如果chinese afterschool只照高楼不顾根基,就换个

f*******e
发帖数: 3433
27
这种是典型国内教法,纯靠记忆。LZ小孩学得过犹不及,不见得是好事,可能向好方向
发展,也可能向差方向发展。 我高中数学一直全班第一,高考满分,大学专业不是
数学。小朋友上K后我才发现我对很多基础东西理解并不行,国内教学适合国内
高考,学完后很多都忘了。 美国学校里注重理解,理解了学得也快,但缺点是速度太
慢。所以我现在结合中美数学教学优势,有时间的时候教教小孩,大部分时间我花在让
小孩
和别的小朋友玩,提高SOCIAL,COMMUNICATION能力,因为不想小孩成为学术
NERD。所以我对那些12岁上大学什么的一点都不感冒。有很强的地方就必有很弱
的地方,关键是小孩长大后是不是能养活自己,同时还能开心快乐

【在 r*g 的大作中提到】
: 其实也没那么可怕。
: 不理解,但是对记规则,利用规则感到舒适,能推的动,那也是一种思维方法。
: 搞不好这么做,小孩将来是代数思维方式,对抽象的东西越学越快,反而是好事。

t******l
发帖数: 10908
28
如果需要给小学娃公立学校数学内容补课的话,那我觉得不要考虑推数学加料了。。。
速度正确率另说,有相当一部分数学先进娃的速度和正确率也不太行。。。当然其实要
给那型数学先进娃补课速度正确率,娃通常也不肯补课,所以还是没法补

:我没说要重复基建啊。学校教的如果小孩都掌握了就往上教,如果没掌握就要补课。
家长教难的之前至少与应该确认小孩基本功已经扎实了再教,循序渐进。但LZ小孩的情
况显然小孩很多基本东西都没学,就往上建高楼了,教学过程过于注重技巧却忽略了理
解的重要性。这个年纪小孩,很多靠记忆。LZ教他9*9=10*10-10-9,小孩当然会套
:8*8=9*9-9-8,但却不懂公式的意义是什么。这样教出的小孩看似天才,但实际上是
t******l
发帖数: 10908
29
关于“过犹不及”这点,我觉得版上通常父母通常娃,都被版上的 BSO 搞成杯弓蛇影
了。。。这也包括我在我家大娃四年级之前。
其实观察一下那些 BSO 就能发现,要么那些娃娃是 Hyper-Numeracy,要么那些父母是
Control-Freak,or both。
而俺乔装打扮深入 pushy mama 微信群侦察的结果就是:对于正常父母正常娃,推到鸡
飞狗跳也推不出那种 BSO 的,更不要说“过犹不及了”。。。pushy mama 微信群里的
正常哥们,你们曾经 “过” 哪怕就一次没?。。。啥 “及” 还是 “不及”,特么
这不都是扯?。。。当然这也包括我自己。
或者这就好比正常下室索男比看完东方不败以后,根本不需要担心自己会不会挥刀自宫
。。。因为一来正常索男根本就没东方不败那把大刀,第二就算有刀正常索男怕疼也下
不了自宫的手。。。当然如果不幸被 pushy mama control freak 在鸡汤里下了麻药的
话,另说。。。不过这个关键是警惕 control freak 在鸡汤里下的麻药,而不是 “过
犹不及” 本身。

:这种是典型国内教法,纯靠记忆。LZ小孩学得过犹不及,不见得是好事,可能向好方
向发展,也可能向差方向发展。 我高中数学一直全班第一,高考满分,大学专业不是
:数学。小朋友上K后我才发现我对很多基础东西理解并不行,国内教学适合国内
c********5
发帖数: 207
30
你太牛了,从我第一个帖子就能看出有轻微的hypernumeracy。
我特意查了一下。A precocious, self-taught ability to read words which
appears before age 5, and/or an intense fascination with letters, numbers,
logos…[accompanied with] significant difficulty in understanding and
developing oral language.”
每一个字都是我娃的真实写照,当然像你说的,是轻微,不是什么大咖。我好长一段时
间都特担心他。运动,表达,理解都是落后同龄人的,1年甚至2年不等。他足球玩了3
个session,我一点不夸张,每次上场都是拖后退的,最后一个session别说进球了,碰
到球的次数一个巴掌都能数过来。我都愁死了。总之狗血的事情太多,以至于上K,我
就放羊。
说出来请大家再帮我看看
但另一方面我又觉得他有超前的地方。他英文环境只是:3岁上的半天daycare,之前一
直在家,都是中文环境,期末老师说他是班上为数不多的几个能认简单单词的娃。4岁
换了蒙校,因为之前的一家太强调academy了。我和儿子都不喜欢。全天上到3点放学。
5岁ESL test, 59%, 5岁半能读magic tree house和magic school bus,我才发现他
好多词能读出来,但是不知道是什么意思。但至少说明phonics基本过关了。以他投入
到英文的时间来看,他还是有点语言天分的。


81

【在 t******l 的大作中提到】
: 楼主娃非常聪明。。。这种情况下最好去测一下智商,如果是 gifted math kid 就找
: gifted class。。。隔壁楼提到了轻微的 hyperlexia,对于万一有轻微的
: hypernumeracy 的情况,测过智商也就知道怎么教娃。
: 我过去目测看猪跑,祖国一般程度的大路货的重点高中的数理校队里,十年一见的娃娃
: ,基本也不会有楼主娃娃这么先进,甚至有可能差的很远。。。所以我觉得楼主描述的
: 情况,八九不离十是超常娃娃。。。而超常娃娃就可能可以解释娃娃为啥知道 9*9=81
: ,而父母不知道娃已经知道,最大的可能超常娃娃某一次 glance 过后就记住了。
:
: :他为什么知道9*9=81?

相关主题
今天和幼儿园老师开会review娃的progress,非常upsetExplaining Your Math: Unnecessary at Best, Encumbering at W
好学生去普通公立高中真的会被耽误吗?是不是任我儿子继续迷象棋
陶天才论天才 (转载)9岁, 如何报名考amc10
进入Parenting版参与讨论
c********5
发帖数: 207
31
我和娃爸都没这么教过娃,但是不能保证其他人,要是真是他自己想出来的,我也认为
很牛。说明他理解乘法的原理了。
我根本就不会这么算。是娃教我的。现在我有时候也这么算一下,感觉挺好玩的。

【在 s**********y 的大作中提到】
: 9* 9 - 9 - 8 能够不是从头算而得到 8 * 8 对三年级娃已属不易。能猜到这一关
: 系, 就像潮老大说的, 绝对超常哈。

c********5
发帖数: 207
32
再次谢谢你的建议,智商我打算去测一下。
请问测智商,和年龄,还有英语水平有关系吗?是随时测,还是刚好周岁的时候去测?
这玩意毕竟不便宜。
隔壁楼讨论hyperlexia,能给个链接吗?
谢谢


81

【在 t******l 的大作中提到】
: 楼主娃非常聪明。。。这种情况下最好去测一下智商,如果是 gifted math kid 就找
: gifted class。。。隔壁楼提到了轻微的 hyperlexia,对于万一有轻微的
: hypernumeracy 的情况,测过智商也就知道怎么教娃。
: 我过去目测看猪跑,祖国一般程度的大路货的重点高中的数理校队里,十年一见的娃娃
: ,基本也不会有楼主娃娃这么先进,甚至有可能差的很远。。。所以我觉得楼主描述的
: 情况,八九不离十是超常娃娃。。。而超常娃娃就可能可以解释娃娃为啥知道 9*9=81
: ,而父母不知道娃已经知道,最大的可能超常娃娃某一次 glance 过后就记住了。
:
: :他为什么知道9*9=81?

f*******e
发帖数: 3433
33
我就是前面说我娃有hyperlexia的,就是那种photographical memory,认字写字能力
超强,但阅读理解能力弱。很多researcher认为有hyperlexia的都有自闭,包括高功能
自闭。当然,这本身也是长处,父母不应该忽略了。这种情况的教育方法是扬长推短,
充分发挥长处,让娃长处自由发展。短处努力提高,不求做得很好,但求达到平均水平
。我家的经过一年训练后,语言能力测评达到中等偏上,理解能力和社交能力也向中等
靠近。对这种娃,教育方法很重要,否则长大容易变美国人口中的NERD.

3

【在 c********5 的大作中提到】
: 你太牛了,从我第一个帖子就能看出有轻微的hypernumeracy。
: 我特意查了一下。A precocious, self-taught ability to read words which
: appears before age 5, and/or an intense fascination with letters, numbers,
: logos…[accompanied with] significant difficulty in understanding and
: developing oral language.”
: 每一个字都是我娃的真实写照,当然像你说的,是轻微,不是什么大咖。我好长一段时
: 间都特担心他。运动,表达,理解都是落后同龄人的,1年甚至2年不等。他足球玩了3
: 个session,我一点不夸张,每次上场都是拖后退的,最后一个session别说进球了,碰
: 到球的次数一个巴掌都能数过来。我都愁死了。总之狗血的事情太多,以至于上K,我
: 就放羊。

d******e
发帖数: 2265
34
拼出单词和数学都是简化和推理能力强。
你家孩子方向感如何。
感觉学代数,物理时可能有优势吧。
不过这个也好像没大用。
做个马公,物理学家,和数学家也不算好出路。
等8,9岁丢一本趣味数学吧。

3

【在 c********5 的大作中提到】
: 你太牛了,从我第一个帖子就能看出有轻微的hypernumeracy。
: 我特意查了一下。A precocious, self-taught ability to read words which
: appears before age 5, and/or an intense fascination with letters, numbers,
: logos…[accompanied with] significant difficulty in understanding and
: developing oral language.”
: 每一个字都是我娃的真实写照,当然像你说的,是轻微,不是什么大咖。我好长一段时
: 间都特担心他。运动,表达,理解都是落后同龄人的,1年甚至2年不等。他足球玩了3
: 个session,我一点不夸张,每次上场都是拖后退的,最后一个session别说进球了,碰
: 到球的次数一个巴掌都能数过来。我都愁死了。总之狗血的事情太多,以至于上K,我
: 就放羊。

t******l
发帖数: 10908
35
:做个马公,物理学家,和数学家也不算好出路。
哥们您这是要生物千老感觉森森悲愤么?// run

【在 d******e 的大作中提到】
: 拼出单词和数学都是简化和推理能力强。
: 你家孩子方向感如何。
: 感觉学代数,物理时可能有优势吧。
: 不过这个也好像没大用。
: 做个马公,物理学家,和数学家也不算好出路。
: 等8,9岁丢一本趣味数学吧。
:
: 3

c********5
发帖数: 207
36
大家好像很纠结高斯定理,我就多说几句
我娃哪能真正理解伟大的高斯定理呢,他其实是最淳朴的算法
1+2+3+4+5+6+7+8+9+10
=(10+1)+2+3+4+5+6+7+8+9
=(11+9)+2+3+4+5+6+7+8
=(20+2)+3+4+5+6+7+8
=(22+8)+3+4+5+6+7
=(30+3)+3+4+5+6
=(33+7)+4+5+6
=(40+4)+5+6
=(44+6)+5
=(50+5)
=55
然后我想1+2+。。。。。+100是不是也可以引入了。结果娃就崩溃了。哈哈哈,后
来我又找机会尝试1+2+。。。。。+20发现娃也算不对。我的感觉是娃吃透了10以内数
之间的关系而已。
f*******e
发帖数: 3433
37
研究显示,对这类小孩马工,工程师,和计算机,工程方面的教职工作很适合,成功率
高,而且有得天独厚优势。数学,无力也有到华尔街工作的,也不错。学那些对社交要
求比较高的专业的,找工作失败率高。

【在 d******e 的大作中提到】
: 拼出单词和数学都是简化和推理能力强。
: 你家孩子方向感如何。
: 感觉学代数,物理时可能有优势吧。
: 不过这个也好像没大用。
: 做个马公,物理学家,和数学家也不算好出路。
: 等8,9岁丢一本趣味数学吧。
:
: 3

t******l
发帖数: 10908
38
楼上 flyingpie 回答了你的问题。。。而我回答不了你的问题,因为我家小娃虽然在
幼儿园里的 comments 算是比较先进的娃,但她是个普通娃,不符合任何这些指标。
阅读方面:
我家小娃现在 6 岁,她的 phonics 差不多就是该年级的正常水平。虽然小娃她对
阅读有通常小孩的兴趣,但跟通常小孩一样,她对娃版教育节目兴趣更大。。。但
另一方面,而娃版教育 video 的 non-interactive 的方式,对通常小娃影响不大
。。。因为通常小娃还有 attention span 调节,或者 “一边XX一边YY”,也不太
容易到过犹不及的程度。
记忆方面:
我家小娃虽然在幼儿园里被夸记忆力很好,但实际上她还是经常忘记。。。这并不
是说小娃她的记忆力有啥问题,而是说通常小孩的大脑是被布线成忘记大脑认为不
重要的东西,或者暂时无法理解的东西(大脑内部的马工循环校验码一定要 pass,
否则就不是数学阅读学习的问题,这可能有精神分裂症的风险了)。。。这导致通常
小娃都是采取 spiral 的学习方式。。。而且通常小娃会抵抗简单重复的练习,宁可
遗忘以后再学习。。。所以通常小娃学习上的对付理解和对付遗忘,实际上是同一个
方向。
数字/字母/单词等等符号的 attention span 方面:
通常 6 岁小娃,以及 12 岁大娃,在纯符号方面的 attention span 非常短。为了
学习数学的目的,最简单的增加 attention span 的方法是大人一起 joint attention
span。。。因为这么短的 attention span,对于通常娃而言,数学/文字方面的理解
永远会领先于记忆,这个是 nature,不需要特别干预。。。而另一方面,这也是及早
引入 “娃版 foundation of mathematics”,由理解拉动记忆的正常途径。。。所以
娃娃都会倾向于先了解 “娃版 foundation of mathematics”,抗拒 Kumon 也不会
抗拒 “娃版 foundation of mathematics”。
Piaget 老先生理论里的学习阶段:
通常娃基本符合 Piaget 老先生的学习阶段理论,也通常是建立了 conserve /
reverse / mathematical-identity (比如 additive identity "zero") 之后,
然后从 concrete operation 进入 formal operation,这种通常教育学道路。。。
当然,在数学方面,所谓的 formal operation 其实是 concrete operation
on complex mathematical structure。。。所以通常娃的教育道路,一个字:
“抄”。。。再加一个字:“抄 Piaget 老先生的论文”。

3

【在 c********5 的大作中提到】
: 你太牛了,从我第一个帖子就能看出有轻微的hypernumeracy。
: 我特意查了一下。A precocious, self-taught ability to read words which
: appears before age 5, and/or an intense fascination with letters, numbers,
: logos…[accompanied with] significant difficulty in understanding and
: developing oral language.”
: 每一个字都是我娃的真实写照,当然像你说的,是轻微,不是什么大咖。我好长一段时
: 间都特担心他。运动,表达,理解都是落后同龄人的,1年甚至2年不等。他足球玩了3
: 个session,我一点不夸张,每次上场都是拖后退的,最后一个session别说进球了,碰
: 到球的次数一个巴掌都能数过来。我都愁死了。总之狗血的事情太多,以至于上K,我
: 就放羊。

t******l
发帖数: 10908
39
通常娃如果要教 sum of arithmetic sequence 的话,在教育学上的前提是
娃能够 (1) conserve, (2) reverse (Piaget 教育学术语),再加上一个
mathematical "indexing" (matching)。
其实我不严格地测了一下 6 岁通常小娃,其实 6 岁通常小娃上面那些都能做。
但是 sum of arithmetic sequence 想都不要想,我测试了一下教她理解
division is the "reverse" of multiplication,这都还不行。。。原因
我一看就知道,困难是并不是在 conserve / reverse / indexing 这些
concrete operation 本身,而是要把这些 operation 放在 “娃版
mathematical structure”上操作,而这 structure 上面还有一个头疼的
operator 叫 "any"。。。难倒 6 岁通常小娃了。
我给大娃从四年级开始加料一些数学,一直加料到六年级快结束的时候,她
终于有一天说她会(明白概念的意思,基本是要算错的) sum of geometric
sequence 了,这时我觉得大娃她 可能才是理解了 sum of arithmetic
sequence 了。。。而这时大娃她都已经会 complete a square 了。
所以我觉得对通常娃的瓶颈,其实是在于 “娃版 mathematical structure
with operator "any"”。。。这玩意儿对通常娃并不容易,一般在 ready
后需要三年我觉得。。。这玩意儿可能还需要慢火,欲速可能会不达。。。

【在 c********5 的大作中提到】
: 大家好像很纠结高斯定理,我就多说几句
: 我娃哪能真正理解伟大的高斯定理呢,他其实是最淳朴的算法
: 1+2+3+4+5+6+7+8+9+10
: =(10+1)+2+3+4+5+6+7+8+9
: =(11+9)+2+3+4+5+6+7+8
: =(20+2)+3+4+5+6+7+8
: =(22+8)+3+4+5+6+7
: =(30+3)+3+4+5+6
: =(33+7)+4+5+6
: =(40+4)+5+6

t******l
发帖数: 10908
40
补充说一句:我上面说的是“通常娃”,也就是“普通娃”。。。天才娃可能可以
“直接海空直降在敌军司令部”,不一定需要这个 n 年的过程。。。但普通娃
没办法,欲速可能不达,慢火可能是必须的。。。
相关主题
9岁, 如何报名考amc10二年级女儿的report
有明天考AMC 8的吗?觉得美帝数学进度慢的进来看一下
[转载] 为什么说usamo 简单我招,我是猪
进入Parenting版参与讨论
d******e
发帖数: 2265
41
直接海空直降在敌军司令部 有什么用?
当第二个老张吗?
真不如跟周恩来学,小小年纪就去跑生活,练就一身社交本领。

【在 t******l 的大作中提到】
: 补充说一句:我上面说的是“通常娃”,也就是“普通娃”。。。天才娃可能可以
: “直接海空直降在敌军司令部”,不一定需要这个 n 年的过程。。。但普通娃
: 没办法,欲速可能不达,慢火可能是必须的。。。

t******l
发帖数: 10908
42
:以至于上K,我就放羊。
这个问题对于通常娃(也就是普通娃)而言,放不放都是羊,因为上帝放了一条细细
的红线,普通父母普通鸡飞狗跳是根本跨不过那条红线的。。。比如一条红线就是
attention span,另一条红线就是 “扔 Kumon 习题集”。。。当然我家领导老是梦想
跨过那条看不见的红线,我都不希得说你特么这是跟上帝过不去啊。。。至于一脚绊倒
上帝的成功率为多少吧,自己看!

3

【在 c********5 的大作中提到】
: 你太牛了,从我第一个帖子就能看出有轻微的hypernumeracy。
: 我特意查了一下。A precocious, self-taught ability to read words which
: appears before age 5, and/or an intense fascination with letters, numbers,
: logos…[accompanied with] significant difficulty in understanding and
: developing oral language.”
: 每一个字都是我娃的真实写照,当然像你说的,是轻微,不是什么大咖。我好长一段时
: 间都特担心他。运动,表达,理解都是落后同龄人的,1年甚至2年不等。他足球玩了3
: 个session,我一点不夸张,每次上场都是拖后退的,最后一个session别说进球了,碰
: 到球的次数一个巴掌都能数过来。我都愁死了。总之狗血的事情太多,以至于上K,我
: 就放羊。

t******l
发帖数: 10908
43
但人也有天性不是?你要是把张曼玉跟周恩来对调一下,估计两个都更苦鼻。。。

【在 d******e 的大作中提到】
: 直接海空直降在敌军司令部 有什么用?
: 当第二个老张吗?
: 真不如跟周恩来学,小小年纪就去跑生活,练就一身社交本领。

d******e
发帖数: 2265
44
我感觉还是要锻炼。周公当年少年就独立撑起一个家,四处陪笑当债。这才锻炼出来将
来的功夫。
现在,我外甥都高中了,都不敢独自去北京站接人,说手机都被抢了好几个了。
这样子,将来走上社会,得适应多长时间?
人家20岁久扬长而去了,你30岁才成熟。什么赢在起跑线也白搭。会点数学魔用啊。
美国也一样,现在都是温室里面长大的。

【在 t******l 的大作中提到】
: 但人也有天性不是?你要是把张曼玉跟周恩来对调一下,估计两个都更苦鼻。。。
t******l
发帖数: 10908
45
我觉得还是要有至少底线的天资吧。。。否则既被抢手机一辈子,也还不会数学,麦当
劳翻烧饼也没耐力,那就是真的被父母推成炮灰了。
老实说成王败寇也是要有天资的,你就说范冰冰多打排球能不能成为朱婷吧。
当然话说回来,外甥又不是自家娃,被炮灰了也不关自家的事。。。关键还是自个儿在
机场最好要有人接,人之常情了。

:我感觉还是要锻炼。周公当年少年就独立撑起一个家,四处陪笑当债。这才锻炼出来
将来的功夫。
:现在,我外甥都高中了,都不敢独自去北京站接人,说手机都被抢了好几个了。
c********5
发帖数: 207
46
我之前教他5以内的加减法,他不爱搭理我,超过7基本是要错的。有一次playdate,朋
友问她4+5=?他努力滴看了半天手指头,说出了答案,把我还大大的吃了一惊。我之
前肯定是没问过这么难的。又过了1.2个月,我尝试着推10以内加减法,搞得很不愉快
。加上知道K要求5以内的加减法,我就没推过数学了。当然他问我,我会回答,有时候
也会启发一下。娃爸有时候倒是喜欢教娃数学,但也不是很多。他4岁的蒙校,带回来
的作业,最多做到14以内的加减法。身边的同龄娃们都100以内竖式计算了。我好生羡
慕别人家的娃呀。
你说的很对,我也认为基本功很对,尤其是加减法,基础中的基础。这也是我后来不推
的原因之一。我还特意和中文学校说不上数学课,换成其他的课。如果娃都会了,在学
校里得多无聊呀。可惜老师不是很配合。最最最可怕的是半瓶子乱得瑟。说不会还懂点
。这其实非常可怕,还不如按部就班的来,放长线钓大鱼。

【在 f*******e 的大作中提到】
: 怪不得,pre-k就推这么狠了。我们这K才要求5以内加减法。算100以内加减反而不难,
: 我家5岁半竖式加减也差不多会了,刚开始教他make10的解法, 反而你给的2和3难(如
: 果没人教过的话;从你写的看,你教过和2类似的了)。我觉得对这个年纪小孩,最重要
: 的是基本功,而不是加减乘除还没学精就学分配律什么的。与其现在教深的东西,不如
: 把学校要讲的先教会,再教深的,否则路基不稳楼再高也不行。
:
: prek
: 100

c********5
发帖数: 207
47
9*9=10*10-10-9这是娃教我的(10*10我教过娃,这个我承认),我开始还没听懂,但
他唠叨半天说对答案,才引起我的注意,我是当趣事说给娃爸听的。
我其实一直想启发娃背小九九的。认为他不懂,趁着小记忆力好,先背了再说。但看他
这么算之后,我就放弃了,让他自己体会数学的美吧。
小九九不会错呀,他这么搞来搞去,出错的比例还是很高的。
我虽然不才,但高斯定理还是知道的。

【在 f*******e 的大作中提到】
: 我没说要重复基建啊。学校教的如果小孩都掌握了就往上教,如果没掌握就要补课。家
: 长教难的之前至少与应该确认小孩基本功已经扎实了再教,循序渐进。但LZ小孩的情况
: 显然小孩很多基本东西都没学,就往上建高楼了,教学过程过于注重技巧却忽略了理解
: 的重要性。这个年纪小孩,很多靠记忆。LZ教他9*9=10*10-10-9,小孩当然会套
: 8*8=9*9-9-8,但却不懂公式的意义是什么。这样教出的小孩看似天才,但实际上是
: 囫囵吞枣,拔苗助长的结果。
: 如果分配律不是家长教的,那可能小孩学区亚裔居多或者是中文after school教的,
: 否则不会kindergarten天才班就学分配率了。而且这家长说的一些我不大相信。
: 而且把交换律记成高斯定理了,有点匪夷所思,如果这么基础的都记错,家长
: 就应该放手让学校教。如果chinese afterschool只照高楼不顾根基,就换个

f*******e
发帖数: 3433
48
看来是他学的这些东西是chinese afterschool 老师教的。可能很多中国推妈想让
afterschool 交数学,加上after school年级混杂,所以才教的这么没系统。 这种情
况没必要当一回事,还有那个什么竖式加法100的,如果有人孜孜不倦的教,平均智
商的6岁小孩都能学会,只是很多人没这么教罢了。而且起步早不等于以后也强,真没
什么好羡慕的。以后职业发展和这完全不挂勾。我读高中的时候看着很多小学的优等生
成绩哗哗往下掉,成了强弩之末,而我这小学老师说考初中都成问题的却成了班上
第一。我是除了正规老师和自己,不会让别人来叫我娃的,如果after school找那些
半瓶子水的教,我肯定转到别的after school去。
国内叫1+2+...+n = n(n+1)/2高斯定律或高斯公式。高斯定理通常指另一个复杂
很多的定理。

【在 c********5 的大作中提到】
: 9*9=10*10-10-9这是娃教我的(10*10我教过娃,这个我承认),我开始还没听懂,但
: 他唠叨半天说对答案,才引起我的注意,我是当趣事说给娃爸听的。
: 我其实一直想启发娃背小九九的。认为他不懂,趁着小记忆力好,先背了再说。但看他
: 这么算之后,我就放弃了,让他自己体会数学的美吧。
: 小九九不会错呀,他这么搞来搞去,出错的比例还是很高的。
: 我虽然不才,但高斯定理还是知道的。

c********5
发帖数: 207
49
我又想起来一件事(说这么多,想让大家帮我分析分析娃,和如何引导娃。如果需要干
预,我肯定会干预的,),
娃很喜欢玩计算器,因为回国,玩具有限,他有时一天甚至玩2,3个小时计算器,就是
各种按。但最多时候是选一个数,然后就一直+1,一直按,乐此不疲。他问我为什么
一个数,按不同的键,数不一样呀。我就给他讲了什么是八进制,二进制,十六进制。
我也没寄什么希望他能理解。然后他就开始自己按计
算器了,然后他告诉我,有字母abcdef。大概两周以后,我问他3的2进制是什么,4呢
,5呢。20的16进制是什么,他都答对了。然后他又说如果是7进制,怎么怎么样,5进
制怎么怎么样,我们谈论了半天有没有一进制。我想他应该是理解了。这有点太先进了
吧。
我儿子对我朋友说,你看你按这个键,数变了,再按另一个键,它又变回来了。
我没注意,但我朋友后来说,我儿当时的两眼放光,想发现了宝藏一样。我不知道是什
么,但是我猜应该是10的方倍和开根号
他K的时候,问他数学是从哪学的,他一概回答,我知道,我就是知道呀。看来很可能
是计算器的功劳。真未必是学校教的。
不过我真没做好他往数学方面发展的心里准备,他如果以后在数学界混的不上不下,这
玩应能自己养活自己,和老婆孩子吗?如果不能,还不如做码工呢。
我觉得娃感兴趣的东西有些能记得很清楚,但记忆力真不咋样。
代数思维的好处是什么?

【在 r*g 的大作中提到】
: 其实也没那么可怕。
: 不理解,但是对记规则,利用规则感到舒适,能推的动,那也是一种思维方法。
: 搞不好这么做,小孩将来是代数思维方式,对抽象的东西越学越快,反而是好事。

c********5
发帖数: 207
50
我很同意你“提高SOCIAL,COMMUNICATION能力”,“小孩长大后是不是能养活自己,
同时还能开心快乐”
我的深刻体会是,娃不ready,根本推不动。能把人累死气死。一旦ready,分分钟搞定。
12岁上大学,绝大部分是自推的聪明娃。推基本是推不出来滴。当然极端情况,家长放
弃一切,专心推也不是不可能滴。

【在 f*******e 的大作中提到】
: 这种是典型国内教法,纯靠记忆。LZ小孩学得过犹不及,不见得是好事,可能向好方向
: 发展,也可能向差方向发展。 我高中数学一直全班第一,高考满分,大学专业不是
: 数学。小朋友上K后我才发现我对很多基础东西理解并不行,国内教学适合国内
: 高考,学完后很多都忘了。 美国学校里注重理解,理解了学得也快,但缺点是速度太
: 慢。所以我现在结合中美数学教学优势,有时间的时候教教小孩,大部分时间我花在让
: 小孩
: 和别的小朋友玩,提高SOCIAL,COMMUNICATION能力,因为不想小孩成为学术
: NERD。所以我对那些12岁上大学什么的一点都不感冒。有很强的地方就必有很弱
: 的地方,关键是小孩长大后是不是能养活自己,同时还能开心快乐

相关主题
大学录取率来点美好的时光
学校早晚要教的东西,早早学会了又如何?有些小学四年级的数学题不太会做
amc 10 的A, B卷有啥区别转载: 从首个IMO季军谈起 by 付云皓
进入Parenting版参与讨论
f*******e
发帖数: 3433
51
6岁小孩教二进制,八进制,十六进制。。。我都懒得说你这当妈的了。

【在 c********5 的大作中提到】
: 我又想起来一件事(说这么多,想让大家帮我分析分析娃,和如何引导娃。如果需要干
: 预,我肯定会干预的,),
: 娃很喜欢玩计算器,因为回国,玩具有限,他有时一天甚至玩2,3个小时计算器,就是
: 各种按。但最多时候是选一个数,然后就一直+1,一直按,乐此不疲。他问我为什么
: 一个数,按不同的键,数不一样呀。我就给他讲了什么是八进制,二进制,十六进制。
: 我也没寄什么希望他能理解。然后他就开始自己按计
: 算器了,然后他告诉我,有字母abcdef。大概两周以后,我问他3的2进制是什么,4呢
: ,5呢。20的16进制是什么,他都答对了。然后他又说如果是7进制,怎么怎么样,5进
: 制怎么怎么样,我们谈论了半天有没有一进制。我想他应该是理解了。这有点太先进了
: 吧。

f*******e
发帖数: 3433
52
“ 12岁上大学,绝大部分是自推的聪明娃。推基本是推不出来滴。当然极端情况,家
长放
弃一切,专心推也不是不可能滴。”
美国报道了很多这样的,都是home school, 上了个没听过的大学,就上报纸了,印度
裔居多。还有个吹自己小孩上了斯坦福的,结果发现是斯坦福online education. 这
年头BSO的太多了。你也别老想自己小孩是不是数学天才了。如果是数学天才,自己会
自推,不会被埋没的。你这样无来由的教6岁小孩高斯定律,教2进制,8进制,就算是
天才也会被教没的。你小孩是不是天才我不知道,但这当妈的绝对有问题。
c********5
发帖数: 207
53
我娃的加减法速度和正确率,其实是我最大的心病。我很担心他半懂不懂,还自认为自
己很厉害。让他补速度和正确率,估计又要鸡飞狗跳了。我都不敢想像那个画面。
我娃喜欢自由的学,不喜欢坐下来,老师讲或教。原本他有兴趣画画,我就找了老师,
半年不到他就不想学了。现在几乎就不画了。我还难敢推呀。小人消极怠工的本事我算
是领教了。那真是一级棒呀。

【在 t******l 的大作中提到】
: 如果需要给小学娃公立学校数学内容补课的话,那我觉得不要考虑推数学加料了。。。
: 速度正确率另说,有相当一部分数学先进娃的速度和正确率也不太行。。。当然其实要
: 给那型数学先进娃补课速度正确率,娃通常也不肯补课,所以还是没法补
:
: :我没说要重复基建啊。学校教的如果小孩都掌握了就往上教,如果没掌握就要补课。
: 家长教难的之前至少与应该确认小孩基本功已经扎实了再教,循序渐进。但LZ小孩的情
: 况显然小孩很多基本东西都没学,就往上建高楼了,教学过程过于注重技巧却忽略了理
: 解的重要性。这个年纪小孩,很多靠记忆。LZ教他9*9=10*10-10-9,小孩当然会套
: :8*8=9*9-9-8,但却不懂公式的意义是什么。这样教出的小孩看似天才,但实际上是

l*********e
发帖数: 387
54
好奇一下,6岁的孩子,还会在什么场合听到过十六进制?

【在 c********5 的大作中提到】
: 我又想起来一件事(说这么多,想让大家帮我分析分析娃,和如何引导娃。如果需要干
: 预,我肯定会干预的,),
: 娃很喜欢玩计算器,因为回国,玩具有限,他有时一天甚至玩2,3个小时计算器,就是
: 各种按。但最多时候是选一个数,然后就一直+1,一直按,乐此不疲。他问我为什么
: 一个数,按不同的键,数不一样呀。我就给他讲了什么是八进制,二进制,十六进制。
: 我也没寄什么希望他能理解。然后他就开始自己按计
: 算器了,然后他告诉我,有字母abcdef。大概两周以后,我问他3的2进制是什么,4呢
: ,5呢。20的16进制是什么,他都答对了。然后他又说如果是7进制,怎么怎么样,5进
: 制怎么怎么样,我们谈论了半天有没有一进制。我想他应该是理解了。这有点太先进了
: 吧。

l*********e
发帖数: 387
55
你娃数学很有天分,你却不知道该怎么推,真让人着急
要么自己赤膊上阵,网上各种资料都有
要么上课外数学班,各种各样的,新加坡数学啊,北京数学啊,俄罗斯数学啊
这些课外班都有快班慢班数学队儿啥的
2年级就可以参加各种数学竞赛了
然后就在数学竞赛这条路上往下走吧

【在 c********5 的大作中提到】
: 我娃的加减法速度和正确率,其实是我最大的心病。我很担心他半懂不懂,还自认为自
: 己很厉害。让他补速度和正确率,估计又要鸡飞狗跳了。我都不敢想像那个画面。
: 我娃喜欢自由的学,不喜欢坐下来,老师讲或教。原本他有兴趣画画,我就找了老师,
: 半年不到他就不想学了。现在几乎就不画了。我还难敢推呀。小人消极怠工的本事我算
: 是领教了。那真是一级棒呀。

t******l
发帖数: 10908
56
http://www.coolmath.com/prealgebra/06-properties
http://www.coolmath.com/prealgebra

:我娃的加减法速度和正确率,其实是我最大的心病。我很担心他半懂不懂,还自认为
自己很厉害。让他补速度和正确率,估计又要鸡飞狗跳了。我都不敢想像那个画面。
t******l
发帖数: 10908
57
AMC 10 以前的数学竞赛都是民科玩玩的。
AIME 以前不算真正的数学竞赛。
USAMO 才是真正的数学竞赛,入门级。

:你娃数学很有天分,你却不知道该怎么推,真让人着急
:要么自己赤膊上阵,网上各种资料都有
t******l
发帖数: 10908
58
当然我家普通娃就不搞啥真的数学竞赛了,当个提升兴趣的工具学点实用数学就好了。
t******l
发帖数: 10908
59
楼主家娃有可能真的能搞数学竞赛。。。这个跟现在的数学水平是不是超前关系不大,
关键而是 hypernumeracy 有可能将来的 ceiling 很高(至少存在可能性)。。。而一
般的普通娃到高中时,再牛掰一般也就最多 AIME 搞定前 8 题,然后大概率的基本歇
菜。
当然话说回来,数学竞赛没搞成的炮灰也很多,毕竟每年美国队总共也就几号人。。。

:你娃数学很有天分,你却不知道该怎么推,真让人着急
:要么自己赤膊上阵,网上各种资料都有
f*******e
发帖数: 3433
60
就像有ASPERGER的都是天生的程序员。硅谷现在ASPERGER小孩人数巨增,都是遗传了程
序员的。

【在 t******l 的大作中提到】
: 楼主家娃有可能真的能搞数学竞赛。。。这个跟现在的数学水平是不是超前关系不大,
: 关键而是 hypernumeracy 有可能将来的 ceiling 很高(至少存在可能性)。。。而一
: 般的普通娃到高中时,再牛掰一般也就最多 AIME 搞定前 8 题,然后大概率的基本歇
: 菜。
: 当然话说回来,数学竞赛没搞成的炮灰也很多,毕竟每年美国队总共也就几号人。。。
:
: :你娃数学很有天分,你却不知道该怎么推,真让人着急
: :要么自己赤膊上阵,网上各种资料都有

相关主题
新加坡数学对小学生帮助大吗?好学生去普通公立高中真的会被耽误吗?
WISC-V 智商测试陶天才论天才 (转载)
今天和幼儿园老师开会review娃的progress,非常upsetExplaining Your Math: Unnecessary at Best, Encumbering at W
进入Parenting版参与讨论
l*********e
发帖数: 387
61
入门级竞赛的,都是从民科玩上来的

【在 t******l 的大作中提到】
: AMC 10 以前的数学竞赛都是民科玩玩的。
: AIME 以前不算真正的数学竞赛。
: USAMO 才是真正的数学竞赛,入门级。
:
: :你娃数学很有天分,你却不知道该怎么推,真让人着急
: :要么自己赤膊上阵,网上各种资料都有

t******l
发帖数: 10908
62
民科级谁都可以玩,但是我等普通娃大概率的最后玩不上去,所以还是悠着点。。。毕
竟一脚绊倒上帝不是那么容易的干活。。。

:入门级竞赛的,都是从民科玩上来的
l*********e
发帖数: 387
63
楼主的娃,貌似可不是普通娃。。。。

【在 t******l 的大作中提到】
: 民科级谁都可以玩,但是我等普通娃大概率的最后玩不上去,所以还是悠着点。。。毕
: 竟一脚绊倒上帝不是那么容易的干活。。。
:
: :入门级竞赛的,都是从民科玩上来的

t******l
发帖数: 10908
64
硅谷赚钱商业程序猿里,还是实用程序猿更多一些吧。。。理论数学系 pure math 之
Costco 大西瓜啥的,那种概率才更高些?

:就像有ASPERGER的都是天生的程序员。硅谷现在ASPERGER小孩人数巨增,都是遗传了
程序员的。
t******l
发帖数: 10908
65
属实


:楼主的娃,貌似可不是普通娃。。。。
f*******e
发帖数: 3433
66
数学系的应该是楼主娃这样的多些吧。编程也不是随便人都能编好的。
很多好的程序员都有ASPERGER。我看过很多报道,说马工是有
ASPERGER的最好职业之一。我娃诊断是ASPERGER,
是很轻的,医生觉得过一两年可能OUTGROW的那种。
当时我顺口问了一下医生ASPERGER什么职业好,她不假思索
说计算机。所以我特意搜索了一下ASPERGER的职业。



【在 t******l 的大作中提到】
: 硅谷赚钱商业程序猿里,还是实用程序猿更多一些吧。。。理论数学系 pure math 之
: Costco 大西瓜啥的,那种概率才更高些?
:
: :就像有ASPERGER的都是天生的程序员。硅谷现在ASPERGER小孩人数巨增,都是遗传了
: 程序员的。

t******l
发帖数: 10908
67
逆命题不成立吧。。。马工需求量这么大,世上哪有那么多阿斯博格?。。。你们高中
有谁见过阿斯博格哪怕就一个?反正我是连猪跑都没见过。
而且马工取决于工种,很多时候要唬客户开心。。。阿斯博格码 code 也太认真了,不
能干离前台较近的混日子型马工。

:数学系的应该是楼主娃这样的多些吧。优秀马工是ASPERGER的多。编程也不是随便人
都能编好的。很多好的程序员都有ASPERGER。我看过很多报道,说马工是有ASPERGER的
最好职业之一。
f*******e
发帖数: 3433
68
我指的是编程能力强的马工。 APSPERGER的BRAIN适合编程。国内实际上很多ASPERGER,
只不过国内不诊断ASPERGER, 只有低功能自闭才算自闭。

【在 t******l 的大作中提到】
: 逆命题不成立吧。。。马工需求量这么大,世上哪有那么多阿斯博格?。。。你们高中
: 有谁见过阿斯博格哪怕就一个?反正我是连猪跑都没见过。
: 而且马工取决于工种,很多时候要唬客户开心。。。阿斯博格码 code 也太认真了,不
: 能干离前台较近的混日子型马工。
:
: :数学系的应该是楼主娃这样的多些吧。优秀马工是ASPERGER的多。编程也不是随便人
: 都能编好的。很多好的程序员都有ASPERGER。我看过很多报道,说马工是有ASPERGER的
: 最好职业之一。
: :

t******l
发帖数: 10908
69
属实

:我指的是编程能力强的马工。 APSPERGER的BRAIN适合编程
t******l
发帖数: 10908
70
这跟国内是不是诊断阿斯博格没关系,你就看一眼高中数理校队前五名就行了,化学生
物啥的不用看。。。反正我没见过一个阿斯博格。
当然阿斯博格大脑适合编程这点没错。。。我的意思是阿斯博格在数量上,肯定不能围
歼普通大众的。

:我指的是编程能力强的马工。 APSPERGER的BRAIN适合编程。国内实际上很多
ASPERGER,
:只不过国内不诊断ASPERGER, 只有低功能自闭才算自闭。
相关主题
Explaining Your Math: Unnecessary at Best, Encumbering at W有明天考AMC 8的吗?
是不是任我儿子继续迷象棋[转载] 为什么说usamo 简单
9岁, 如何报名考amc10二年级女儿的report
进入Parenting版参与讨论
f*******e
发帖数: 3433
71
这个我同意。
t******l
发帖数: 10908
72
这个普通家长也没辙,我家六岁小娃的计算器是 CASIO fx-991EX,按键显示对小娃而
言够花哨了。。。但小娃也就三分钟热度玩两下后就不玩了。。。当然本来就是当个三
分钟热度的玩具买的。

:我又想起来一件事(说这么多,想让大家帮我分析分析娃,和如何引导娃。如果需要
干预,我肯定会干预的,),
f*******e
发帖数: 3433
73
我家也是。过于专注一个东西也是自闭的诊断之一。我家这条不符合,最后只达到了
ASPERGER最低的诊断要求。

【在 t******l 的大作中提到】
: 这个普通家长也没辙,我家六岁小娃的计算器是 CASIO fx-991EX,按键显示对小娃而
: 言够花哨了。。。但小娃也就三分钟热度玩两下后就不玩了。。。当然本来就是当个三
: 分钟热度的玩具买的。
:
: :我又想起来一件事(说这么多,想让大家帮我分析分析娃,和如何引导娃。如果需要
: 干预,我肯定会干预的,),
: :

t******l
发帖数: 10908
74
只满足最低要求的,将来长大后不一定是阿斯博格。。。如果将来不是阿斯博格的话,
做对 AIME 超过 8 题的难度还是很大的。// run
当然我们这种普通娃就不要想了,幼儿园那么多老师眼皮底下混的。

:我家也是。过于专注一个东西也是自闭的诊断之一。我家这条不符合,最后只达到了
:ASPERGER最低的诊断要求。
f*******e
发帖数: 3433
75
别跑啊:-)我不打算推数学竞赛。如果他最后OUTGROW了ASPERGER,social
communication上去了我更高兴。聪明的普通娃活得最开心。没有谁为了赢数学竞赛想
让自己娃成ASPERGER或者HYPERXXX的吧。

【在 t******l 的大作中提到】
: 只满足最低要求的,将来长大后不一定是阿斯博格。。。如果将来不是阿斯博格的话,
: 做对 AIME 超过 8 题的难度还是很大的。// run
: 当然我们这种普通娃就不要想了,幼儿园那么多老师眼皮底下混的。
:
: :我家也是。过于专注一个东西也是自闭的诊断之一。我家这条不符合,最后只达到了
: :ASPERGER最低的诊断要求。

t******l
发帖数: 10908
76
哥们去 pushy mama 微信群卧底打探一下,就知道“围城”永远是人之长情。。。

:别跑啊:-)我不打算推数学竞赛。如果他最后OUTGROW了ASPERGER,social
:communication上去了我更高兴。聪明的普通娃活得最开心。没有谁为了赢数学竞赛想
t******l
发帖数: 10908
77
不过这个可能不是 pushy mama 微信群的问题。。。我觉得存在一种可能,就是高端数
学竞赛里阿斯博格的比例可能也确实不低。。。这样数学竞赛的经验一传十十传百,但
很多人忘了在进不了 500 强的情况下,普通娃在 AMC 10 / 12 上不一定能抄阿斯博格
型的数学竞赛学习经验,因为大脑走线不太一样。倒不是说那些经验本身不对。
我觉得普通娃还是老老实实抄 Piaget 老先生的 paper 更靠谱些。

:哥们去 pushy mama 微信群卧底打探一下,就知道“围城”永远是人之长情。。。
f*******e
发帖数: 3433
78
普通娃也有很强的地方。我觉得关键要针对小娃的强项推对地方。国人太专注于数学。
我那些国内大学同学一谈都是小孩在推奥数。所以我觉得在美国除非娃数学真的特别突
出,推数学根本没用,都是打酱油的。所以我要推也推别的。

【在 t******l 的大作中提到】
: 不过这个可能不是 pushy mama 微信群的问题。。。我觉得存在一种可能,就是高端数
: 学竞赛里阿斯博格的比例可能也确实不低。。。这样数学竞赛的经验一传十十传百,但
: 很多人忘了在进不了 500 强的情况下,普通娃在 AMC 10 / 12 上不一定能抄阿斯博格
: 型的数学竞赛学习经验,因为大脑走线不太一样。倒不是说那些经验本身不对。
: 我觉得普通娃还是老老实实抄 Piaget 老先生的 paper 更靠谱些。
:
: :哥们去 pushy mama 微信群卧底打探一下,就知道“围城”永远是人之长情。。。
: :

t******l
发帖数: 10908
79
STEM 家庭其他也推不动。。。推奥外数学至少可以给计算科学啥的打个基础。。。文
科娃政治娃体育娃另说。。。

:普通娃也有很强的地方。我觉得关键要针对小娃的强项推对地方。国人太专注于数学
。我那些国内大学同学一谈都是小孩在推奥数。所以我觉得在美国除非娃数学真的特别
突出,推数学根本没用,都是打酱油的。所以我要推也推别的。
f*******e
发帖数: 3433
80
我推小娃专项,小娃以后专业可能是什么就是什么。专业肯定不是数学
。数学向你说的,主要针对打基础推,能达到他想学的专业的要求,SAT什么的能考好
就行了。

【在 t******l 的大作中提到】
: STEM 家庭其他也推不动。。。推奥外数学至少可以给计算科学啥的打个基础。。。文
: 科娃政治娃体育娃另说。。。
:
: :普通娃也有很强的地方。我觉得关键要针对小娃的强项推对地方。国人太专注于数学
: 。我那些国内大学同学一谈都是小孩在推奥数。所以我觉得在美国除非娃数学真的特别
: 突出,推数学根本没用,都是打酱油的。所以我要推也推别的。
: :

相关主题
觉得美帝数学进度慢的进来看一下学校早晚要教的东西,早早学会了又如何?
我招,我是猪amc 10 的A, B卷有啥区别
大学录取率来点美好的时光
进入Parenting版参与讨论
t******l
发帖数: 10908
81
至少 AMC 12 吧。。。美帝 SAT 数学就是个 “人肉 TI-82 计算器接口“,文科生的
数学,根本不够理工,当然读生物可能也够了。

:我推小娃专项,小娃以后专业可能是什么就是什么。专业肯定不是数学
:。数学向你说的,主要针对打基础推,能达到他想学的专业的要求,SAT什么的能考好
f*******e
发帖数: 3433
82
AMC12答对多少题算合格了?我看了一下2015年AMC12的题。开始很简单,最后几题难度
就上去了。我觉得ACM12后面的题对很多理工专业没多大用,除非是想学数学的或者做
理工科研究的。不过可以靠ACM的题锻炼一下逻辑思维。ACM12和SAT数学比难度怎样?

考好

【在 t******l 的大作中提到】
: 至少 AMC 12 吧。。。美帝 SAT 数学就是个 “人肉 TI-82 计算器接口“,文科生的
: 数学,根本不够理工,当然读生物可能也够了。
:
: :我推小娃专项,小娃以后专业可能是什么就是什么。专业肯定不是数学
: :。数学向你说的,主要针对打基础推,能达到他想学的专业的要求,SAT什么的能考好

t******l
发帖数: 10908
83
AMC 12 也就是祖国高考数学难度。

:AMC12答对多少题算合格了?我看了一下2015年AMC12的题。开始很简单,最后几题难
度就上去了。我觉得ACM12后面的题对很多理工专业没多大用,除非是想学数学的或者做
:理工科研究的。不过可以靠ACM的题锻炼一下逻辑思维。ACM12和SAT数学比难度怎样?
f*******e
发帖数: 3433
84
那SAT呢?我只要求娃达到SAT水平,不要求达到祖国高考水平:-)国内高中学的那些
数学我现在工作大部分都用不上,还给老师了。事实上高中学的大部分东西我现在都用
不上。

者做
样?

【在 t******l 的大作中提到】
: AMC 12 也就是祖国高考数学难度。
:
: :AMC12答对多少题算合格了?我看了一下2015年AMC12的题。开始很简单,最后几题难
: 度就上去了。我觉得ACM12后面的题对很多理工专业没多大用,除非是想学数学的或者做
: :理工科研究的。不过可以靠ACM的题锻炼一下逻辑思维。ACM12和SAT数学比难度怎样?

t******l
发帖数: 10908
85
AMC 10 / 12 和 AIME 是针对一般理工非数学专业的。。。数学专业之类的得上 USAMO
证明题。
AMC 10 / 12 和 AIME 对大多数理工专业有用,这个有用的意思,是因为不是阿斯博格
也不是衡水黄岗的普通娃,看到 AMC / AIME 必然放弃死背题型,因为实在是会被累死
。。。这样促进理解。。。而且 AMC / AIME 的 puzzle problem driven,增加兴趣。
。。老实说读数学书学数学,只能是搞美帝课内文科数学。。。理工科数学对大部分不
是阿斯博格的娃娃,不可能通过读厚厚的 AoPS 数学书的方式来学习,不说别的,光
attention span 就不行了。

:AMC12答对多少题算合格了?我看了一下2015年AMC12的题。开始很简单,最后几题难
度就上去了。我觉得ACM12后面的题对很多理工专业没多大用,除非是想学数学的或者做
:理工科研究的。不过可以靠ACM的题锻炼一下逻辑思维。ACM12和SAT数学比难度怎样?
s***n
发帖数: 1280
86
一般情况下,做对15-17题可以进AIME,19-21题可以进top 1%。
AMC12题目难度梯度比较明显。前面的题和SAT难度相当,后面题难度会比SAT高很多,
不是一个量级。
去年的AMC 12B相对容易,高分比较多。

【在 f*******e 的大作中提到】
: AMC12答对多少题算合格了?我看了一下2015年AMC12的题。开始很简单,最后几题难度
: 就上去了。我觉得ACM12后面的题对很多理工专业没多大用,除非是想学数学的或者做
: 理工科研究的。不过可以靠ACM的题锻炼一下逻辑思维。ACM12和SAT数学比难度怎样?
:
: 考好

t******l
发帖数: 10908
87
AMC 10 / 12 和 AIME 对数学知识本身的要求不超过 SAT。。。AIME 用不上的话,SAT
也用不上其实。
况且 SAT 还背二次方程求根公式,AMC 10 至少是要求 complete a square。。。求根
公式谁还背得出来现在?Complete a square 至少一辈子也不会忘记。

:那SAT呢?我只要求娃达到SAT水平,不要求达到祖国高考水平:-)国内高中学的那
些数学我现在工作大部分都用不上,还给老师了。
f*******e
发帖数: 3433
88
这些题目对锻炼逻辑思维,理解能力还是不错的。

USAMO
者做

【在 t******l 的大作中提到】
: AMC 10 / 12 和 AIME 对数学知识本身的要求不超过 SAT。。。AIME 用不上的话,SAT
: 也用不上其实。
: 况且 SAT 还背二次方程求根公式,AMC 10 至少是要求 complete a square。。。求根
: 公式谁还背得出来现在?Complete a square 至少一辈子也不会忘记。
:
: :那SAT呢?我只要求娃达到SAT水平,不要求达到祖国高考水平:-)国内高中学的那
: 些数学我现在工作大部分都用不上,还给老师了。

f*******e
发帖数: 3433
89
记下了。这么一说我对娃以后考SAT数学有了信心:-)

【在 s***n 的大作中提到】
: 一般情况下,做对15-17题可以进AIME,19-21题可以进top 1%。
: AMC12题目难度梯度比较明显。前面的题和SAT难度相当,后面题难度会比SAT高很多,
: 不是一个量级。
: 去年的AMC 12B相对容易,高分比较多。

s***n
发帖数: 1280
90
你娃的数学抽象思维不错,在同龄人中应该是很优秀的。不过,现在谈论他有没有数学
天分,还有点早。真正有天分的数学娃要抽象思维和逻辑思维都强,而逻辑思维要到小
学高年级,初中甚至高中才开始趋于成熟。
如果是以STEM为目标,现在这个阶段,主要应该提高孩子对数学和科学的兴趣,培养爱
动脑思考的习惯。象你教孩子进制,就是很好的例子。

【在 c********5 的大作中提到】
: 娃6岁了,一直都是普通娃,有些方面和同龄人比,还delay一年甚至更多。最近回国,
: 大家都说他数学有天分,可以多培养培养。当妈的我第一次有了飘飘然滴赶脚。上来一
: 是想问问,是不是大家忽悠我呢。二是想问问,下一步数学该如何推(无论是不是有天
: 赋)。真心讨论娃的问题,不喜欢,请右上角点X退出。万分谢谢。
: 1.我原本想给娃讲讲高斯定理,娃一看到我出的题,马上的反应是:
: 1+2+3+4+5+6+7+8+9+10
: =10+1+9+2+8+3+7+4+6+5
: =55
: (这个学校里倒是常常要求凑10,也可能有人问过他这道题)
: 2.

相关主题
有些小学四年级的数学题不太会做WISC-V 智商测试
转载: 从首个IMO季军谈起 by 付云皓今天和幼儿园老师开会review娃的progress,非常upset
新加坡数学对小学生帮助大吗?好学生去普通公立高中真的会被耽误吗?
进入Parenting版参与讨论
s***n
发帖数: 1280
91
同意普通娃也有很强的地方。我觉得不少孩子在某些方面有天分。家长看不到的,就是
普通娃;看到了并着意培养,就可能成了牛蛙。所以小娃教育,应该着重广度,让孩子
尝试很多新东西,然后观察孩子的兴趣和长项。
推数学比较多,有一个原因是数学天分最容易被发现。不象其他天分,往往要靠机缘,
孩子才能表现出来。另一个原因是推数学比推很多其他学术科目有用,特别对小娃。对
小娃来说,很多科目就是背,最多加点理解。推数学则更偏重锻炼抽象思维和逻辑思维
。自然,推法很重要。有些家长不看孩子资质的蛮推也是我不赞成的。

【在 f*******e 的大作中提到】
: 普通娃也有很强的地方。我觉得关键要针对小娃的强项推对地方。国人太专注于数学。
: 我那些国内大学同学一谈都是小孩在推奥数。所以我觉得在美国除非娃数学真的特别突
: 出,推数学根本没用,都是打酱油的。所以我要推也推别的。

f*******e
发帖数: 3433
92
同意。小娃教育广度,自然,培养兴趣很重要。小时候蛮木的推,推的太猛长大后容易
成强弩之末。看了AMC12题目后,觉得AMC12一些题目确实对锻炼逻辑思维有帮助。逻辑
思维对很多理工科还是挺有用的。等娃大些让他做点锻炼逻辑思维的数学题。

【在 s***n 的大作中提到】
: 同意普通娃也有很强的地方。我觉得不少孩子在某些方面有天分。家长看不到的,就是
: 普通娃;看到了并着意培养,就可能成了牛蛙。所以小娃教育,应该着重广度,让孩子
: 尝试很多新东西,然后观察孩子的兴趣和长项。
: 推数学比较多,有一个原因是数学天分最容易被发现。不象其他天分,往往要靠机缘,
: 孩子才能表现出来。另一个原因是推数学比推很多其他学术科目有用,特别对小娃。对
: 小娃来说,很多科目就是背,最多加点理解。推数学则更偏重锻炼抽象思维和逻辑思维
: 。自然,推法很重要。有些家长不看孩子资质的蛮推也是我不赞成的。

P******e
发帖数: 1325
93
跑个题问个Asperger的事。你家这种minor的,是不是就不需要定期治疗了?静等
outgrow就行?
当时有什么症状让你们觉得要查Asperger的?
我家大娃现在也在等待诊断Asperger,很忐忑。

【在 f*******e 的大作中提到】
: 我家也是。过于专注一个东西也是自闭的诊断之一。我家这条不符合,最后只达到了
: ASPERGER最低的诊断要求。

f*******e
发帖数: 3433
94
刚做了一下SAT sample, 确实很简单,和AMC12前面的题难度差不多,和后面的题不是
一个数量级。看来做SAT如果要得高分关键是不要粗心,对逻辑能力有些要求。

【在 s***n 的大作中提到】
: 一般情况下,做对15-17题可以进AIME,19-21题可以进top 1%。
: AMC12题目难度梯度比较明显。前面的题和SAT难度相当,后面题难度会比SAT高很多,
: 不是一个量级。
: 去年的AMC 12B相对容易,高分比较多。

f*******e
发帖数: 3433
95
有EARLY INTERVETION快一点。我家的开始没给SPEECH therapy, 说语言只delay了一年
,不够资格。后来我们在PRESCHOOL发现他完全玩自己的,只和大人说话,不和任何小
孩说话,就要求在preschool re-evaluate. 结果social skills的evaluation很低
。然后开始speech therapy. 一年后语言已经超过平均,他现在也主动和别的小孩子聊
天。今年是1周5天在学校therapy, 明年感觉他的speech therapy时间会减少。一年多下
来他的OT也从1-2年DELAY变得超过平均,成了进步最快的OT学生,明年估计会
失去OT。
Asperger症状不如低功能自闭明显,因为很多人只有轻微自闭症状,同时还有很强的地
方,比如高智商,很好的visual memory, 能注意很多细小的地方。明显的症状是理解
能力差,语言DELAY,社交DELAY or AKWARD, 不懂别人说话的CONTEXT,
有些rigidity症状, 对某些东西过于专注。做诊断的developmental pediatrician说
刚看时测试没看出一点自闭症状,直到最后测理解能力时发现了端倪,然后也
发现了轻微rigidity. 所以她说很可能outgrow。
不用担心了,如果真有ASPERGER,掌握方法,小孩社交能力会变强的, 不大可能
会像那些天生社交好的那样社交方面游刃有余,但至少能交几个朋友,不会在学校
孤孤单单。这点是最重要的。

【在 P******e 的大作中提到】
: 跑个题问个Asperger的事。你家这种minor的,是不是就不需要定期治疗了?静等
: outgrow就行?
: 当时有什么症状让你们觉得要查Asperger的?
: 我家大娃现在也在等待诊断Asperger,很忐忑。

P******e
发帖数: 1325
96
1周5天在学校therapy也是相当多了,看来还蛮有用的。
我娃好像没有说话的问题,她的中文能达到国内同龄小孩的程度。但是她的顽固,
Sensory issue,gross motor skill落后,social问题,情绪容易失控,这些问题让她
看起来是个典型的Asperger。
我都快愁死了。

【在 f*******e 的大作中提到】
: 有EARLY INTERVETION快一点。我家的开始没给SPEECH therapy, 说语言只delay了一年
: ,不够资格。后来我们在PRESCHOOL发现他完全玩自己的,只和大人说话,不和任何小
: 孩说话,就要求在preschool re-evaluate. 结果social skills的evaluation很低
: 。然后开始speech therapy. 一年后语言已经超过平均,他现在也主动和别的小孩子聊
: 天。今年是1周5天在学校therapy, 明年感觉他的speech therapy时间会减少。一年多下
: 来他的OT也从1-2年DELAY变得超过平均,成了进步最快的OT学生,明年估计会
: 失去OT。
: Asperger症状不如低功能自闭明显,因为很多人只有轻微自闭症状,同时还有很强的地
: 方,比如高智商,很好的visual memory, 能注意很多细小的地方。明显的症状是理解
: 能力差,语言DELAY,社交DELAY or AKWARD, 不懂别人说话的CONTEXT,

f*******e
发帖数: 3433
97
上K前是一周三天,上K是换成学校therapy, 推荐一周5天。他也有sensory issue, 在
preschool 因为sensory issue melt down两次, 哭了很久。上K时好多了,但还有一些
。K结束后sensory issue对他的日常生活已经没有影响了。sensory issue最好的解决
办法是多带他到一些地方去,鼓励但不强迫他面对害怕的东西,并且解释为什么有些
东西不可怕。向我娃这样轻微的1年就好了很多,重点的可能要几年甚至更长,因为
你可能解决了几个sensory issue, 新的又出现了。rigidity和情绪失控不大好解决,
要靠小孩观察别的孩子的反应,慢慢成熟。做家长的要不停的教他怎么控制情绪。

【在 P******e 的大作中提到】
: 1周5天在学校therapy也是相当多了,看来还蛮有用的。
: 我娃好像没有说话的问题,她的中文能达到国内同龄小孩的程度。但是她的顽固,
: Sensory issue,gross motor skill落后,social问题,情绪容易失控,这些问题让她
: 看起来是个典型的Asperger。
: 我都快愁死了。

t******l
发帖数: 10908
98
从数理教育因材施教的角度探讨一下。。。我觉得你文章里说的 “Asperger 同时还有
很强的地方,比如高智商,很好的visual memory, 能注意很多细小的地方。”,加上
“Asperger 的 long attention span 专注力”。。。我觉得这些导致在 AMC / AIME
数学教育上,普通娃无法采用 Asperger 的解题策略(因为看起来就算再合理,但记不
住或飞不出,也不行)。。。我觉得因为普通娃/父母不存在真正的数学逻辑,也痛
恨繁无图景的琐计算。。。所以普通娃/父母,本质上是 analytic intuition,而不是
真的在大脑里跑形式逻辑。这样导致“理解”和“时空图景”对普通娃/父母非常重要。

:有EARLY INTERVETION快一点。我家的开始没给SPEECH therapy, 说语言只delay了一
年,不够资格。后来我们在PRESCHOOL发现他完全玩自己的,只和大人说话,不和任何小
:孩说话,就要求在preschool re-evaluate. 结果social skills的evaluation很低
t******l
发帖数: 10908
99
我觉得一个例子就是 1983 AIME Problem #4(我那天就是教点几何和根号数运算)。
。。AoPS wiki 上的两个答案我觉得都不是一般正常人的思路。。。第一个答案解二元
二次方程组烦死,attention span 不够(这个二元二次方程有技巧可以直接先 reduce
成线性方程,play with squares,但还是很烦)。。。第二个答案的辅助线是天外飞
仙添出来的。
我觉得正常人的思路是连接 AC 取其中点记为 D,连接 DO 并延长,于 CB 延长线相交
,记为 E 点。然后 chase the triangles,所有数字都标记在图上,目测求解。不用
解繁琐二元二次方程。也不用天外飞仙辅助线,所有辅助线都是用 analytic
intuition 的 clue,跑 wishful thinking 猜测的。
http://www.artofproblemsolving.com/wiki/index.php?title=1983_AIME_Problems/Problem_4

【在 t******l 的大作中提到】
: 从数理教育因材施教的角度探讨一下。。。我觉得你文章里说的 “Asperger 同时还有
: 很强的地方,比如高智商,很好的visual memory, 能注意很多细小的地方。”,加上
: “Asperger 的 long attention span 专注力”。。。我觉得这些导致在 AMC / AIME
: 数学教育上,普通娃无法采用 Asperger 的解题策略(因为看起来就算再合理,但记不
: 住或飞不出,也不行)。。。我觉得因为普通娃/父母不存在真正的数学逻辑,也痛
: 恨繁无图景的琐计算。。。所以普通娃/父母,本质上是 analytic intuition,而不是
: 真的在大脑里跑形式逻辑。这样导致“理解”和“时空图景”对普通娃/父母非常重要。
:
: :有EARLY INTERVETION快一点。我家的开始没给SPEECH therapy, 说语言只delay了一
: 年,不够资格。后来我们在PRESCHOOL发现他完全玩自己的,只和大人说话,不和任何小

f*******e
发帖数: 3433
100
ASPERGER娃好像也分很多种,有专注于数学的,也有专注于画画的,也有专注于计算机
的。我家的目前还不像数学ASPERGER娃。我每天最多给他讲7,8分钟数学,他倒是很快
理解了,但7,8分钟一过就要玩了,看不出一点对数学的兴趣。他老师也说他数学
STRONG,但在学校里给他数学题,经常要催他做他才动手。所以我不清楚数学ASPERGER
娃和普通娃区别。他现在5.5岁,数学上的天分还没看出来。
我家的主要是很强的VISUAL记忆力,4岁半我们就发现他单词看一遍就能记住而且能写
出,当然与此照成强烈反差的是理解能力的落后。这个hyperlexia还是我从他的
therapist那听来的--他的therapist发现他认字后把listening comprehension改成
reading comprehension发现这个问题,然后告诉我们。我开始还不信,然后在reading
comprehension上做测试,果然很差。再改成listening comprehension, 还是不行,
查了很多资料,确实吻合,终于接受这个词了
要说别的天分,我只发现了他对计算机和拆装玩具的兴趣。计算机什么软件自己上手就
会,在学校里经常改IPAD和计算机SETTING,弄得鸡飞狗跳的。还把家里的玩具拆了装
上。我给他下载了6-8岁小孩玩的那种PROGRAMMING,没教他,因为我也不懂。他很快就
自己学会了,每天他30分钟的IPAD时间都在完这个。但数学天分迄今确实还没看到,感
觉数学上目前只属于理解比较快的普通娃,不见得比你娃强。


AIME
要。
何小

【在 t******l 的大作中提到】
: 从数理教育因材施教的角度探讨一下。。。我觉得你文章里说的 “Asperger 同时还有
: 很强的地方,比如高智商,很好的visual memory, 能注意很多细小的地方。”,加上
: “Asperger 的 long attention span 专注力”。。。我觉得这些导致在 AMC / AIME
: 数学教育上,普通娃无法采用 Asperger 的解题策略(因为看起来就算再合理,但记不
: 住或飞不出,也不行)。。。我觉得因为普通娃/父母不存在真正的数学逻辑,也痛
: 恨繁无图景的琐计算。。。所以普通娃/父母,本质上是 analytic intuition,而不是
: 真的在大脑里跑形式逻辑。这样导致“理解”和“时空图景”对普通娃/父母非常重要。
:
: :有EARLY INTERVETION快一点。我家的开始没给SPEECH therapy, 说语言只delay了一
: 年,不够资格。后来我们在PRESCHOOL发现他完全玩自己的,只和大人说话,不和任何小

相关主题
陶天才论天才 (转载)9岁, 如何报名考amc10
Explaining Your Math: Unnecessary at Best, Encumbering at W有明天考AMC 8的吗?
是不是任我儿子继续迷象棋[转载] 为什么说usamo 简单
进入Parenting版参与讨论
t******l
发帖数: 10908
101
其实我的疑惑并不是 Aspergers 里的细分类型,其实我一辈子也没面对面见过
Aspergers。
我的疑惑是大伙儿都说 AMC / AIME 层次的数学最最重要的是严谨的推理逻辑(
deduction),但我个人感觉对普通人来说,数学最最重要的是时空图景,因为普通人
记不住细节也经不起枯燥。。。我觉得这个问题可能是在数学教育方面,我觉得可能只
有欧几里德还算是个正常人,希尔伯特我觉得是半个 Aspergers,高斯估计是至少是一
个顶仨的 Super-Aspergers。// run
或者其实可以用 IBM 下象棋的 DeepBlue 跟 GooG 下围棋的 AlphaGo 来类比一下。。
。AlphaGo 跟 DeepBlue 不一样的是,AlphaGo 下棋基本靠猜。。。以至于 AlphaGo
跟李九段的第四盘,AlphaGo 前面都猜得八九不离十了,但后来出了一手臭子(猜错简
单情况),结果在形势一片大好的情况下输了。。。有争议的说,我觉得 AlphaGo 比
DeepBlue 的棋风更接近于普通人类的思维。
所以我觉得高中数学老师说的辅助线各种记忆模式,以及添对了辅助线一定能做对题,
我觉得其实都不是普通人的思维。。。所以我觉得其实是高中数学老师搞错了。。。一
个简单的例子就是一旦到 AMC / AIME,普通娃添辅助线都是根据欧几里德的精神到处
狂猜(analytic intuition),而且一般猜上五种必然有一款样子上比较像的,但很可
能后面 chase shapes 没来得及 chase 出来(deduction),于是 200 米差了 0.1 秒
被预赛出局。

【在 f*******e 的大作中提到】
: ASPERGER娃好像也分很多种,有专注于数学的,也有专注于画画的,也有专注于计算机
: 的。我家的目前还不像数学ASPERGER娃。我每天最多给他讲7,8分钟数学,他倒是很快
: 理解了,但7,8分钟一过就要玩了,看不出一点对数学的兴趣。他老师也说他数学
: STRONG,但在学校里给他数学题,经常要催他做他才动手。所以我不清楚数学ASPERGER
: 娃和普通娃区别。他现在5.5岁,数学上的天分还没看出来。
: 我家的主要是很强的VISUAL记忆力,4岁半我们就发现他单词看一遍就能记住而且能写
: 出,当然与此照成强烈反差的是理解能力的落后。这个hyperlexia还是我从他的
: therapist那听来的--他的therapist发现他认字后把listening comprehension改成
: reading comprehension发现这个问题,然后告诉我们。我开始还不信,然后在reading
: comprehension上做测试,果然很差。再改成listening comprehension, 还是不行,

t******l
发帖数: 10908
102
我觉得对数学以及理工教育的实践而言,我觉得中小学数学教育受纯数学家影响太深,
其实作为中小学实用数学,应该多参考物理学和计算科学的意见,因为毕竟中小学数学
是实用基础课。
在这点上,我觉得中小学数学太强调 pattern 本身,而忽略了 space-time pattern(
我生造的词),也就是把 pattern 投射到 space-time 上。。。纯数学家会觉得这个
投射破坏了数学美,带来了 intuition。。。但我觉得有争议的是,中小学数学首先得
考虑实用。。。而从实用的角度看,因为大部分人从解数学题到实践中的 problem-
solving,其实更像 AlphaGo 而不是 DeepBlue,也就是普通人类思维最最基本的是
analytic intuition (深神经网络不是?),而不是 deductive-logic(这个本质上
是图灵机)。。。而对 space-time 的 intuition(而不是形式逻辑),有争议的说,
我觉得那才是普通人的数理思维的基石。

【在 t******l 的大作中提到】
: 其实我的疑惑并不是 Aspergers 里的细分类型,其实我一辈子也没面对面见过
: Aspergers。
: 我的疑惑是大伙儿都说 AMC / AIME 层次的数学最最重要的是严谨的推理逻辑(
: deduction),但我个人感觉对普通人来说,数学最最重要的是时空图景,因为普通人
: 记不住细节也经不起枯燥。。。我觉得这个问题可能是在数学教育方面,我觉得可能只
: 有欧几里德还算是个正常人,希尔伯特我觉得是半个 Aspergers,高斯估计是至少是一
: 个顶仨的 Super-Aspergers。// run
: 或者其实可以用 IBM 下象棋的 DeepBlue 跟 GooG 下围棋的 AlphaGo 来类比一下。。
: 。AlphaGo 跟 DeepBlue 不一样的是,AlphaGo 下棋基本靠猜。。。以至于 AlphaGo
: 跟李九段的第四盘,AlphaGo 前面都猜得八九不离十了,但后来出了一手臭子(猜错简

s**********y
发帖数: 509
103
逻辑是一部分。
想象力也重要。 数学用逻辑撑起想象力, 倒也是理性思维的一种高级阶段。
顺便说一句, 算数不是数学。 我看一伙人总是算来算去, 老觉得好好的娃, 被带坏
了。 疾走哈!

【在 t******l 的大作中提到】
: 其实我的疑惑并不是 Aspergers 里的细分类型,其实我一辈子也没面对面见过
: Aspergers。
: 我的疑惑是大伙儿都说 AMC / AIME 层次的数学最最重要的是严谨的推理逻辑(
: deduction),但我个人感觉对普通人来说,数学最最重要的是时空图景,因为普通人
: 记不住细节也经不起枯燥。。。我觉得这个问题可能是在数学教育方面,我觉得可能只
: 有欧几里德还算是个正常人,希尔伯特我觉得是半个 Aspergers,高斯估计是至少是一
: 个顶仨的 Super-Aspergers。// run
: 或者其实可以用 IBM 下象棋的 DeepBlue 跟 GooG 下围棋的 AlphaGo 来类比一下。。
: 。AlphaGo 跟 DeepBlue 不一样的是,AlphaGo 下棋基本靠猜。。。以至于 AlphaGo
: 跟李九段的第四盘,AlphaGo 前面都猜得八九不离十了,但后来出了一手臭子(猜错简

f*******e
发帖数: 3433
104
你这些问题估计要数学ASPERGER娃的爸妈或者数学ASPERGER的本人才能回答了:-)我
肯定不是ASPERGER,因为我记忆力不是很好,语言没有滞后,没问题交朋友,我高中的
第一也是靠努力和题海战术得到了(说白了就是很努力自推的普通娃),所以也回答不
出:-)过几年小孩长大了可能我能回答一些这方面问题。

【在 t******l 的大作中提到】
: 其实我的疑惑并不是 Aspergers 里的细分类型,其实我一辈子也没面对面见过
: Aspergers。
: 我的疑惑是大伙儿都说 AMC / AIME 层次的数学最最重要的是严谨的推理逻辑(
: deduction),但我个人感觉对普通人来说,数学最最重要的是时空图景,因为普通人
: 记不住细节也经不起枯燥。。。我觉得这个问题可能是在数学教育方面,我觉得可能只
: 有欧几里德还算是个正常人,希尔伯特我觉得是半个 Aspergers,高斯估计是至少是一
: 个顶仨的 Super-Aspergers。// run
: 或者其实可以用 IBM 下象棋的 DeepBlue 跟 GooG 下围棋的 AlphaGo 来类比一下。。
: 。AlphaGo 跟 DeepBlue 不一样的是,AlphaGo 下棋基本靠猜。。。以至于 AlphaGo
: 跟李九段的第四盘,AlphaGo 前面都猜得八九不离十了,但后来出了一手臭子(猜错简

t******l
发帖数: 10908
105
进一步有争议的说,纯数学自身都没有完美的解决 intuition 的问题,比如 Costco
大西瓜问题(Banach-Tarski paradox / Axiom of Choice)至今还依赖马工的各种补
丁凑合过日子。。。(当然如果连守恒定律都不要了的话,那也不需要补丁。。。但
Piaget 老先生的水杯测试铁定不过了,估计要被 Piaget 老先生敲个弱智的图章关进
CalTech 疯人院确保不闹事!// run )。
所以 Andrew Lloyd Webber 先知很多年前在音乐剧 Evita 里就曾经写道:
Deductions are illusions
They are not the solutions they promised to be
The answer was here all in the space-time
I love intuitions and hope you love intuitions too
http://www.youtube.com/watch?v=giB7Ye-ar5M

【在 t******l 的大作中提到】
: 我觉得对数学以及理工教育的实践而言,我觉得中小学数学教育受纯数学家影响太深,
: 其实作为中小学实用数学,应该多参考物理学和计算科学的意见,因为毕竟中小学数学
: 是实用基础课。
: 在这点上,我觉得中小学数学太强调 pattern 本身,而忽略了 space-time pattern(
: 我生造的词),也就是把 pattern 投射到 space-time 上。。。纯数学家会觉得这个
: 投射破坏了数学美,带来了 intuition。。。但我觉得有争议的是,中小学数学首先得
: 考虑实用。。。而从实用的角度看,因为大部分人从解数学题到实践中的 problem-
: solving,其实更像 AlphaGo 而不是 DeepBlue,也就是普通人类思维最最基本的是
: analytic intuition (深神经网络不是?),而不是 deductive-logic(这个本质上
: 是图灵机)。。。而对 space-time 的 intuition(而不是形式逻辑),有争议的说,

P******e
发帖数: 1325
106
我们那个年代,国内对Asperger根本没有认识和诊断。你就是认识Aspergers,
你也不知道他们是。
我最近因为大娃,收集了一些Asperger的资料,发现原来娃爸就是典型Asperger,
当然他自己坚决不肯承认。
Asperger也不都是高智商,只是大于等于平均而已。

【在 t******l 的大作中提到】
: 其实我的疑惑并不是 Aspergers 里的细分类型,其实我一辈子也没面对面见过
: Aspergers。
: 我的疑惑是大伙儿都说 AMC / AIME 层次的数学最最重要的是严谨的推理逻辑(
: deduction),但我个人感觉对普通人来说,数学最最重要的是时空图景,因为普通人
: 记不住细节也经不起枯燥。。。我觉得这个问题可能是在数学教育方面,我觉得可能只
: 有欧几里德还算是个正常人,希尔伯特我觉得是半个 Aspergers,高斯估计是至少是一
: 个顶仨的 Super-Aspergers。// run
: 或者其实可以用 IBM 下象棋的 DeepBlue 跟 GooG 下围棋的 AlphaGo 来类比一下。。
: 。AlphaGo 跟 DeepBlue 不一样的是,AlphaGo 下棋基本靠猜。。。以至于 AlphaGo
: 跟李九段的第四盘,AlphaGo 前面都猜得八九不离十了,但后来出了一手臭子(猜错简

P******e
发帖数: 1325
107
养这样的孩子真心累啊。

【在 f*******e 的大作中提到】
: 上K前是一周三天,上K是换成学校therapy, 推荐一周5天。他也有sensory issue, 在
: preschool 因为sensory issue melt down两次, 哭了很久。上K时好多了,但还有一些
: 。K结束后sensory issue对他的日常生活已经没有影响了。sensory issue最好的解决
: 办法是多带他到一些地方去,鼓励但不强迫他面对害怕的东西,并且解释为什么有些
: 东西不可怕。向我娃这样轻微的1年就好了很多,重点的可能要几年甚至更长,因为
: 你可能解决了几个sensory issue, 新的又出现了。rigidity和情绪失控不大好解决,
: 要靠小孩观察别的孩子的反应,慢慢成熟。做家长的要不停的教他怎么控制情绪。

P******e
发帖数: 1325
108
小娃对算数有兴趣愿意自推,应该受到鼓励。
数学本身也是个循序渐进的过程,幼时的思维训练总是有帮助的。

【在 s**********y 的大作中提到】
: 逻辑是一部分。
: 想象力也重要。 数学用逻辑撑起想象力, 倒也是理性思维的一种高级阶段。
: 顺便说一句, 算数不是数学。 我看一伙人总是算来算去, 老觉得好好的娃, 被带坏
: 了。 疾走哈!

f*******e
发帖数: 3433
109
是累,但方法适当的话会越来越好带的。我一年前还焦头烂额,一年后终于看到曙光了。

【在 P******e 的大作中提到】
: 养这样的孩子真心累啊。
s**********y
发帖数: 509
110
我不知怎的, 一看到“训练”两字, 眼前中总浮现训狗的画面。 这肯定是我的问题
, 是吧?

【在 P******e 的大作中提到】
: 小娃对算数有兴趣愿意自推,应该受到鼓励。
: 数学本身也是个循序渐进的过程,幼时的思维训练总是有帮助的。

相关主题
[转载] 为什么说usamo 简单我招,我是猪
二年级女儿的report大学录取率
觉得美帝数学进度慢的进来看一下学校早晚要教的东西,早早学会了又如何?
进入Parenting版参与讨论
t******l
发帖数: 10908
111
我上传一个那道题目的草稿纸。。。当然我本意是用兴趣题给娃介绍一下几何里的
chase triangle 和代数里的 play with roots,并不是要搞难题。。。但是正好碰到
跟 wiki 上答案不一样的解法。(BTW: 有些计算过程是娃写的,因为教娃要求娃
participation,否则娃给我睡着了。。。participation 其实也是教普通娃用 joint
attention 来对付 attention span 的困难。。。。大人不会写这么傻的根号计算过程
。不要把我当沙比就好。)。
所以我觉得普通正常学生是 follow the intuition 而不是形式逻辑,而且都不会写啥
等式/证明,直接标记在图上,或者对图形直接剪枝/放大等等。。。证明只是走个概念
保证 intuition 不出错,并不走严格证明形式。
所以过多的题型总结/严谨逻辑,我觉得并不适于普通娃。。。因为那种看着虽然漂亮
,但对于普通娃最大的问题是(1)普通娃 attention span 不行(2)普通娃没有照相
式记忆记住细节。。。。但同时有没能发挥普通娃最大的长处就是 intuition 和合理
猜测,对真实空间实在事物的直感,以及经验猜测,来作为解题引导。

【在 s**********y 的大作中提到】
: 逻辑是一部分。
: 想象力也重要。 数学用逻辑撑起想象力, 倒也是理性思维的一种高级阶段。
: 顺便说一句, 算数不是数学。 我看一伙人总是算来算去, 老觉得好好的娃, 被带坏
: 了。 疾走哈!

P******e
发帖数: 1325
112
对,是你的问题。

【在 s**********y 的大作中提到】
: 我不知怎的, 一看到“训练”两字, 眼前中总浮现训狗的画面。 这肯定是我的问题
: , 是吧?

w***g
发帖数: 5958
113
这孩子以后能那费尔兹奖. 不过最好不要太早让他接触数学.

【在 c********5 的大作中提到】
: 娃6岁了,一直都是普通娃,有些方面和同龄人比,还delay一年甚至更多。最近回国,
: 大家都说他数学有天分,可以多培养培养。当妈的我第一次有了飘飘然滴赶脚。上来一
: 是想问问,是不是大家忽悠我呢。二是想问问,下一步数学该如何推(无论是不是有天
: 赋)。真心讨论娃的问题,不喜欢,请右上角点X退出。万分谢谢。
: 1.我原本想给娃讲讲高斯定理,娃一看到我出的题,马上的反应是:
: 1+2+3+4+5+6+7+8+9+10
: =10+1+9+2+8+3+7+4+6+5
: =55
: (这个学校里倒是常常要求凑10,也可能有人问过他这道题)
: 2.

t******l
发帖数: 10908
114
当然如果改成 O 点向 EC 做垂线,那两边都会出整数。。。但是这并不是正常人的第
一思维,因为正常人首先是图形直感驱动,而不会首先是数字直感驱动(因为不是
hypernumeracy)。。。垂线做成一个翻转的相似三角形不会是普通娃普通人的第一自
然想法(虽然这样做就能够 cancel sqrt(10) 导致三角形两直角边都是整数),除非
是数字计算太烦导致去找 2nd option。
这也就是我看到的其他不少答案的写法里,对我而言比较奇怪的地方。。。我以前是认
为可能是从答案的反凑回来的写法,现在我觉得可能的确有相当一部分是
hypernumeracy 的倾向写的,也就是对数字本身更直感敏感。。。而大多数普通正常人
首先是对空间和结构更直感。
这个好比一般人去看 open house,首先不会去注意和记住门牌号,首先注意记住的是
房子是啥样子,进去看看大致的 floorplan 是咋样,从厨房走到厕所要多久,Kitchen
CounterTop 上是不是适合 pushy mama creative sex position 啥的。。。所以那些
才是普通人的思维,而不是一上来先记住个门牌号,看房子的时候是几点几分几秒啥的
。。。

【在 t******l 的大作中提到】
: 我上传一个那道题目的草稿纸。。。当然我本意是用兴趣题给娃介绍一下几何里的
: chase triangle 和代数里的 play with roots,并不是要搞难题。。。但是正好碰到
: 跟 wiki 上答案不一样的解法。(BTW: 有些计算过程是娃写的,因为教娃要求娃
: participation,否则娃给我睡着了。。。participation 其实也是教普通娃用 joint
: attention 来对付 attention span 的困难。。。。大人不会写这么傻的根号计算过程
: 。不要把我当沙比就好。)。
: 所以我觉得普通正常学生是 follow the intuition 而不是形式逻辑,而且都不会写啥
: 等式/证明,直接标记在图上,或者对图形直接剪枝/放大等等。。。证明只是走个概念
: 保证 intuition 不出错,并不走严格证明形式。
: 所以过多的题型总结/严谨逻辑,我觉得并不适于普通娃。。。因为那种看着虽然漂亮

f*******e
发帖数: 3433
115
看看这个吧:
http://www.huffingtonpost.com/entry/autism-study-photos_us_562a51dfe4b0443bb563ad29

Kitchen

【在 t******l 的大作中提到】
: 当然如果改成 O 点向 EC 做垂线,那两边都会出整数。。。但是这并不是正常人的第
: 一思维,因为正常人首先是图形直感驱动,而不会首先是数字直感驱动(因为不是
: hypernumeracy)。。。垂线做成一个翻转的相似三角形不会是普通娃普通人的第一自
: 然想法(虽然这样做就能够 cancel sqrt(10) 导致三角形两直角边都是整数),除非
: 是数字计算太烦导致去找 2nd option。
: 这也就是我看到的其他不少答案的写法里,对我而言比较奇怪的地方。。。我以前是认
: 为可能是从答案的反凑回来的写法,现在我觉得可能的确有相当一部分是
: hypernumeracy 的倾向写的,也就是对数字本身更直感敏感。。。而大多数普通正常人
: 首先是对空间和结构更直感。
: 这个好比一般人去看 open house,首先不会去注意和记住门牌号,首先注意记住的是

t******l
发帖数: 10908
116
我觉得你给的 link 就是我说的,区别是不同环境下对输入信息剪枝的问题。
你的文章里说的是,在人脸和树枝同时出现的时候,正常普通人是先注意人脸,其实才
注意树枝。。。当然如果把树枝换成棕熊,另说。
而我上面说的就是,在几何题里形状和数字同时出现的时候,正常普通人是先注意形状
,选择最简形状往下走。。。而 hypernumeracy 可能反过来,先注意数字的简洁,然
后才是形状。
当然我不是说这道题的 AoPS wiki 的答案。我是说某些其他题目的答案,也不一定是
几何题。。。总之就是写法跟本能习惯反过来。。。所以我猜奥数和奥外数学,可能
hypernumeracy 比通常环境下的比例更高。

【在 f*******e 的大作中提到】
: 看看这个吧:
: http://www.huffingtonpost.com/entry/autism-study-photos_us_562a51dfe4b0443bb563ad29
:
: Kitchen

f*******e
发帖数: 3433
117
我刚刚去做了这个题目。结果发现我的解法和下面链接的SOLUTION1一模一样:
https://www.artofproblemsolving.com/wiki/index.php?title=1983_AIME_Problems/
Problem_4
SOLUTION 1是正常解法吧。

joint

【在 t******l 的大作中提到】
: 我上传一个那道题目的草稿纸。。。当然我本意是用兴趣题给娃介绍一下几何里的
: chase triangle 和代数里的 play with roots,并不是要搞难题。。。但是正好碰到
: 跟 wiki 上答案不一样的解法。(BTW: 有些计算过程是娃写的,因为教娃要求娃
: participation,否则娃给我睡着了。。。participation 其实也是教普通娃用 joint
: attention 来对付 attention span 的困难。。。。大人不会写这么傻的根号计算过程
: 。不要把我当沙比就好。)。
: 所以我觉得普通正常学生是 follow the intuition 而不是形式逻辑,而且都不会写啥
: 等式/证明,直接标记在图上,或者对图形直接剪枝/放大等等。。。证明只是走个概念
: 保证 intuition 不出错,并不走严格证明形式。
: 所以过多的题型总结/严谨逻辑,我觉得并不适于普通娃。。。因为那种看着虽然漂亮

t******l
发帖数: 10908
118
另外刚才敲错了,因为是 EC 而不是 AE。。。这也其实是普通人解数学题,暂时记忆
里首先是形状本身,而不是两个字母。。。也就是说,图景本身比字母符号优先。。。
其实我给娃讲题的时候,经常是这个这个那个那个,首先用笔来比划,首先并不说/记
字母本身,字母常常是后续加上去的。。。

【在 f*******e 的大作中提到】
: 看看这个吧:
: http://www.huffingtonpost.com/entry/autism-study-photos_us_562a51dfe4b0443bb563ad29
:
: Kitchen

t******l
发帖数: 10908
119
Solution 1 算二元二次方程比较烦,如果是我娃就听都听不下去,更不要说算,还在
旁边笑话我(当然是首先看过了几何解法)。
我那个添辅助线的解法不一定在给定时间里能想到。。。但是如果能想到的话,我觉得
一般娃会 prefer 我那个添辅助线解法,因为图形表述对 attention span 的要求,比
那个解二元二次方程对 attention span 的要求低。。。而且那个辅助线是有迹可寻的
,而不是天外飞仙型的辅助线。
当然我们祖国把代数练的很熟,另说。。。但就是我在娃面前边讲边算解那个二元二次
方程,当中还算错不少地方(我还用了 play with squares 一步就搞出一个线性方程
,然后代回去,但这样还是很烦)。。。在知道两个解法的情况下,几何解法绝对比代
数解法快,显然不容易出错。。。我觉得大多数人会选择几何解法,如果知道的话。

Problems/

【在 f*******e 的大作中提到】
: 我刚刚去做了这个题目。结果发现我的解法和下面链接的SOLUTION1一模一样:
: https://www.artofproblemsolving.com/wiki/index.php?title=1983_AIME_Problems/
: Problem_4
: SOLUTION 1是正常解法吧。
:
: joint

t******l
发帖数: 10908
120
我觉得中肯的说,有一种可能是不同的人对于 (1) photographic memory, (2)
intuition, (3) attention span on "boring" tasks 的标准和接受程度不一样,有可
能导致这种看法的差别。
或者用另一个比方,如果要赢一盘象棋,不外乎是著名的以下三条路数的结合:
(1) Play the opening-game like a book. -- Photographic memory.
(2) Play the middle-game like a magician. -- Intuition.
(3) Play the end-game like a machine. -- Attention Span (on "boring" task).
但是各家根据自己的实际情况,混合比都不太一样,可能导致这类争议。

【在 P******e 的大作中提到】
: 对,是你的问题。
相关主题
amc 10 的A, B卷有啥区别转载: 从首个IMO季军谈起 by 付云皓
来点美好的时光新加坡数学对小学生帮助大吗?
有些小学四年级的数学题不太会做WISC-V 智商测试
进入Parenting版参与讨论
c***x
发帖数: 1826
121

这段思考有点意思,去读一下那本Thinking fast and slow
据说,有阿斯伯格的人大概通常不会觉得自己有。

【在 t******l 的大作中提到】
: 其实我的疑惑并不是 Aspergers 里的细分类型,其实我一辈子也没面对面见过
: Aspergers。
: 我的疑惑是大伙儿都说 AMC / AIME 层次的数学最最重要的是严谨的推理逻辑(
: deduction),但我个人感觉对普通人来说,数学最最重要的是时空图景,因为普通人
: 记不住细节也经不起枯燥。。。我觉得这个问题可能是在数学教育方面,我觉得可能只
: 有欧几里德还算是个正常人,希尔伯特我觉得是半个 Aspergers,高斯估计是至少是一
: 个顶仨的 Super-Aspergers。// run
: 或者其实可以用 IBM 下象棋的 DeepBlue 跟 GooG 下围棋的 AlphaGo 来类比一下。。
: 。AlphaGo 跟 DeepBlue 不一样的是,AlphaGo 下棋基本靠猜。。。以至于 AlphaGo
: 跟李九段的第四盘,AlphaGo 前面都猜得八九不离十了,但后来出了一手臭子(猜错简

c***x
发帖数: 1826
122

Pat, pat,这些问题都会outgrow的,父母要付出额外的心血是真的。
高超的智商和特别的能力,也许是命运许给孩子的collateral gift。
很多事情,并不是我们能选择的,另一些则是我们可以去努力的。

【在 P******e 的大作中提到】
: 1周5天在学校therapy也是相当多了,看来还蛮有用的。
: 我娃好像没有说话的问题,她的中文能达到国内同龄小孩的程度。但是她的顽固,
: Sensory issue,gross motor skill落后,social问题,情绪容易失控,这些问题让她
: 看起来是个典型的Asperger。
: 我都快愁死了。

t******l
发帖数: 10908
123
对你来说这是个理论问题,对我来说这可能是个实际问题。。。因为我的担心是,大部
分普通娃,在 AMC / AIME 上,随着时间推移,可能真心比不过 Aspergers。。。人不
能以短击长,过犹不及。所以啥时见好就收,是个很实在的问题。
另一个事实是,你看我在这里灌数学。。。其实我从来没有真正意义上参加数学竞赛,
我参加的是物理竞赛(当然那苦鼻实验赛部分真心比不过)。。。数学竞赛那时我真心
不行,我也不愿花时间,而且也不能喧宾夺主影响高考。。。我觉得争议的说,物理和
实用计算科学,比数学能得到更多 Intuition 的支持,这样相对而言可以在
hypernumeracy 前找回点场子。
还有一个问题,在类似的环境里,似乎我更多的在记不住公式/计算错误率较高的那群
。。。我以前一直以为我是不是记忆力差错误率高。。。而现在看来,可能我就是
normal。可能就是当 normal 的大脑把力气花在相对有趣的 intuition 上面以后,就
不会有体力和 attention span 去搞 open-game book 和 end-game machine 了。
总而言之我觉得在 Aspergers 擅长的方面,可能普通娃真心搞不过。。。普通娃适可
而止,别过犹不及,才是大方向的智慧。

:... ...
:据说,有阿斯伯格的人大概通常不会觉得自己有。
c********5
发帖数: 207
124
说说他的短处,请帮我看看。
从小各个指标基本就是踩着线达标的。儿医提议让做测试,但是回家训练一下,没几天
就会了,例如point finger,用指头抓豆子动作。所以就没做测试。
从爬,坐,站,走,delay应该是3个月-9个月不等,后来我都麻木了。
再后来语言有delay,但儿医说因为是英文测试,他只懂中文,所以也测不出来。
我应该是从他三个月开始,就一直关注自闭,叫名字有反应,和人有互动。有眼神接触
。没有小狗动作。出现过要求客人出去,从新进屋的无理要求,但过一阵子也好了。
他虽然好多东西都delay,但后来都会了,我想也许就是他运动方面发育比同类人晚吧。
搞过baseball,搞不清状况,动作滑稽可笑。而且没兴趣。更喜欢玩沙子或者和同学说
笑,
搞过足球,属于跟在后面跑,碰不到球,搞不清状况,而且没兴趣。
所以K我就放羊了。游泳兴趣也不大,但现在好歹能游几feet了。

【在 f*******e 的大作中提到】
: 我就是前面说我娃有hyperlexia的,就是那种photographical memory,认字写字能力
: 超强,但阅读理解能力弱。很多researcher认为有hyperlexia的都有自闭,包括高功能
: 自闭。当然,这本身也是长处,父母不应该忽略了。这种情况的教育方法是扬长推短,
: 充分发挥长处,让娃长处自由发展。短处努力提高,不求做得很好,但求达到平均水平
: 。我家的经过一年训练后,语言能力测评达到中等偏上,理解能力和社交能力也向中等
: 靠近。对这种娃,教育方法很重要,否则长大容易变美国人口中的NERD.
:
: 3

f*******e
发帖数: 3433
125
至少他不会是低功自闭,高功自闭(比如ASPERGER)的诊断需要详细EVALUATE,很难从
父母的简单描述看出除非父母对自闭有一定了解并且能精确的描述小孩行为。比如说,
他从一个活动转到另一个活动时难不难,他会不会和其他小朋友聊天互动(和大人聊天
互动好也不能排测高功自闭),是否RIGID(或者说顽固),会不会害怕一些同龄小孩
都不怕的东西,听力理解怎样(比如读一本6岁年龄段的英文书,他能不能回答问题)
,是不是过于专注于某项活动,是否不大理解别人的表情。
高功自闭眼神交流不一定有问题(我家眼神交流没问题)。
语言是否DELAY可以要求做中文测试,只要是在家说的语言没有DELAY,就没有DELAY。
你说的里面和高功自闭可能相关的是”玩计算器能玩2-3个小时(过于专注一样东西)
“,”出现过要求客人出去,重新进屋的无理要求(RIGIDITY)“。但光这些行为还不
能说他有高功自闭。如果他在学校社交上没什么问题,老师没有抱怨过他
RIGID或有时候难以TRANSIT活着别的一些问题,那就不用管,说明对日常生活没影响。
我小孩班上有两个小孩上K前没有EVALUATE,后来被老师建议EVALUATE以后,
发现有ADHD或者自闭(老师没说,我从他们行为看出的),都安排了AID,开始了
THERAPY。那两小孩比我家的还难管。还有一些高功的到3年级左右才能发现自闭
(一个迹象是发现没朋友了,学习成绩也下降因为理解力没跟上去--三年级前靠
记忆就够了,3年级后开始要求理解,所以有些迹象这时候才出现)。

吧。

【在 c********5 的大作中提到】
: 说说他的短处,请帮我看看。
: 从小各个指标基本就是踩着线达标的。儿医提议让做测试,但是回家训练一下,没几天
: 就会了,例如point finger,用指头抓豆子动作。所以就没做测试。
: 从爬,坐,站,走,delay应该是3个月-9个月不等,后来我都麻木了。
: 再后来语言有delay,但儿医说因为是英文测试,他只懂中文,所以也测不出来。
: 我应该是从他三个月开始,就一直关注自闭,叫名字有反应,和人有互动。有眼神接触
: 。没有小狗动作。出现过要求客人出去,从新进屋的无理要求,但过一阵子也好了。
: 他虽然好多东西都delay,但后来都会了,我想也许就是他运动方面发育比同类人晚吧。
: 搞过baseball,搞不清状况,动作滑稽可笑。而且没兴趣。更喜欢玩沙子或者和同学说
: 笑,

t******l
发帖数: 10908
126
说到这个大脑的 “理解 vs 记忆”,还是个挺头疼的玩意儿。。。俺在 BBS 上灌数学
教育的时候,曾被那谁谁谁下定论说这哥们除了集合论以外,啥都记不住的感觉。
俺家大娃五年级开始的 school report 和州考方面,理解应用方面永远比词汇和其他
记忆的分高,以至于有一种记忆力不行的感觉。。。计算错误一大堆。。。但实际上大
娃三年级时州考是满分,班上满分一堆但也满分了不是?。。。现在的法语课刚开始的
死背单词也基本不出错,那法语单词尼玛真心是死记的活。。。我基本都能总结出规律
来了:娃只要一开始理解,记忆力立即下降三十个点;娃只要一觉得烦没有兴趣,记忆
力立马下降六十个点有木有!!!
我对以上的理解是:狗屁心理学家说的啥人类大脑只用了 10% 的理论,就是扯几把蛋
。。。大脑利用率,我目测没有 100% 也有 90%。。。所以发生的其实就是跑完 4x100
接力后,紧接着跳远就不行了。。。当然心理学家的说法是你肚腩上不是还有一堆脂
肪么?我就问心理学家特么 pushy mama creative sex position 的时候,都知道速度
肌白肌是有不应期的。。。在不应期里西楚霸王硬上别姬的话,不管肚腩上有多少吨脂
肪,精尽人亡的时候脂肪有屁用啊?

:至少他不会是低功自闭,高功自闭(比如ASPERGER)的诊断需要详细EVALUATE,很难
从父母的简单描述看出除非父母对自闭有一定了解并且能精确的描述小孩行为。比如说
,他从一个活动转到另一个活动时难不难,他会不会和其他小朋友聊天互动(和大人聊
天互动好也不能排测高功自闭),是否RIGID(或者说顽固),会不会害怕一些同龄小孩
:都不怕的东西,听力理解怎样(比如读一本6岁年龄段的英文书,他能不能回答问题)
f*******e
发帖数: 3433
127
你说的这个记忆VS理解,我也感觉有这么回事。我娃理解能力和社交能力上来以后,我
娃虽然VISUAL记忆还是很强,认字写字能力不受影响,但我总觉得他记忆力下降了,具
体怎么下降了却又说不上来,可能还是降得不够厉害。

4x100

【在 t******l 的大作中提到】
: 说到这个大脑的 “理解 vs 记忆”,还是个挺头疼的玩意儿。。。俺在 BBS 上灌数学
: 教育的时候,曾被那谁谁谁下定论说这哥们除了集合论以外,啥都记不住的感觉。
: 俺家大娃五年级开始的 school report 和州考方面,理解应用方面永远比词汇和其他
: 记忆的分高,以至于有一种记忆力不行的感觉。。。计算错误一大堆。。。但实际上大
: 娃三年级时州考是满分,班上满分一堆但也满分了不是?。。。现在的法语课刚开始的
: 死背单词也基本不出错,那法语单词尼玛真心是死记的活。。。我基本都能总结出规律
: 来了:娃只要一开始理解,记忆力立即下降三十个点;娃只要一觉得烦没有兴趣,记忆
: 力立马下降六十个点有木有!!!
: 我对以上的理解是:狗屁心理学家说的啥人类大脑只用了 10% 的理论,就是扯几把蛋
: 。。。大脑利用率,我目测没有 100% 也有 90%。。。所以发生的其实就是跑完 4x100

c********5
发帖数: 207
128
我回贴慢,大家别着急。今天晚上就写到这里,我要好好休息一下。
他还有几个我们都很郁闷的事情。
学爬的动力是抓东西。可他无欲无求呀,如果伸伸手抓不到,就算了。他就趴着,没有
欲望拿东西,如何学爬呀。
一般小孩看大人吃饭,会流哈喇子,吧嗒嘴。可他根本就不敢兴趣。大概两岁以后,恨
不得只吃大米饭(只有家里有这样的孩子才能理解)。面包和肉也行,但是菜和水果都
不爱,要威逼利诱,能吃上几口。因为他过敏性哮喘(症状是咳嗽),也不敢太强迫。
一个不小心,之前的东西全都能吐出来,白忙活呀。2岁以后精力基本都用在他过敏性
哮喘和吃饭上了。但可喜的是每年能多吃几样菜或水果。可是也老费劲了。葡萄好不容
易吃了,只吃绿葡萄。真不是我惯着。是真不吃呀。
还有他吃东西前,先闻。鼻子倒是挺灵敏的。例如我吃红枣,他嫌弃的躲老远。饭里放
了小米,红豆,也是决绝吃的。5岁才肯吃,但是也不爱吃,要抱怨。或吃的比平时少
一半。
应该是3岁以后,很多次在外边playdate的时候,头5分钟或事10分钟一定要耍闹一下。
然后就一切正常了,大家玩的很开心。在家里会好些。(这个我始终没想明白,作的点
在哪里呀)
对好多事情careless,喜欢和小朋友玩,但是处理不好关系。容易发脾气
f*******e
发帖数: 3433
129
这帖子里说的我看出一点端倪了。比如“不怎么吃菜和水果(对食物TEXTURE敏感)”
,"吃东西前,先闻 (sensory的需求)“,“对好多事情careless,喜欢和小朋友玩,
但是处理不好关系。容易发脾气”。他现在6岁了还这样吗?如果是的话,那他可能是
ASPERGER 。
建议你先弄清楚学区怎样handle special education小孩,如果小孩必须去那种专门给
special ed小孩准备的班上课,那就不要做任何evaluation(如果做了评估结果是
DELAY他们会要求你上), 因为ASPERGER的小孩不适合那种班。这种
情况下平时多在网上看看提高这类小孩SOCIAl的文章,买几本social story
book, 比如"The Social Skills Picture Book Teaching play, emotion, and
communication to children with autism" .多带他到外面玩(室外活动有助于
SENSOR的发展)。他的情况不严重,处理得当可能会outgrow.
如果special education学生可以上regular class 和gifted class,那么你可以考虑
给他做speech evaluation. 是否做autism evaluation取决于你们学区如何handle高
功自闭的。据我所知很多学区ASPERGER不给aid(如果这样,没必要evaluate). 我娃所
在的学校对special education娃的service很好,自闭小娃都有one-on-one aid or
shared aid(大了outgrow后就没了)。 而且学区对语言和理解能力很看重,所以
speech therapy给K娃比较多。 虽然也有一个专门的special ed班,但只针对严重自闭
的娃, 而且学校从来不说这是个什么班,估计照顾小孩感觉(我当时也是看到
SCHOOLBOOK才知道有这么个班,好奇为什么这个班7个学生,7个老师,然后
google发现的)。

【在 c********5 的大作中提到】
: 我回贴慢,大家别着急。今天晚上就写到这里,我要好好休息一下。
: 他还有几个我们都很郁闷的事情。
: 学爬的动力是抓东西。可他无欲无求呀,如果伸伸手抓不到,就算了。他就趴着,没有
: 欲望拿东西,如何学爬呀。
: 一般小孩看大人吃饭,会流哈喇子,吧嗒嘴。可他根本就不敢兴趣。大概两岁以后,恨
: 不得只吃大米饭(只有家里有这样的孩子才能理解)。面包和肉也行,但是菜和水果都
: 不爱,要威逼利诱,能吃上几口。因为他过敏性哮喘(症状是咳嗽),也不敢太强迫。
: 一个不小心,之前的东西全都能吐出来,白忙活呀。2岁以后精力基本都用在他过敏性
: 哮喘和吃饭上了。但可喜的是每年能多吃几样菜或水果。可是也老费劲了。葡萄好不容
: 易吃了,只吃绿葡萄。真不是我惯着。是真不吃呀。

P******e
发帖数: 1325
130
这些坐站走的delay,看起来是gross motor skill比较弱,这也是Asperger的red flag
之一。

吧。

【在 c********5 的大作中提到】
: 说说他的短处,请帮我看看。
: 从小各个指标基本就是踩着线达标的。儿医提议让做测试,但是回家训练一下,没几天
: 就会了,例如point finger,用指头抓豆子动作。所以就没做测试。
: 从爬,坐,站,走,delay应该是3个月-9个月不等,后来我都麻木了。
: 再后来语言有delay,但儿医说因为是英文测试,他只懂中文,所以也测不出来。
: 我应该是从他三个月开始,就一直关注自闭,叫名字有反应,和人有互动。有眼神接触
: 。没有小狗动作。出现过要求客人出去,从新进屋的无理要求,但过一阵子也好了。
: 他虽然好多东西都delay,但后来都会了,我想也许就是他运动方面发育比同类人晚吧。
: 搞过baseball,搞不清状况,动作滑稽可笑。而且没兴趣。更喜欢玩沙子或者和同学说
: 笑,

相关主题
今天和幼儿园老师开会review娃的progress,非常upsetExplaining Your Math: Unnecessary at Best, Encumbering at W
好学生去普通公立高中真的会被耽误吗?是不是任我儿子继续迷象棋
陶天才论天才 (转载)9岁, 如何报名考amc10
进入Parenting版参与讨论
P******e
发帖数: 1325
131
我也觉得不太正常,尤其是'恨不得只吃大米饭'简直跟我娃一样啊。
显然也有些 sensory issue。赶紧跟儿医约了谈一谈吧。

【在 f*******e 的大作中提到】
: 这帖子里说的我看出一点端倪了。比如“不怎么吃菜和水果(对食物TEXTURE敏感)”
: ,"吃东西前,先闻 (sensory的需求)“,“对好多事情careless,喜欢和小朋友玩,
: 但是处理不好关系。容易发脾气”。他现在6岁了还这样吗?如果是的话,那他可能是
: ASPERGER 。
: 建议你先弄清楚学区怎样handle special education小孩,如果小孩必须去那种专门给
: special ed小孩准备的班上课,那就不要做任何evaluation(如果做了评估结果是
: DELAY他们会要求你上), 因为ASPERGER的小孩不适合那种班。这种
: 情况下平时多在网上看看提高这类小孩SOCIAl的文章,买几本social story
: book, 比如"The Social Skills Picture Book Teaching play, emotion, and
: communication to children with autism" .多带他到外面玩(室外活动有助于

t******l
发帖数: 10908
132
我觉得现代心理学还很弱。。。Aspergers 可能有很多亚型,而我怀疑其中一种亚型可
能跟 neuroplasticity 有关。
我的猜测是,如果认为大脑用了 90% 的话,那么 neuroplasticity 就首先需要大脑的
“选择性失忆” 的能力,否则 disk full 了啥程序都只能当机。。。而这个 “选择
性失忆” 能力还必须跟 “记忆力” 平衡,否则进水比出水快还是得 disk full 不是
。。。所以真是平均智商啥也记不住,那倒反而也不会 disk full,反而就可能就不会
是 Aspergers 了。
如果是上面那样的话,可能也能解释为啥三年级以后会又显现一批。。。可能就是小时
候 disk 反正还有空,不需要清磁盘,但是三年级以后慢慢开始 disk full 了。
而这个时间点正好跟我家大娃开始发生计算错误,捅篓子的时间点大致吻合。。。粗心
马工删磁盘时,把有用 code 不小心删掉属于正常,没错删有用的 code 那是见证奇迹
!!!
而这个 gross motor 可能也占硬盘,我家大娃去接力队的那个 quarter,尼玛 GPA 又
低上
一茬。
这也可能可以解释为啥有些 Aspergers 难以确诊。。。因为就好比在 track & field
赛季的时候,可能跟普通娃也没啥差别,甚至可能 gross motor 更好。。。差别可能
是在于 track & field 赛季结束后,其他的普通娃都开始删硬盘准备下一个 AMC 8 赛
季了,但 Aspergers 删硬盘的速度慢了点,导致下一个 AMC 8 赛季 都已经开始了,
那 gross motor 还没删干净,所以装 AMC 8 的时候总是跳出 “disk full, please
empty recycle bin“ 的对话框。。。而心理学老军医在 AMC 8 赛季到来之前,通过
捕风捉影的技术来诊断,可能真心很难判断。
当然以上纯属猜测,毫无证据,也就是京片子出租车司机侃中南海的做派。

:你说的这个记忆VS理解,我也感觉有这么回事。我娃理解能力和社交能力上来以后,
我娃虽然VISUAL记忆还是很强,认字写字能力不受影响,但我总觉得他记忆力下降了,
具体怎么下降了却又说不上来,可能还是降得不够厉害。
f*******e
发帖数: 3433
133
观点新颖有趣啊。LOL

【在 t******l 的大作中提到】
: 我觉得现代心理学还很弱。。。Aspergers 可能有很多亚型,而我怀疑其中一种亚型可
: 能跟 neuroplasticity 有关。
: 我的猜测是,如果认为大脑用了 90% 的话,那么 neuroplasticity 就首先需要大脑的
: “选择性失忆” 的能力,否则 disk full 了啥程序都只能当机。。。而这个 “选择
: 性失忆” 能力还必须跟 “记忆力” 平衡,否则进水比出水快还是得 disk full 不是
: 。。。所以真是平均智商啥也记不住,那倒反而也不会 disk full,反而就可能就不会
: 是 Aspergers 了。
: 如果是上面那样的话,可能也能解释为啥三年级以后会又显现一批。。。可能就是小时
: 候 disk 反正还有空,不需要清磁盘,但是三年级以后慢慢开始 disk full 了。
: 而这个时间点正好跟我家大娃开始发生计算错误,捅篓子的时间点大致吻合。。。粗心

t******l
发帖数: 10908
134
所以从这个角度看,pushy mama faulty memory 还是有其相当的必要性的。。。那不
仅是 “选择性失忆”,还是更高一招的 ”选择性改变记忆“。。。总之过了
evolution 选择的玩意儿看起来没用也不一定可以随便删掉,否则可能出大事体。。。
反正人类一思考,上帝就发笑。。。

:观点新颖有趣啊。LOL
t******l
发帖数: 10908
135
你这个链接里的图形不错,但我觉得文章可能把因果关系搞反了。。。我觉得是大脑里
的先验知识经验直感的不同,导致大脑预处理大致概略图像以后,驱动眼睛注重在图像
不同的局部找细节,而不是反过来图像驱动大脑。
或者拿中学几何题目做比方好了。。。即使对于同一个人看同一道题目的图形,如果
wishful
thinking 决定先尝试 欧几/解几/三角 的决定不同,那眼睛观察的重点和找辅助线立
马就不一样了。。。因为欧几的直感是通常是对称/全等/相似三角形;解几的直感通常
是坐标系加直线点斜式表达;三角函数的直感通常是直角三角形但不需要相似。。。至
于盯着图形当中看半天的情况,或者盯着最大的形状看半天的情况,十之八九是几何学
不及格,completely clueless 二十分钟以后,交白卷!

:看看这个吧:
t******l
发帖数: 10908
136
另外刚才提到解几解法,我又看了一眼,我觉得这个 solution 1 从解几解法的角度也
是真心不行。
解几解法也应该是以 CA 为 x 轴,y 轴通过圆心 O,建立直角坐标系,然后一个点斜
式写出直线 AB 的方程,再来一个点斜式写出直线 CB 的方程,然后一个二元一次方程
组解出 B 点的坐标,然后一个勾股定理求出 OB 距离。。。这样至少也不用去解二元
二次方程组。。。我看着 solution 1 平白无故让人解二元二次方程组,我很想骂娘。
。。

:我刚刚去做了这个题目。结果发现我的解法和下面链接的SOLUTION1一模一样:
t******l
发帖数: 10908
137
其实这个解几解法也可以换成等价欧几解法。。。过 O 做 AC 的垂线,过 B 做 AC 的
垂线,过 B 做 AC 的平行线,相似三角形 chase the triangles 不用解线性方程组了
,可能比我前面那个欧几辅助线添法更直接更快一些,少计算一个相似三角形好像。。
。但总之啥解法都犯不着solution 1 那样引入二元二次方程组我觉得。

:另外刚才提到解几解
f*c
发帖数: 687
138
这题3分钟内做出来不大容易。AMC给25题75分钟,平均每题耗时3分钟吧,不专门训练
一下是不行的。

【在 t******l 的大作中提到】
: Solution 1 算二元二次方程比较烦,如果是我娃就听都听不下去,更不要说算,还在
: 旁边笑话我(当然是首先看过了几何解法)。
: 我那个添辅助线的解法不一定在给定时间里能想到。。。但是如果能想到的话,我觉得
: 一般娃会 prefer 我那个添辅助线解法,因为图形表述对 attention span 的要求,比
: 那个解二元二次方程对 attention span 的要求低。。。而且那个辅助线是有迹可寻的
: ,而不是天外飞仙型的辅助线。
: 当然我们祖国把代数练的很熟,另说。。。但就是我在娃面前边讲边算解那个二元二次
: 方程,当中还算错不少地方(我还用了 play with squares 一步就搞出一个线性方程
: ,然后代回去,但这样还是很烦)。。。在知道两个解法的情况下,几何解法绝对比代
: 数解法快,显然不容易出错。。。我觉得大多数人会选择几何解法,如果知道的话。

t******l
发帖数: 10908
139
其实我觉得前面欧几解法的 “小失误”(多算了一个相似三角形慢了几秒),还是挺
能说明普通娃在数学上真心搞不过 hypernumeracy 的记忆力加 attention span。
我觉得普通娃的 intuition,比如这题的例子,在避免二次方程的这种大方向上,是绝
对靠谱的。。。但在具体执行细节上,intuition 非常不靠谱,比如辅助线添成多算一
个三角形,结果慢了几秒就被淘汰。更不要说还有辅助线添对了但是 chase triangles
没 chase 出来的情况。
而史上说那句 “play the middle game like a magician” 的 Rudolf Spielmann,
是个不错的棋手,但是下不过记忆力和计算力都超强的一流棋手。。。这也说明,实战
上 intuition 虽然是最适合普通娃特点的,也少需要记忆题型,并且极大的有利于长
期兴趣。。。但 intuition 有其固有的弱点,最终还是拼不过记忆力和计算力超强,
同时直感也不弱的一流棋手。
所以我真心觉得大部分普通娃在数学上真心拼不过 Aspergers 或 hypernumeracy(数
学直感非常非常好的另说)。。。人不能以短击长,无底线火并,损人不利己。。。我
觉得大部分普通娃,还是用奥外数学打个基础,将来高中转成物理学或计算机学这种依
赖 intuition 更多的玩意儿,避免直接跟 hypernumeracy 天才娃正面交火是真的。

:其实这个解几解法也可以换成等价欧几解法。。。过 O 做 AC 的垂线,过 B 做 AC
的垂线,过 B 做 AC 的平行线,相似三角形 chase the triangles 不用解线性方程组
了,可能比我前面那个欧几辅助线添法更直接更快一些,少计算一个相似三角形好像。
。。但总之啥解法都犯不着solution 1 那样引入二元二次方程组我觉得。
t******l
发帖数: 10908
140
这题是 AIME 而不是 AMC 10 / 12。。。AIME 是 3 小时做 15 题,平均每题 12 分钟
而不是 3 分钟。。。但这题是第四题,所以不应该超过 10 分钟,最好 8 分钟内。。
。当然 1983 年的题目比较容易就是了。
但话说回来,我觉得到这个层次,对题型记忆力不行的,平时 attention span 不够小
时数上不去的,除非数学解题直感非常非常好,否则我觉得真心搞不过 hypernumeracy
型天才娃 。。。我觉得一般普通娃(除了数学直感确实非常非常好的),最多到
AIME 第 8 题,就可以考虑收手改干别的了。

:这题3分钟内做出来不大容易。AMC给25题75分钟,平均每题耗时3分钟吧,不专门训练
:一下是不行的。
相关主题
9岁, 如何报名考amc10二年级女儿的report
有明天考AMC 8的吗?觉得美帝数学进度慢的进来看一下
[转载] 为什么说usamo 简单我招,我是猪
进入Parenting版参与讨论
t******l
发帖数: 10908
141
其实我比你想的更多也更实际。。。我觉得根据目前的 “大脑在推到将要鸡飞狗跳的
时候,利用率已经达到 95%” 的最新猜想,我觉得在这个时候就是天分决定了,即使
attention span 这种小学生憋尿推崇者不认为是天分的玩意儿,对普通娃也还是天分
,绝对拼不过 hypernumeracy 娃。。。所以我觉得差不多就可以收手,否则让大脑一
直开加力燃烧室跑 150% 推力的话,大脑其他方面出大事体的风险可能会指数上升。。。

:据说,有阿斯伯格的人大概通常不会觉得自己有。
t******l
发帖数: 10908
142
其实这是个选材的问题,这就好比女排校队 1.70 米就可以打着玩,健身社交集体活动
随便找理由。。。但 1.70 米去打省队女排的话,就是个重度残疾。。。而对真正的数
学竞赛而言,我觉得记忆力和 attention span,就好比女排的身高。
所以 pushy mama 微信群里说的要刷速度和正确率,也是对的。。。但问题我觉得是普
通娃这点也没法抄。

:其实我比你想的更多也更实际。。。我觉得根据目前的 “大脑在推到将要鸡飞狗跳的
:时候,利用率已经达到 95%” 的最新猜想,我觉得在这个时候就是天分决定了,即
使 attention span 这种小学生憋尿推崇者不认为是天分的玩意儿,对普通娃也还是天分
s***n
发帖数: 1280
143
你对Hypernumeracy有不少误解。绝大多数hypernumeracy的孩子只是对数字感兴趣,但
数学并不好,特别是到了初高中的时候。就好比Hyperlexia的孩子对字母单词感兴趣,
但reading和ELA并不好。这其实都是在同一个环节上出了问题 - 理解。Hypernumeracy
的孩子由于对数字敏感,记忆力好,小学低年级的时候数学都不错,但真正学好数学,
特别是初高中数学,达到竞赛水平,光靠对数字敏感记忆力好是不够的,更需要的是对
数学概念的理解和应用 - 而这正是典型hypernumeracy孩子的弱项。
有一些hypernumeracy的孩子长大后数学也很好,但如果你观察他们的发展,会发现他
们数学进步的时候,也是他hypernumeracy症状减弱甚至消失的时候。换言之,不是
hypernumeracy导致了数学天分,而是孩子outgrow hypernumeracy。
你所讨论的普通娃和hyoernumeracy在解题上的差别,其实和hypernumeracy没啥关系,
只是数学智商高低和数学教育好坏的问题。
另外记忆力好,专注力强只是诊断ASD,Asperger,hypernumeracy,hyperlexia的诊断
参数之一,不是标签。我曾经看到一个研究对比HGT(highly gifted)娃,普通IQ娃,高
功自闭娃的各项ASD诊断参数。这个研究发现很多指数HGT娃和高功自闭娃在高低两头,
普通IQ娃在中间,而且HGT娃和平均IQ娃表现很接近。但在记忆力和专注力的参数上面
,HGT娃接近甚至超过高功自闭娃,和普通IQ娃差别很大。

triangles

【在 t******l 的大作中提到】
: 其实我觉得前面欧几解法的 “小失误”(多算了一个相似三角形慢了几秒),还是挺
: 能说明普通娃在数学上真心搞不过 hypernumeracy 的记忆力加 attention span。
: 我觉得普通娃的 intuition,比如这题的例子,在避免二次方程的这种大方向上,是绝
: 对靠谱的。。。但在具体执行细节上,intuition 非常不靠谱,比如辅助线添成多算一
: 个三角形,结果慢了几秒就被淘汰。更不要说还有辅助线添对了但是 chase triangles
: 没 chase 出来的情况。
: 而史上说那句 “play the middle game like a magician” 的 Rudolf Spielmann,
: 是个不错的棋手,但是下不过记忆力和计算力都超强的一流棋手。。。这也说明,实战
: 上 intuition 虽然是最适合普通娃特点的,也少需要记忆题型,并且极大的有利于长
: 期兴趣。。。但 intuition 有其固有的弱点,最终还是拼不过记忆力和计算力超强,

f*******e
发帖数: 3433
144
计算机编程本身并不很依赖intuition,对逻辑思维要求很高。编程算法是一个逻辑活
,不适用于hypernumeracy的抽象思维, 但很适合asperger的思维. 学图形,网站设计
的可能依赖intuition多一些。不过AMC等确实对编程思维有帮助,里面有些题目可以直
接转成算法题。

triangles

【在 t******l 的大作中提到】
: 其实我觉得前面欧几解法的 “小失误”(多算了一个相似三角形慢了几秒),还是挺
: 能说明普通娃在数学上真心搞不过 hypernumeracy 的记忆力加 attention span。
: 我觉得普通娃的 intuition,比如这题的例子,在避免二次方程的这种大方向上,是绝
: 对靠谱的。。。但在具体执行细节上,intuition 非常不靠谱,比如辅助线添成多算一
: 个三角形,结果慢了几秒就被淘汰。更不要说还有辅助线添对了但是 chase triangles
: 没 chase 出来的情况。
: 而史上说那句 “play the middle game like a magician” 的 Rudolf Spielmann,
: 是个不错的棋手,但是下不过记忆力和计算力都超强的一流棋手。。。这也说明,实战
: 上 intuition 虽然是最适合普通娃特点的,也少需要记忆题型,并且极大的有利于长
: 期兴趣。。。但 intuition 有其固有的弱点,最终还是拼不过记忆力和计算力超强,

f*******e
发帖数: 3433
145
我之前也查了很多HGT娃和ASPERGER娃的区别。两者实际上有很多交集,比如很好的记
忆力,对一些喜欢的东西有很好的FOCUS,都有一些RIGID(但比较轻微),很早也能读
书认字。但HGT和ASPERGER娃一个很大区别是对人对话对故事书的理解能力。HGT对
人的理解能力小时后就很强了,ASPERGER却不行。所以ASPERGER IQ很高的那部分
是属于高智商低情商的。
而且理解力也分种类,我认得一些有ASPERGER的计算机系教授,对专业理解能力很强,
逻辑能力很强,让我自叹不如,但他们对人的理解能力却很弱。高智商低情商就是这样
得来的。很多人认为BILL GATES和FACEBOOK的zuckerberg都有ASPERGER,我也
这么感觉。
也许这么说跟合适。GIFTED和special education并不矛盾。gifted kids可能没有
aspergeror hypernumeray等, 也可能有。但有asperger等的并不一定是HGT。
HGT是高智商高情商,asperger是高智商低情商。之所以
很多竞赛拿高分的有asperger等症状,是因为这些人对社交需求低,能全身心投入竞赛
中,而HGT的对社交有需求,所以投入竞赛的时间少一点,但朋友多,活得更快乐。

Hypernumeracy

【在 s***n 的大作中提到】
: 你对Hypernumeracy有不少误解。绝大多数hypernumeracy的孩子只是对数字感兴趣,但
: 数学并不好,特别是到了初高中的时候。就好比Hyperlexia的孩子对字母单词感兴趣,
: 但reading和ELA并不好。这其实都是在同一个环节上出了问题 - 理解。Hypernumeracy
: 的孩子由于对数字敏感,记忆力好,小学低年级的时候数学都不错,但真正学好数学,
: 特别是初高中数学,达到竞赛水平,光靠对数字敏感记忆力好是不够的,更需要的是对
: 数学概念的理解和应用 - 而这正是典型hypernumeracy孩子的弱项。
: 有一些hypernumeracy的孩子长大后数学也很好,但如果你观察他们的发展,会发现他
: 们数学进步的时候,也是他hypernumeracy症状减弱甚至消失的时候。换言之,不是
: hypernumeracy导致了数学天分,而是孩子outgrow hypernumeracy。
: 你所讨论的普通娃和hyoernumeracy在解题上的差别,其实和hypernumeracy没啥关系,

t******l
发帖数: 10908
146
上面 flyingpie 说到了点子上。。。说的 attention span 是特别指对数学的选择性
attention span。。。说的记忆力是特别指对对数学公式的选择性记忆力。。。这些心
理学 IQ 测试都不会专门去测。
拿实际的例子说,我家大娃上了六年的酱油 chess 班图开心,但她就没有背过开局棋
谱哪怕就一篇。。。但她才开始上法语课,就得意洋洋的主动对我说单词考试她全都背
出来了。。。同样都是死记硬背,IQ 心理学测试加统计数据,根本不能反映这个差别。
而我前面说的 supernumeracy 的优势,不是说智商本身的优势,而是说
supernumeracy 娃,能玩一个计算器玩上三小时。。。而同龄的普通娃,给三小时只去
会看 PBS KIDS Matha Speaks 或 Cyberspace 等等,还把节目换来换去看而不是只盯
着一个。。。这个差别实在太太太太巨大了,即使 supernumeracy outgrow 了,普通
娃也还是望尘莫及。
老实说,我家领导老是问我大娃做数学是不是就敷衍一下,因为领导觉得大娃一直在玩
自由涂鸦。然后我们不断提醒,她才在涂鸦的间隙里做几道题。。。对此我对领导的回
答就是:普通娃,正常,你还图个啥?。。。天天给娃提醒,但别提醒到鸡飞狗跳,这
就是父母的活。


:你对Hypernumeracy有不少误解。绝大多数hypernumeracy的孩子只是对数字感兴趣,
但数学并不好,特别是到了初高中的时候。就好比Hyperlexia的孩子对字母单词感兴趣
,但reading和ELA并不好。这其实都是在同一个环节上出了问题 - 理解。
Hypernumeracy的孩子由于对数字敏感,记忆力好,小学低年级的时候数学都不错,但
真正学好数学,特别是初高中数学,达到竞赛水平,光靠对数字敏感记忆力好是不够的
,更需要的是对数学概念的理解和应用 - 而这正是典型hypernumeracy孩子的弱项。
t******l
发帖数: 10908
147
计算机编程不太依赖 intuition。。。但计算机建模 intuition 能有帮助,虽然
intuition 并不是一定必要的。。。物理竞赛同理。。。所以普通娃能找回更多的场子。

:计算机编程本身并不很依赖intuition,对逻辑思维要求很高。编程算法是一个逻辑活
:,不适用于hypernumeracy的抽象思维, 但很适合asperger的思维. 学图形,网站设
计的可能依赖intuition多一些。不过AMC等确实对编程思维有帮助,里面有些题目可以
直接转成算法题。
t******l
发帖数: 10908
148
另外你提到的心理学测试里 HGT(Highly Gifted) 比 ASD 更专注更持久。。。我觉得
有一个问题是为了比较测试,心理学测试里多半创建了同等的需求。。。而现实世界里
普通娃的问题,常常就是没有需求让普通娃去专注。。。但这个 “没有需求” 也可能
是大脑的自我保护,保证平衡大脑各方面的工作量,同时也避免处于长期加力燃烧状态
。。。所以普通父母老实说,也不敢天天鸡飞狗跳试图一脚绊倒上帝。

:你对Hypernumeracy有不少误解。绝大多数hypernumeracy的孩子只是对数字感兴趣,
但数学并不好,特别是到了初高中的时候。就好比Hyperlexia的孩子对字母单词感兴趣
,但reading和ELA并不好。这其实都是在同一个环节上出了问题 - 理解。
Hypernumeracy的孩子由于对数字敏感,记忆力好,小学低年级的时候数学都不错,但
真正学好数学,特别是初高中数学,达到竞赛水平,光靠对数字敏感记忆力好是不够的
,更需要的是对数学概念的理解和应用 - 而这正是典型hypernumeracy孩子的弱项。
t******l
发帖数: 10908
149
我觉得心理学测试跟实际情况常常差别不小。。。我大娃平时作业忘交漏写名字丢三落
四是家常便饭。。。但如果心理学家据此外推俺大娃会在 County 烧烤大会 4x100 接
力上因为丢三落四而掉棒,那估计就外推错了。。。因为平时啥都不太 care 的,到
County 烧烤大会时正好也不紧张。。。而平时一丝不苟兢兢业业的,到 County 烧烤
大会也可能紧张到腿软(这不是“可能”,好多娃都紧张,人之常情)。。。心理学测
试要测这个,就难度很高。因为两种情况都可能出现,外推不一定有效。而要在心理学
医生小黑屋里,创建 County 烧烤大会的真实氛围,我觉得要上大卫科波菲尔的催眠术
才行!!!!

:另外你提到的心理学测试里 HGT(Highly Gifted) 比 ASD 更专注更持久。。。我觉得
:有一个问题是为了比较测试,心理学测试里多半创建了同等的需求。。。而现实世界
里普通娃的问题,常常就是没有需求让普通娃去专注。。。
S*E
发帖数: 3662
150
反对最后一句。
asperger可能没有那么多朋友,但是能够自得其乐,内心未必不如朋友多的人
快乐。何况参加竞赛也很有乐趣。

【在 f*******e 的大作中提到】
: 我之前也查了很多HGT娃和ASPERGER娃的区别。两者实际上有很多交集,比如很好的记
: 忆力,对一些喜欢的东西有很好的FOCUS,都有一些RIGID(但比较轻微),很早也能读
: 书认字。但HGT和ASPERGER娃一个很大区别是对人对话对故事书的理解能力。HGT对
: 人的理解能力小时后就很强了,ASPERGER却不行。所以ASPERGER IQ很高的那部分
: 是属于高智商低情商的。
: 而且理解力也分种类,我认得一些有ASPERGER的计算机系教授,对专业理解能力很强,
: 逻辑能力很强,让我自叹不如,但他们对人的理解能力却很弱。高智商低情商就是这样
: 得来的。很多人认为BILL GATES和FACEBOOK的zuckerberg都有ASPERGER,我也
: 这么感觉。
: 也许这么说跟合适。GIFTED和special education并不矛盾。gifted kids可能没有

相关主题
大学录取率来点美好的时光
学校早晚要教的东西,早早学会了又如何?有些小学四年级的数学题不太会做
amc 10 的A, B卷有啥区别转载: 从首个IMO季军谈起 by 付云皓
进入Parenting版参与讨论
t******l
发帖数: 10908
151
属实。。。就好比有些 pushy mama 就是喜欢在 Kitchen Granite CounterTop 上
Creative Sex Position Push Push Push。。。但另一些 pushy mama 会嫌 Granite
CounterTop 太冷太硬屁股硌得慌。。。所以古人云:子非鱼,焉知鱼之乐?
当然另一方面,古人也说了,强扭的瓜不甜。。。小康社会里,鸡飞狗跳总之不是太有
必要。

:反对最后一句。

:asperger可能没有那么多朋友,但是能够
:自得其乐,内心未必不如朋友多的人
:快乐。何况参加竞赛也很有乐趣。
t******l
发帖数: 10908
152
这心理测试跟实际的差别,打个比方,就好比心理学研究搞了个 “猛男测试指南”,
于是马蓉大喜,偷偷吩咐凤姐和高圆圆去测试一下。
结果凤姐写的测试报告是王宝强坚挺、宋喆根本就是不举嘛。。。但高圆圆写的测试报
告是王宝强早泄、宋喆持久坚硬欲仙欲死。
于是马蓉把两份心理学测试报告拿在手里,叹了口气,也不知道是到该送哪一位去学区
的 GT 班。

:我觉得心理学测试跟实际情况常常差别不小。。。
t******l
发帖数: 10908
153
还有我觉得这个 HGT (High Gifted) 也可能有 selection bias。。。除了好比陶天才
小学低年级直接超前到高中代数实在没办法的那种,一般如果单单是 gifted 也没造成
任何问题,那为啥要吃饱了撑的去测试?
或者就拿上面的比方,有没有人因为自己每周比别人多几次性高潮,也没其他方面的问
题,就屁颠得不行,主动付钱到医生那边去跪求做个猛男测试的?。。。陶天才那种好
比实在是性高潮太频繁到影响父母日常工作的,另说。。。当然如果是马蓉和宝强不得
不说的故事,那也另说。。。

:我之前也查了很多HGT娃和ASPERGER娃的区别。两者实际上有很多交集,比如很好的记
:忆力,对一些喜欢的东西有很好的FOCUS,都有一些RIGID(但比较轻微),很早也能
读书认字。但HGT和ASPERGER娃一个很大区别是对人对话对故事书的理解能力。HGT对
t******l
发帖数: 10908
154
回到这道题,这道题还有一个更快的标准型不动脑子的解析几何心算解法,也不用解二
次方程组。解法是:
把坐标原点定在 B 点,BC 为 x 轴,BA 为 y 轴。。。记 AC 的中点为 D,得 D 点坐
标为 (1, 3)。。。直线 AC 的斜率为 -3,直线 OD 垂直于直线 AC,所以直线 OD 的
斜率为 1/3。。。线段 OD 的长度为 2*sqrt(10)。。。根据斜率的 rise-over-run 的
概念,得 O 点坐标是 ((1-6) , (3-2)),心算得坐标 (-5, 1),所以心算可知答案是
5^2 + 1^2 = 026。

:我刚刚去做了这个题目。结果发现我的解法和下面链接的SOLUTION1一模一样:
t******l
发帖数: 10908
155
我做完这个以后,看了一下网上 solution 2,,终于明白了 solution 2 的天外飞仙辅
助线。。。但我以前一看 solution 2 时候看不见思路的原因,是 solution 2 先给我
来个 AC = MO,这个是计算长度得到的数字巧合,而不是通常人直觉看到的 geometry
clue。。。所以这些解法的一个共同点,是先注意数字和字母表达式,甚至数字巧合都
能在标准 geometry clue 前先被发现,所以我觉得很可能有 hypernumeracy 的倾向。
。。通常人不会往这个方向想。

:回到这道题,这道题还有一个更快的标准型不动脑子的解析几何心算解法,也不用解
二次方程组。解法是:
t******l
发帖数: 10908
156
而这里还有普通娃思路的另一个弱点,普通娃依赖 intuition,而 intuition 则依赖
于 wishful thinking 所选择的数学模型。。。这导致虽然很多解几解法可以换成等价
欧几解法,但在 wishful thinking 一旦选择了欧几模型还是解几模型以后,其
intuition 是很不一样的。。。甚至解几模型第一步坐标系选的不一样,其后解题的
intuition 就不一样。(写证明的写法,常常是等价转换后让老师看着顺眼的)。。。
这样导致普通娃的 intuition,虽然能保证大概率的走对大方向,但很难第一步尝试就
是最优解方向,所以常常会发生 200 米当中卡壳两次慢了 0.3 秒,预赛被出局。

:回到这道题,这道题还有一个更快的标准型不动脑子的解析几何心算解法,也不用解
二次方程组。解法是:
t******l
发帖数: 10908
157
当然另一方面,我觉得教依赖 intuition 的普通娃,首先是诚实。。。欧几思路就是
欧几思路,解几思路就是解几思路。。。不能因为数学上等价,就直接教娃等价转换以
后的解法。。。因为数学上等价,不等于解题的 intuition 上等价。
另一方面,这个我觉得也就是 AMC 选择题,AIME填空题,虽然看起来反人类,但是优
点是在于每个普通娃的 intuition 直觉不一样,一开始避免要求娃写出大家认可的大
伙儿看着舒服的解题解释/证明。。。因为 problem solving 的教育首先是解题人自己
解着舒服正确率高,等价转换成大众喜闻乐见的表达方式是第二步的事

:而这里还有普通娃思路的另一个弱点,普通娃依赖 intuition,而 intuition 则依赖
:于 wishful thinking 所选择的数学模型。。。这导致虽然很多解几解法可以换成等
价欧几解法,但在 wishful thinking 一旦选择了欧几模型还是解几模型以后,其
d****r
发帖数: 300
158
看到很多专业分析,有数学牛娃都有天才病的感觉,但可以感到你没真正见过进过
usamo, mop乃至imo的中国孩子。我有幸见过几个,想说几点。他们都是正常阳光的孩
子,都很全面,对数学是真爱,化了很多时间做题,当然天赋一定程度决定了最好成绩
,大家最后都差不多努力。推推小孩数学没错,就算aime做到8题,对上大学有帮助,
尤其女生。

依赖

【在 t******l 的大作中提到】
: 当然另一方面,我觉得教依赖 intuition 的普通娃,首先是诚实。。。欧几思路就是
: 欧几思路,解几思路就是解几思路。。。不能因为数学上等价,就直接教娃等价转换以
: 后的解法。。。因为数学上等价,不等于解题的 intuition 上等价。
: 另一方面,这个我觉得也就是 AMC 选择题,AIME填空题,虽然看起来反人类,但是优
: 点是在于每个普通娃的 intuition 直觉不一样,一开始避免要求娃写出大家认可的大
: 伙儿看着舒服的解题解释/证明。。。因为 problem solving 的教育首先是解题人自己
: 解着舒服正确率高,等价转换成大众喜闻乐见的表达方式是第二步的事
:
: :而这里还有普通娃思路的另一个弱点,普通娃依赖 intuition,而 intuition 则依赖
: :于 wishful thinking 所选择的数学模型。。。这导致虽然很多解几解法可以换成等

f*******e
发帖数: 3433
159
阳光和高功自闭也不矛盾。我家的在他班上是最阳光的一个,因为即使别人不大喜欢他
,他也看不出,自得其乐。这也是为什么很多高功自闭诊断的晚的原因。而且轻微的
高功自闭长大后可能会OUTGROW,但带来的TALENT并不会消失。

【在 d****r 的大作中提到】
: 看到很多专业分析,有数学牛娃都有天才病的感觉,但可以感到你没真正见过进过
: usamo, mop乃至imo的中国孩子。我有幸见过几个,想说几点。他们都是正常阳光的孩
: 子,都很全面,对数学是真爱,化了很多时间做题,当然天赋一定程度决定了最好成绩
: ,大家最后都差不多努力。推推小孩数学没错,就算aime做到8题,对上大学有帮助,
: 尤其女生。
:
: 依赖

t******l
发帖数: 10908
160
俺话糙理不糙的说,在真爱数学娃前面,普通娃就是渣。。。人真爱数学娃的智商比普
通娃高;intuition 比普通娃好;而且对数学是真爱,在数学上化的时间比 Aspergers
都还多
。。。我真心觉得在真爱娃数学面前,Aspergers 都是炮灰;普通娃就不用说了,废柴。
真心觉得普通非真爱数学娃,推数学差不多就可以了,中庸之道适可而止才是实在的做
派。

:看到很多专业分析,有数学牛娃都有天才病的感觉,但可以感到你没真正见过进过
:usamo, mop乃至imo的中国孩子。我有幸见过几个,想说几点。他们都是正常阳光的孩
相关主题
新加坡数学对小学生帮助大吗?好学生去普通公立高中真的会被耽误吗?
WISC-V 智商测试陶天才论天才 (转载)
今天和幼儿园老师开会review娃的progress,非常upsetExplaining Your Math: Unnecessary at Best, Encumbering at W
进入Parenting版参与讨论
t******l
发帖数: 10908
161
我觉得普通数学偷懒娃搞到不超过 AIME 第 8 题的目的,倒不一定是对大学入学有多
大用处,特别是当今 AA 的大形势下。
我觉得主要还是在于对一个人来说,首先是人生要 consistent。。。而对于一个从小
就盼着收拾书包走人的数学偷懒娃而言,既然知道自己会一辈子偷懒,那还不如从小开
始学习偷懒,如何在偷懒的情况下,在早早收拾书包走人之前,把 AMC / AIME 尽可能
往后多搞几题。。。这样在大学毕业长大以后,也就会比较自然而顺理成章,而不会突
然有一种“假若生活欺骗了你”的感觉。。。

:看到很多专业分析,有数学牛娃都有天才病的感觉,但可以感到你没真正见过进过
:usamo, mop乃至imo的中国孩子。我有幸见过几个,想说几点。他们都是正常阳光的孩
a****n
发帖数: 8553
162
我说你怎么老是煎饼果子…一套一套的…感情是被LD逼的…还是欠火候…应该自己主动
给自己找根红线跨以转移LD的注意力…

【在 t******l 的大作中提到】
: :以至于上K,我就放羊。
: 这个问题对于通常娃(也就是普通娃)而言,放不放都是羊,因为上帝放了一条细细
: 的红线,普通父母普通鸡飞狗跳是根本跨不过那条红线的。。。比如一条红线就是
: attention span,另一条红线就是 “扔 Kumon 习题集”。。。当然我家领导老是梦想
: 跨过那条看不见的红线,我都不希得说你特么这是跟上帝过不去啊。。。至于一脚绊倒
: 上帝的成功率为多少吧,自己看!
:
: 3

t******l
发帖数: 10908
163
为了比蛋而使劲擦蛋,我觉得也是人之常情。。。但在信息交流无比通倡的微信时代,
我觉得要谨记克制 “一脚绊倒上帝” 的冲动,这样可以避免擦蛋时用力过猛直接把蛋
擦碎,也算是人之常情。。。跨过了红线而不自知,导致赌博风险极度增大,才是新时
代的新问题。。。不过正如黄小木同学所说的,control freak 在鸡飞狗跳的时候能不
能知道自己是 control freak,可能也是个问题。

:我说你怎么老是煎饼果子…一套一套的…感情是被LD逼的…还是欠
火候…应该自己主动给自己找根红线跨以转移LD的注意力…
t******l
发帖数: 10908
164
或者说,这就像古代没 AoPS 网站的时候,大伙儿好比都是拿着 AK47 鸟铳打坦克。。
。但现在有了 AoPS 网站了,就好比委员长的美援 RPG 到货了。。。然后一性格刚烈
的小战士立马一发 RPG 出去,对面坦克应声倒地。小战士正兴高采烈的摆出英雄姿势
的时候,被旁人指了指背后的火焰区。。。火焰区内那位不幸的哥们已经满脸青烟地挂
了。。。战地照片右下角一行小字曰:collateral damage。。。

:为了比蛋而使劲擦蛋,
s***n
发帖数: 1280
165
你对普通娃的定义是什么?是智商和学习成绩Average孩子,还是无ASD/ADHD等问题的
正常孩子?如果是后者的话,我不同意你说的现实世界里没有需求让正常娃去专注。
以前看过一个研究,分析智商,专注力和学习成绩的关联,发现专注力和学习成绩的关
联性远高于智商对学习成绩的影响。所以智商和学习成绩Average孩子可能是因为没有
需求对学习去专注,所以学习时不专注,导致学习成绩Average。甚至很多高智商的孩
子也成绩一般。但如果你认为正常孩子都无法专注的话,就言过了。很多智商普通的孩
子学习成绩优秀,很多就是靠的专注力方面的优势。
对学习的无所需求,不是大脑的自我保护,是教育的不成功。我以前的帖子里强调兴趣
。兴趣是怎么帮助学习的?不是通过改变智商,不是通过提高记忆力,而是通过提高专
注力。你对一个东西感兴趣,不会提高你的智商或记忆能力,但可以提高你对这个东西
的专注力。很多推娃失败的根本原因就是因为忽视了孩子自身对学习某一项目的兴趣。
说起兴趣的作用,想起以前看过的另一个相关的研究。心理学家让一群孩子秉住呼吸,
看能坚持多长。一次是直接让他们秉住呼吸,一次是告诉他们让他们扮演皇宫的卫兵,
他们要秉住呼吸观察坏人。结果第二种情况下孩子秉住呼吸的时间提高了一倍多。

【在 t******l 的大作中提到】
: 另外你提到的心理学测试里 HGT(Highly Gifted) 比 ASD 更专注更持久。。。我觉得
: 有一个问题是为了比较测试,心理学测试里多半创建了同等的需求。。。而现实世界里
: 普通娃的问题,常常就是没有需求让普通娃去专注。。。但这个 “没有需求” 也可能
: 是大脑的自我保护,保证平衡大脑各方面的工作量,同时也避免处于长期加力燃烧状态
: 。。。所以普通父母老实说,也不敢天天鸡飞狗跳试图一脚绊倒上帝。
:
: :你对Hypernumeracy有不少误解。绝大多数hypernumeracy的孩子只是对数字感兴趣,
: 但数学并不好,特别是到了初高中的时候。就好比Hyperlexia的孩子对字母单词感兴趣
: ,但reading和ELA并不好。这其实都是在同一个环节上出了问题 - 理解。
: Hypernumeracy的孩子由于对数字敏感,记忆力好,小学低年级的时候数学都不错,但

s***n
发帖数: 1280
166
我同意你说的"GIFTED和special education并不矛盾。gifted kids可能没有
aspergeror hypernumeracy等, 也可能有。但有asperger等的并不一定是HGT" 教育方
面最近出的一个新分类Twice Exceptional就是特指那些又gifted又有learning
disability (ASD/ADHD, etc)的孩子。我觉得这种描述比认为Asperger的孩子数理好更
贴近实际情况,也更有助于对这些孩子的教育。
这个班上讨论的Asperger的孩子很多数学不错,所以容易给大家一个假象,以为
Asperger或hypernumeracy的孩子学习数学有优势。我以前也有这样的感觉。但看了身
边和网上的一些例子后,我倾向于认为Asperger或hypernumeracy的孩子在数学方面没
有优势,而他们的弱点就是理解能力。2007年的一个相关研究发现,the majority of
individuals with AS/HFA have average mathematical ability,甚至 have a
significant but clinically modest math weakness。Asperger或hypernumeracy的孩
子在记忆力和专注力上是有优势的,他们应该数学水平不错,但实际情况是他们大多数
数学水平一般,而且有显著弱点。这里面缺失的一环就是我前文提到的理解能力。
那网上和现实生活中那些数学很突出的Asperger娃是怎么回事?我觉得是因为这些孩子
在数学或逻辑思维方面是Gifted。他们是Twice Exceptional,他们是Asperger孩子中的
一小撮。他们在数学方面的理解能力强,不是因为Asperger,是因为Gifted。Asperger
的牛人不仅出现在数理计算机领域,也出现在其它领域,甚至文艺方面,比如建筑,导
演,演艺。这些截然不同领域内的Asperger牛人,不同的不是在Asperger分类或亚型上
面,而是在Gifted领域上面。
对Twice Exceptional的孩子,教育的重点不能只在孩子的learning disability,ASD/
ADHD上面,必须兼顾孩子Gifted方面的发展。
待续...

【在 f*******e 的大作中提到】
: 我之前也查了很多HGT娃和ASPERGER娃的区别。两者实际上有很多交集,比如很好的记
: 忆力,对一些喜欢的东西有很好的FOCUS,都有一些RIGID(但比较轻微),很早也能读
: 书认字。但HGT和ASPERGER娃一个很大区别是对人对话对故事书的理解能力。HGT对
: 人的理解能力小时后就很强了,ASPERGER却不行。所以ASPERGER IQ很高的那部分
: 是属于高智商低情商的。
: 而且理解力也分种类,我认得一些有ASPERGER的计算机系教授,对专业理解能力很强,
: 逻辑能力很强,让我自叹不如,但他们对人的理解能力却很弱。高智商低情商就是这样
: 得来的。很多人认为BILL GATES和FACEBOOK的zuckerberg都有ASPERGER,我也
: 这么感觉。
: 也许这么说跟合适。GIFTED和special education并不矛盾。gifted kids可能没有

f*******e
发帖数: 3433
167
这方面我比较有发言权。我读大学前在一个高考竞争力很强的省,那时全省大约8个学
生才有一个能上大学。而我是属于小学成绩不好,被老师说可能考不上初中,后来依靠
住重点中学旁边的优势进了重点中学(那时按照学区上初中)。初三开始成绩突飞猛进
,高中就在我这最好的中学成了班上第一,高考全省前50。原因是努力和对学习的专注
力。说说下面几点:
1.高中班上第一,高考全省前50,是否意味我智商高?NO。我觉得我智力方面中等偏上
,但我确实非常努力,除了吃饭睡觉和短暂的休息时间都在学习,而且拿班上第一位我
个人觉得是靠题海战术。奥赛的题目我做不来。所以像潮水说的,我是普通娃,无论如
何推,只能上到一个程度,达不到顶尖水平。
2. 智力对于分数是否重要?很重要。我当时班上和我一样努力的不少,甚至比我更努
力更专注,但很多却只是中等。而有些不是那么努力的却是前几名。这些人考试考不过
我,但奥赛比我行。
3. 有些ASPERGER娃智商确实没法比。我大学有个同学,很内向,典型高智商低情商那
种。当年是他门省的高考状元,大学作业中有些很难的数学题,每次全班就他一人会做
,然后大家都是看了他给的答案写的作业。那时就觉得这人脑子长得和大家都不一样,
典型的数学思维
4. 工作后发现很多高中学习成绩一般的比一些成绩好的混得更好,所以结论是不要过
于纠结考试的成绩和智商。职业上全面发展,工作的方向是自己专长更重要。比如
上面这个数学强人,读了研究生,毕业后像其他大部分同学一样到公司工作,专长
被埋没了,也变成默默无闻了。他如果以后做和数学相关研究可能会更成功些。

【在 s***n 的大作中提到】
: 你对普通娃的定义是什么?是智商和学习成绩Average孩子,还是无ASD/ADHD等问题的
: 正常孩子?如果是后者的话,我不同意你说的现实世界里没有需求让正常娃去专注。
: 以前看过一个研究,分析智商,专注力和学习成绩的关联,发现专注力和学习成绩的关
: 联性远高于智商对学习成绩的影响。所以智商和学习成绩Average孩子可能是因为没有
: 需求对学习去专注,所以学习时不专注,导致学习成绩Average。甚至很多高智商的孩
: 子也成绩一般。但如果你认为正常孩子都无法专注的话,就言过了。很多智商普通的孩
: 子学习成绩优秀,很多就是靠的专注力方面的优势。
: 对学习的无所需求,不是大脑的自我保护,是教育的不成功。我以前的帖子里强调兴趣
: 。兴趣是怎么帮助学习的?不是通过改变智商,不是通过提高记忆力,而是通过提高专
: 注力。你对一个东西感兴趣,不会提高你的智商或记忆能力,但可以提高你对这个东西

f*******e
发帖数: 3433
168
ASPERGER高智商的也确实是少数,因为ASPERGER诊断只要求达到自闭的两项或两项以上
+IQ大于或者等于平均。之所以ASPERGER一些小孩容易被注意,是因为他们有某些远超
于常人的地方,比如有些有照相式记忆,有些对数字及其敏感,有些逻辑能力超强。
比如说普通小孩一些方面评估可能是1 1 1, HGT可能是1.5 1.5 1.5, 那些某方面超
强的ASPERGER可能是 2 1 0.5 (某方面超强,另一方面就会超弱)

of

【在 s***n 的大作中提到】
: 我同意你说的"GIFTED和special education并不矛盾。gifted kids可能没有
: aspergeror hypernumeracy等, 也可能有。但有asperger等的并不一定是HGT" 教育方
: 面最近出的一个新分类Twice Exceptional就是特指那些又gifted又有learning
: disability (ASD/ADHD, etc)的孩子。我觉得这种描述比认为Asperger的孩子数理好更
: 贴近实际情况,也更有助于对这些孩子的教育。
: 这个班上讨论的Asperger的孩子很多数学不错,所以容易给大家一个假象,以为
: Asperger或hypernumeracy的孩子学习数学有优势。我以前也有这样的感觉。但看了身
: 边和网上的一些例子后,我倾向于认为Asperger或hypernumeracy的孩子在数学方面没
: 有优势,而他们的弱点就是理解能力。2007年的一个相关研究发现,the majority of
: individuals with AS/HFA have average mathematical ability,甚至 have a

n****f
发帖数: 3580
169
总结很全面,中肯。

【在 f*******e 的大作中提到】
: 这方面我比较有发言权。我读大学前在一个高考竞争力很强的省,那时全省大约8个学
: 生才有一个能上大学。而我是属于小学成绩不好,被老师说可能考不上初中,后来依靠
: 住重点中学旁边的优势进了重点中学(那时按照学区上初中)。初三开始成绩突飞猛进
: ,高中就在我这最好的中学成了班上第一,高考全省前50。原因是努力和对学习的专注
: 力。说说下面几点:
: 1.高中班上第一,高考全省前50,是否意味我智商高?NO。我觉得我智力方面中等偏上
: ,但我确实非常努力,除了吃饭睡觉和短暂的休息时间都在学习,而且拿班上第一位我
: 个人觉得是靠题海战术。奥赛的题目我做不来。所以像潮水说的,我是普通娃,无论如
: 何推,只能上到一个程度,达不到顶尖水平。
: 2. 智力对于分数是否重要?很重要。我当时班上和我一样努力的不少,甚至比我更努

t******l
发帖数: 10908
170
我觉得你提到的屏住呼吸的测试非常不靠谱。。。如果测试的小孩里面一半是耐力型的
,另一半是 100% 速度肌型的,速度肌型的可以直接 pass out 大脑缺氧脑损伤。
当然不是速度肌型的不会有体会就是了。。。就好比学校女生 4x100 接力队的,多次
短跑训练以后都会有一点点肌肉拉伤 hamstring 略有不适啥的,跳远的都会有一点
jumper's knee 的感觉,休息不练后恢复。。。但学校一般女生体育课大部分不太会有
这种现象。。。原因就是因为大部分平均女生没有校短跑队的爆发力,在同等距离同等
次数下,对肌肉肌腱不造成校短跑队那么大的压力。

:你对普通娃的定义是什么?是智商和学习成绩Average孩子,还是无ASD/ADHD等问题的
相关主题
Explaining Your Math: Unnecessary at Best, Encumbering at W有明天考AMC 8的吗?
是不是任我儿子继续迷象棋[转载] 为什么说usamo 简单
9岁, 如何报名考amc10二年级女儿的report
进入Parenting版参与讨论
t******l
发帖数: 10908
171
我觉得能努力有专注力,其实也跟 trait 有相当的关系。。。所以你的说法就有点像
endurance runner 说 sprinter 为啥不更努力点把 200 米跑成 1600 米,来自巴哈马
群岛的 Shaunae Miller 同志以一个鱼跃前扑很好的回答了这个问题。

:这方面我比较有发言权。我读大学前在一个高考竞争力很强的省,那时全省大约8个学
:生才有一个能上大学。而我是属于小学成绩不好,被老师说可能考不上初中,后来依
靠住重点中学旁边的优势进了重点中学(那时按照学区上初中)。初三开始成绩突飞猛
进,高中就在我这最好的中学成了班上第一,高考全省前50。原因是努力和对学习的专
注力。说说下面几点:
t******l
发帖数: 10908
172
另外你说的那个数学强人默默无闻,事后猪哥亮总是容易得多。。。但有人大脑过多思
考会引起胃病、过敏、偏执、分裂啥的,如果这样还搞成个张益唐未遂的话,那时事后
猪哥亮的话,会不会说还不如去公司里混个默默无闻,至少不折寿不是?
所以基于事前不可知,而生物的第一本能是 “理性经济人求生存”,而不是为 “人类
一小步而做董存瑞黄继光,还未遂”,我觉得高中在 USAMO Top 500 强以外的,选择
去公司混默默无闻,是一种理性的选择。。。因为虽然幸福与否的标准是自己定义的,
但成功与否的标准是社会定义的,成功的机会也是社会给的。。。既然社会定义了
USAMO Top 500 的标准和机会,但高中没能入围的话,虽然不是 100% 抹杀了其后成为
张益唐的可能性,但总之是荆棘满途没错的。傻子都能看到这点。。。而且退一步说,
张益唐虽然成功了,但其个人的人生,对很多市井小民来说,还是蛮苦鼻的,实话实说。

:这方面我比较有发言权。我读大学前在一个高考竞争力很强的省,那时全省大约8个学
:生才有一个能上大学。而我是属于小学成绩不好,被老师说可能考不上初中,后来依
靠住重点中学旁边的优势进了重点中学(那时按照学区上初中)。初三开始成绩突飞猛
进,高中就在我这最好的中学成了班上第一,高考全省前50。原因是努力和对学习的专
注力。说说下面几点:
f*******e
发帖数: 3433
173
我的意思是他在学校找个对数学要求高的专业读博然后做教授可能会比在一个商业公司
工作更如鱼得水,因为商业公司不适合他,也用不上他的数学。 他当年是全省理科高
考状元,全国一年也没100高考状元啊。张益唐那样的不可复制,他那样的生活我并不
觉得幸福。

说。

【在 t******l 的大作中提到】
: 另外你说的那个数学强人默默无闻,事后猪哥亮总是容易得多。。。但有人大脑过多思
: 考会引起胃病、过敏、偏执、分裂啥的,如果这样还搞成个张益唐未遂的话,那时事后
: 猪哥亮的话,会不会说还不如去公司里混个默默无闻,至少不折寿不是?
: 所以基于事前不可知,而生物的第一本能是 “理性经济人求生存”,而不是为 “人类
: 一小步而做董存瑞黄继光,还未遂”,我觉得高中在 USAMO Top 500 强以外的,选择
: 去公司混默默无闻,是一种理性的选择。。。因为虽然幸福与否的标准是自己定义的,
: 但成功与否的标准是社会定义的,成功的机会也是社会给的。。。既然社会定义了
: USAMO Top 500 的标准和机会,但高中没能入围的话,虽然不是 100% 抹杀了其后成为
: 张益唐的可能性,但总之是荆棘满途没错的。傻子都能看到这点。。。而且退一步说,
: 张益唐虽然成功了,但其个人的人生,对很多市井小民来说,还是蛮苦鼻的,实话实说。

t******l
发帖数: 10908
174
高考状元是两小时内 routine problem 普通题正确率最高。
USAMO qualifiers 是三小时内 non-routine problem 相对难题解出率最高。(考虑到
满分真心不多)。
数学系是三年内解出前人从来没解出过的问题。。。当然二十年磨一剑也不是不行,前
半生稍微苦点。
按以上分析,你要是觉得高考状元比 USAMO 更适合去数学系的话,那柜台珠算冠军最
强大脑三八红旗手阿姨最最适合去数学系了。。。

:我的意思是他在学校找个对数学要求高的专业读博然后做教授可能会比在一个商业公
司工作更如鱼得水,因为商业公司不适合他,也用不上他的数学。 他当年是全省理科高
:考状元,全国一年也没100高考状元啊。张益唐那样的不可复制,他那样的生活我并不
f*******e
发帖数: 3433
175
我们不是数学系的。我说的是他选个我们这个大专业中一个相对对数学要求更高的小
专业当教授,而不是数学系。具体我什么专业我就不说了,总之里面有些小
专业对数学要求高点(但远远低于数学系),有些要求低。而且我没说他比那些
奥赛什么的更适合数学系啊。

科高
并不

【在 t******l 的大作中提到】
: 高考状元是两小时内 routine problem 普通题正确率最高。
: USAMO qualifiers 是三小时内 non-routine problem 相对难题解出率最高。(考虑到
: 满分真心不多)。
: 数学系是三年内解出前人从来没解出过的问题。。。当然二十年磨一剑也不是不行,前
: 半生稍微苦点。
: 按以上分析,你要是觉得高考状元比 USAMO 更适合去数学系的话,那柜台珠算冠军最
: 强大脑三八红旗手阿姨最最适合去数学系了。。。
:
: :我的意思是他在学校找个对数学要求高的专业读博然后做教授可能会比在一个商业公
: 司工作更如鱼得水,因为商业公司不适合他,也用不上他的数学。 他当年是全省理科高

t******l
发帖数: 10908
176
这是战术层次问题,你说的战术有可能更好些。。。但差别可能也有限。况且很多时候
市场需求的分量,比数学占的比重多点少点,影响因子更大。。。若水说了,都是命。
。。

:我们又不是数学系的。我说的是他选个我们这个大专业中一个相对对数学要求更高的
小专业,而不是数学系
t******l
发帖数: 10908
177
USAMO qualifiers 每年就有 500 个。。。美国一年州立学校把鸟不拉屎地都算上,一
年能不能有 500 个数学教授位子的空缺?

:我们不是数学系的。我说的是他选个我们这个大专业中一个相对对数学要求更高的小
:专业当教授,而不是数学系。具体我什么专业我就不说了,总之里面有些小
f*******e
发帖数: 3433
178
他在国内工作。国内教职在我毕业那个时候还是很好找的。

【在 t******l 的大作中提到】
: USAMO qualifiers 每年就有 500 个。。。美国一年州立学校把鸟不拉屎地都算上,一
: 年能不能有 500 个数学教授位子的空缺?
:
: :我们不是数学系的。我说的是他选个我们这个大专业中一个相对对数学要求更高的小
: :专业当教授,而不是数学系。具体我什么专业我就不说了,总之里面有些小

w*********y
发帖数: 7895
179
这个我有不同的经验。 我家一个娃,理解能力和社会能力都还OK,所以记忆力也好。
因为理解了,可以举一反三。如果不理解的时候,更容易忘记。
另一个娃,理解力比较不好,所以记忆很差。讲东西,就好像天上的云,很快就飘过了。
不留痕迹。

【在 f*******e 的大作中提到】
: 你说的这个记忆VS理解,我也感觉有这么回事。我娃理解能力和社交能力上来以后,我
: 娃虽然VISUAL记忆还是很强,认字写字能力不受影响,但我总觉得他记忆力下降了,具
: 体怎么下降了却又说不上来,可能还是降得不够厉害。
:
: 4x100

w*********y
发帖数: 7895
180
根据我自己的观察和知识,我觉的我家一个娃比较好,是因为她的ATTENTION和MEMORY
配合很好。
所以显的比另一个聪明。另一个因为这2者配合不好,所以记不住自己学过的东西,一
定要反复记忆,
才能够明白。

【在 t******l 的大作中提到】
: 我觉得中肯的说,有一种可能是不同的人对于 (1) photographic memory, (2)
: intuition, (3) attention span on "boring" tasks 的标准和接受程度不一样,有可
: 能导致这种看法的差别。
: 或者用另一个比方,如果要赢一盘象棋,不外乎是著名的以下三条路数的结合:
: (1) Play the opening-game like a book. -- Photographic memory.
: (2) Play the middle-game like a magician. -- Intuition.
: (3) Play the end-game like a machine. -- Attention Span (on "boring" task).
: 但是各家根据自己的实际情况,混合比都不太一样,可能导致这类争议。

相关主题
觉得美帝数学进度慢的进来看一下学校早晚要教的东西,早早学会了又如何?
我招,我是猪amc 10 的A, B卷有啥区别
大学录取率来点美好的时光
进入Parenting版参与讨论
s**********y
发帖数: 509
181
国内高考在美国好像没有对应。 一定要类比, 可能更考board擦不多。 看记忆, 看
耐心, 看稳。

【在 f*******e 的大作中提到】
: 这方面我比较有发言权。我读大学前在一个高考竞争力很强的省,那时全省大约8个学
: 生才有一个能上大学。而我是属于小学成绩不好,被老师说可能考不上初中,后来依靠
: 住重点中学旁边的优势进了重点中学(那时按照学区上初中)。初三开始成绩突飞猛进
: ,高中就在我这最好的中学成了班上第一,高考全省前50。原因是努力和对学习的专注
: 力。说说下面几点:
: 1.高中班上第一,高考全省前50,是否意味我智商高?NO。我觉得我智力方面中等偏上
: ,但我确实非常努力,除了吃饭睡觉和短暂的休息时间都在学习,而且拿班上第一位我
: 个人觉得是靠题海战术。奥赛的题目我做不来。所以像潮水说的,我是普通娃,无论如
: 何推,只能上到一个程度,达不到顶尖水平。
: 2. 智力对于分数是否重要?很重要。我当时班上和我一样努力的不少,甚至比我更努

f*c
发帖数: 687
182
美国乱七八糟的竞赛太多,不知道哪个真的算数。前些日子说某某没被某藤录取抱怨云
云,是不是因为得的这个奖那个奖不是那个啥?我估计USAMO qualifier应该是算数的
,这个上大学能“加分”吗?具体怎么个过程?
记得以前国际足联某大佬看见中国队经常赢个友谊赛,什么桑普多利亚之类的,就撂下
一句狠话:中国队要赢那些必须赢的比赛。嘿嘿嘿,这个好像现在都没做到。。。

【在 t******l 的大作中提到】
: USAMO qualifiers 每年就有 500 个。。。美国一年州立学校把鸟不拉屎地都算上,一
: 年能不能有 500 个数学教授位子的空缺?
:
: :我们不是数学系的。我说的是他选个我们这个大专业中一个相对对数学要求更高的小
: :专业当教授,而不是数学系。具体我什么专业我就不说了,总之里面有些小

t******l
发帖数: 10908
183
如果是去 IMO 美国队的话,AMC 10 / 12 => AIME => USAMO => MOSP,
只看分数,不看肤色。。。其他的都是友谊赛。
如果是去藤校的话,黄皮做好心理准备啥竞赛都不加分,黑皮做好心理准备啥竞赛都加
分。。。俗称 holistic AA 入学标准。

:美国乱七八糟的竞赛太多,不知道哪个真的算数。前些日子说某某没被某藤录取抱怨
云云,是不是因为得的这个奖那个奖不是那个啥?我估计USAMO qualifier应该是算数的
:,这个上大学能“加分”吗?具体怎么个过程?
f*c
发帖数: 687
184
俺的意思是,AIME或者USAMO qualifier这些级别对进大学能帮到什么程度,比如说进
个UCLA/UCB什么的。个人觉得近年来数学竞赛有正规化,职业化的倾向。真想做数学家
的话,16/7岁这个年纪往竞赛这个方向走得太厉害多少有些浪费的感觉。

数的

【在 t******l 的大作中提到】
: 如果是去 IMO 美国队的话,AMC 10 / 12 => AIME => USAMO => MOSP,
: 只看分数,不看肤色。。。其他的都是友谊赛。
: 如果是去藤校的话,黄皮做好心理准备啥竞赛都不加分,黑皮做好心理准备啥竞赛都加
: 分。。。俗称 holistic AA 入学标准。
:
: :美国乱七八糟的竞赛太多,不知道哪个真的算数。前些日子说某某没被某藤录取抱怨
: 云云,是不是因为得的这个奖那个奖不是那个啥?我估计USAMO qualifier应该是算数的
: :,这个上大学能“加分”吗?具体怎么个过程?

t******l
发帖数: 10908
185
大部分娃并不是想做数学家,而是想给 STEM 打个数学的基础。。。如果从打数学基础
这个角度看,AMC 10 / 12 => AIME,比起祖国数学高考,费效比要高。。。而美
国的 SAT / ACT 数学,对理工科而言完全不够。
对于大学入学而言,对黄皮而言,可能都没啥用。。。但美帝州大高校资源丰富,外加
相对祖国而言更宽进严出一些。。。所以考虑娃不要大学二年级主动投诚不需要数学的
文科专业或生物专业,我觉得 AMC 10 / 12 和 AIME 上适当花点时间还是值得的。

:俺的意思是,AIME或者USAMO qualifier这些级别对进大学能帮到什么程度,比如说进
:个UCLA/UCB什么的。个人觉得近年来数学竞赛有正规化,职业化的倾向。真想做数学
家的话,16/7岁这个年纪往竞赛这个方向走得太厉害多少有些浪费的感觉。
t******l
发帖数: 10908
186
我们说的本质上不矛盾,这是 homogeneous vs heterogeneous 的差别。

:根据我自己的观察和知识,我觉的我家一个娃比较好,是因为她的ATTENTION和
MEMORY
:配合很好。
f*c
发帖数: 687
187
听上去大学录取形势严峻啊。俺那个年代国内进了TOP500基本上就保送了,单数学大省
能有四五十人吧。竞赛这个东西没有录取优惠是不太值得。AMC算是够基础了,但它那
个时间限制也是需要一定的题海磨练的。

说进

【在 t******l 的大作中提到】
: 大部分娃并不是想做数学家,而是想给 STEM 打个数学的基础。。。如果从打数学基础
: 这个角度看,AMC 10 / 12 => AIME,比起祖国数学高考,费效比要高。。。而美
: 国的 SAT / ACT 数学,对理工科而言完全不够。
: 对于大学入学而言,对黄皮而言,可能都没啥用。。。但美帝州大高校资源丰富,外加
: 相对祖国而言更宽进严出一些。。。所以考虑娃不要大学二年级主动投诚不需要数学的
: 文科专业或生物专业,我觉得 AMC 10 / 12 和 AIME 上适当花点时间还是值得的。
:
: :俺的意思是,AIME或者USAMO qualifier这些级别对进大学能帮到什么程度,比如说进
: :个UCLA/UCB什么的。个人觉得近年来数学竞赛有正规化,职业化的倾向。真想做数学
: 家的话,16/7岁这个年纪往竞赛这个方向走得太厉害多少有些浪费的感觉。

s***n
发帖数: 1280
188
你说的第一个情况在美国更普遍。由于教学要求低,很多成绩前列的孩子只是学习用功
有效率的孩子,不是智商最高的孩子。智商外的因素对成绩的影响更大。
至于智力对分数的影响,你的个人观察和统计研究结果并不矛盾,因为研究者考察的成
绩不是竞赛成绩,而是统考类型的考试 - 普通智商的孩子可以学得懂的知识。另外这
里的专注力不是简单的小时数,学习努力程度,更多的是对某一道难题持续思考的能力
。这个其实个人观察是很难看到。
我觉得其实哪怕是在竞赛级的数学蛙中间,他们的成绩也是和专注力的相关性多于智商。

【在 f*******e 的大作中提到】
: 这方面我比较有发言权。我读大学前在一个高考竞争力很强的省,那时全省大约8个学
: 生才有一个能上大学。而我是属于小学成绩不好,被老师说可能考不上初中,后来依靠
: 住重点中学旁边的优势进了重点中学(那时按照学区上初中)。初三开始成绩突飞猛进
: ,高中就在我这最好的中学成了班上第一,高考全省前50。原因是努力和对学习的专注
: 力。说说下面几点:
: 1.高中班上第一,高考全省前50,是否意味我智商高?NO。我觉得我智力方面中等偏上
: ,但我确实非常努力,除了吃饭睡觉和短暂的休息时间都在学习,而且拿班上第一位我
: 个人觉得是靠题海战术。奥赛的题目我做不来。所以像潮水说的,我是普通娃,无论如
: 何推,只能上到一个程度,达不到顶尖水平。
: 2. 智力对于分数是否重要?很重要。我当时班上和我一样努力的不少,甚至比我更努

s***n
发帖数: 1280
189
接着谈。
因为Asperger不必然导致逻辑思维强或数学好,所以当一个Asperger孩子在小时候表现
出“数学天分“,家长需要判断这种天分是数学Gifted,还是hypernumeracy。回到楼主
孩子的例子,我前文已经说了,由于孩子的逻辑思维能力还不明,无法判断他的数学天
分。如果楼主的孩子有ASD/Asperger方面的担心,那对他表现出来的数学“天分",要
有更多一层的考量。Asperger/hypernumeracy的孩子由于对数字的喜爱和记忆力强,他
们中一些孩子在幼儿园和小学低年级可以表现出很强的数学水平。但哪怕是这些孩子中
间也只有小部分是真正的数学Gifted。
对那些表现出数学天分的孩子,区分数学Gifted,hypernumeracy对教育有一定的指导
意义。对于前者,要提高兴趣,拓展数学教育的广度和深度;对后者重点要放在对已学
数学的真正理解上,不能急着上广度深度。
我以前一直也是认同Asperger有利于数理方面的学习。但我现在的观点是要把Asperger
和数理Gifted分开来看和对待。很多Asperger孩子到了初高中阶段数理很差,包括一些
小时候的数学牛蛙;另一方面,数理牛蛙中大多数没有Asperger,只是牛蛙中Asperger
的比例比常人群高。我更倾向于把那些有Asperger的数理牛蛙作为Twice Exceptional
。对这些的Gifted领域的教育,要等同Gifted娃,而不是其他Asperger娃。 这对
Asperger孩子的教育更有指导意义。
待续。

of

【在 s***n 的大作中提到】
: 我同意你说的"GIFTED和special education并不矛盾。gifted kids可能没有
: aspergeror hypernumeracy等, 也可能有。但有asperger等的并不一定是HGT" 教育方
: 面最近出的一个新分类Twice Exceptional就是特指那些又gifted又有learning
: disability (ASD/ADHD, etc)的孩子。我觉得这种描述比认为Asperger的孩子数理好更
: 贴近实际情况,也更有助于对这些孩子的教育。
: 这个班上讨论的Asperger的孩子很多数学不错,所以容易给大家一个假象,以为
: Asperger或hypernumeracy的孩子学习数学有优势。我以前也有这样的感觉。但看了身
: 边和网上的一些例子后,我倾向于认为Asperger或hypernumeracy的孩子在数学方面没
: 有优势,而他们的弱点就是理解能力。2007年的一个相关研究发现,the majority of
: individuals with AS/HFA have average mathematical ability,甚至 have a

s***n
发帖数: 1280
190
接下来讨论前文有人提到过的Asperger孩子"对社交需求低"。
这个观点我不赞成的。Asperger的孩子和其它高功自闭的孩子不一样。Asperger的孩子
不是对社交需求低,而是社交能力低。理解这一点,对教育Asperger的孩子是非常重要
的。
Asperger的孩子其实对社交的需求并不低。某种意义上,他们比高功自闭的孩子更可怜
。高功自闭没有什么社交的需求,可以很快乐的活在自己的世界里,只要外人不来打扰
。而对Asperger的孩子,他们是想和别人社交,却由于社交能力低不能很好的社交。当
他们的社交需求不断碰壁后,他们也许会表现出社交需求低。但这其实是种内心痛苦的
自我保护。如果孩子自己不能调整心理来适应这种状况的话,久而久之,容易引发更严
重的心理问题。康州加州两个多人枪击案的主犯都是这种没有朋友没有社交又无法平衡
的Asperger孩子。
所以对于这些Asperger的孩子,帮助他们社交是很重要的。在他们小时候,培养社交能
力其实是次要的,更重要的提供一个友善宽容允许他们犯社交错误的社交环境。先要满
足他们的社交需求,再慢慢帮助他们提高社交能力。提高社交能力是个漫长的活,家长
不能性急。
另外如果这些孩子有某些领域Gifted,对这些孩子Gifted方面的培养也是很重要的。不
象前面一些网友说的,Asperger的孩子只能把重心放在社交能力培养上面。绝大多数时
候,对Gifted方面的培养也有利于孩子的社交。孩子在Gifted领域的成绩会提高孩子的
自我认同,社会认可,也更容易找到兴趣爱好相同的社交环境。
顺带说一句,"HGT的对社交有需求,所以投入竞赛的时间少一点,但朋友多,活得更快
乐"。这一点我也不赞同。非Twice Exceptional 的HGT孩子的自杀比例不低,只怕要高
过Asperger和Twice Exceptional自杀的。家长不能只看到朋友多有社交,就觉得孩子
很快乐。对初高中的孩子,家长要多和孩子正面交流。

【在 s***n 的大作中提到】
: 接着谈。
: 因为Asperger不必然导致逻辑思维强或数学好,所以当一个Asperger孩子在小时候表现
: 出“数学天分“,家长需要判断这种天分是数学Gifted,还是hypernumeracy。回到楼主
: 孩子的例子,我前文已经说了,由于孩子的逻辑思维能力还不明,无法判断他的数学天
: 分。如果楼主的孩子有ASD/Asperger方面的担心,那对他表现出来的数学“天分",要
: 有更多一层的考量。Asperger/hypernumeracy的孩子由于对数字的喜爱和记忆力强,他
: 们中一些孩子在幼儿园和小学低年级可以表现出很强的数学水平。但哪怕是这些孩子中
: 间也只有小部分是真正的数学Gifted。
: 对那些表现出数学天分的孩子,区分数学Gifted,hypernumeracy对教育有一定的指导
: 意义。对于前者,要提高兴趣,拓展数学教育的广度和深度;对后者重点要放在对已学

相关主题
有些小学四年级的数学题不太会做WISC-V 智商测试
转载: 从首个IMO季军谈起 by 付云皓今天和幼儿园老师开会review娃的progress,非常upset
新加坡数学对小学生帮助大吗?好学生去普通公立高中真的会被耽误吗?
进入Parenting版参与讨论
s***n
发帖数: 1280
191
文学城有把各种奖项分10大等级- 大家对号入座。

【在 f*c 的大作中提到】
: 美国乱七八糟的竞赛太多,不知道哪个真的算数。前些日子说某某没被某藤录取抱怨云
: 云,是不是因为得的这个奖那个奖不是那个啥?我估计USAMO qualifier应该是算数的
: ,这个上大学能“加分”吗?具体怎么个过程?
: 记得以前国际足联某大佬看见中国队经常赢个友谊赛,什么桑普多利亚之类的,就撂下
: 一句狠话:中国队要赢那些必须赢的比赛。嘿嘿嘿,这个好像现在都没做到。。。

f*******e
发帖数: 3433
192
如果HGT里自杀的这么多,那么里面肯定有很大一部分实际上是有ASPERGER的。很多
gifted kids没有被诊断出asperger,因为他们的gift掩盖了本身的一些问题。
asperger的也很可能outgrow。 高智商的变得符合HGT标准,但有没有一些asperger的
残留,也很难说。诊断都是人为的。比如我家小孩,他的医生虽然诊断他为asperger,
却不很确定,因为asperger症状只达到最低标准。所以她说明年还要再EVALUATE一下。
我这几天注意了一下小孩,发现一个暑假过后,他的理解力社交力增长很快。我现在给
他读书他都能回答问题,和别的小孩玩的时候主动打招呼,说话,经常SHOW OFF他的东
西,试图帮助别的小孩。最近以前的一些rigidity症状也消失了,除了有时候想要某个
东西再我说了NO以后还一直要几遍以后才放弃了。在别的小孩不接受他帮助或者不
回打招呼时开始有点沮丧了。所以我现在教他如何和别人沟通,比如不应该看别的小孩
有困难就说我帮你,而应该先问他是否需要帮助,如果他说需要再帮助。我不确定现在
他是outgrow了还是社交方面还有障碍,目前还在观察中。

【在 s***n 的大作中提到】
: 接下来讨论前文有人提到过的Asperger孩子"对社交需求低"。
: 这个观点我不赞成的。Asperger的孩子和其它高功自闭的孩子不一样。Asperger的孩子
: 不是对社交需求低,而是社交能力低。理解这一点,对教育Asperger的孩子是非常重要
: 的。
: Asperger的孩子其实对社交的需求并不低。某种意义上,他们比高功自闭的孩子更可怜
: 。高功自闭没有什么社交的需求,可以很快乐的活在自己的世界里,只要外人不来打扰
: 。而对Asperger的孩子,他们是想和别人社交,却由于社交能力低不能很好的社交。当
: 他们的社交需求不断碰壁后,他们也许会表现出社交需求低。但这其实是种内心痛苦的
: 自我保护。如果孩子自己不能调整心理来适应这种状况的话,久而久之,容易引发更严
: 重的心理问题。康州加州两个多人枪击案的主犯都是这种没有朋友没有社交又无法平衡

s***n
发帖数: 1280
193
HGT自杀的肯定有些是Twice Exceptional,但多大比例很难说。因为一些藤校,特别是
大藤,自杀率甚至高于Asperger娃的自杀率,所以把HGT高自杀率完全归结于Asperger
是说不太通的。另外大藤的招生重全面发展,重leadership,Asperger孩子在大藤中的
比例不会太高。
大家看到报道出来的藤校或高中名校的学生自杀个案,大多数孩子都是看上去很正常阳
光,朋友很多的孩子。我不觉得这些HGT孩子自杀可以归结于Asperger。
我觉得Asperger孩子自杀率高不是Asperger内因直接造成的,而是由于我前文提到
的基本社交需求没得到满足以及社会歧视造成的。如果一个孩子已经outgrow了
Asperger,或者解决了社交需求的问题,这些孩子的自杀率应该和其他孩子差不多。
另外Asperger的误诊很常见。在学龄儿童身上,Asperger和HGT的症状有很多项重合。
一些outgrow的case其实是早期误诊。正常孩子的情商发展是个长期的事,情商的发展
有快慢早晚。另外,我以前也提到了HGT的孩子社交环境更恶劣些。有些HGT的孩子在学
龄前和小学低年级的时候表现出的社交问题其实并不是Asperger造成的。不过本着防微
杜渐的原则,误诊大多有益无害。

,

【在 f*******e 的大作中提到】
: 如果HGT里自杀的这么多,那么里面肯定有很大一部分实际上是有ASPERGER的。很多
: gifted kids没有被诊断出asperger,因为他们的gift掩盖了本身的一些问题。
: asperger的也很可能outgrow。 高智商的变得符合HGT标准,但有没有一些asperger的
: 残留,也很难说。诊断都是人为的。比如我家小孩,他的医生虽然诊断他为asperger,
: 却不很确定,因为asperger症状只达到最低标准。所以她说明年还要再EVALUATE一下。
: 我这几天注意了一下小孩,发现一个暑假过后,他的理解力社交力增长很快。我现在给
: 他读书他都能回答问题,和别的小孩玩的时候主动打招呼,说话,经常SHOW OFF他的东
: 西,试图帮助别的小孩。最近以前的一些rigidity症状也消失了,除了有时候想要某个
: 东西再我说了NO以后还一直要几遍以后才放弃了。在别的小孩不接受他帮助或者不
: 回打招呼时开始有点沮丧了。所以我现在教他如何和别人沟通,比如不应该看别的小孩

k**n
发帖数: 6198
194
板凳
深表赞同

【在 s***n 的大作中提到】
: 接下来讨论前文有人提到过的Asperger孩子"对社交需求低"。
: 这个观点我不赞成的。Asperger的孩子和其它高功自闭的孩子不一样。Asperger的孩子
: 不是对社交需求低,而是社交能力低。理解这一点,对教育Asperger的孩子是非常重要
: 的。
: Asperger的孩子其实对社交的需求并不低。某种意义上,他们比高功自闭的孩子更可怜
: 。高功自闭没有什么社交的需求,可以很快乐的活在自己的世界里,只要外人不来打扰
: 。而对Asperger的孩子,他们是想和别人社交,却由于社交能力低不能很好的社交。当
: 他们的社交需求不断碰壁后,他们也许会表现出社交需求低。但这其实是种内心痛苦的
: 自我保护。如果孩子自己不能调整心理来适应这种状况的话,久而久之,容易引发更严
: 重的心理问题。康州加州两个多人枪击案的主犯都是这种没有朋友没有社交又无法平衡

f*******e
发帖数: 3433
195
我家也很阳光,老师说他在学校里每天都笑嘻嘻的。即使同伴不接受他的帮助他明显有
点沮丧,等同伴走了问他,他还是说他很开心。我也不知道这是好事还是应该担心。不
过不管怎样,重点还是教他如何SOCIAl。有ASPERGER症状的(包括没有去DIAGNOSE的)
在大藤理工科比例并不低,虽然在藤校本身比例不高。我见过很多美国理工科的学生,
不少学习能力很强的能看出轻微ASPERGER症状。有些看上去阳光的小孩自杀是因为被
BULLY,被父母逼得压力太大,或者以前光环太多突然遇了点挫折就不知如何处理,
或者把真正的自己隐藏了。我认得的一个学生的弟弟(华裔)因为学习压力精神出了
问题(前两年GPA4.0,第三年开始不知道怎么就出了问题)。他弟弟我见过,也看
着挺阳光。所以我觉得不管是怎样的小孩,心理教育很重要。

Asperger

【在 s***n 的大作中提到】
: HGT自杀的肯定有些是Twice Exceptional,但多大比例很难说。因为一些藤校,特别是
: 大藤,自杀率甚至高于Asperger娃的自杀率,所以把HGT高自杀率完全归结于Asperger
: 是说不太通的。另外大藤的招生重全面发展,重leadership,Asperger孩子在大藤中的
: 比例不会太高。
: 大家看到报道出来的藤校或高中名校的学生自杀个案,大多数孩子都是看上去很正常阳
: 光,朋友很多的孩子。我不觉得这些HGT孩子自杀可以归结于Asperger。
: 我觉得Asperger孩子自杀率高不是Asperger内因直接造成的,而是由于我前文提到
: 的基本社交需求没得到满足以及社会歧视造成的。如果一个孩子已经outgrow了
: Asperger,或者解决了社交需求的问题,这些孩子的自杀率应该和其他孩子差不多。
: 另外Asperger的误诊很常见。在学龄儿童身上,Asperger和HGT的症状有很多项重合。

t******l
发帖数: 10908
196
我同意为了竞赛坐下来苦鼻刷题的话,没有录取优惠是不太值得。。。所以我觉得在这
种情况下,不要把竞赛当刷题拿成绩抓,而把竞赛当灵活数学学习/(相对而言的)趣
味数学来抓。。。这样虽然无法保证竞赛成绩高或者入围,但是能保证学习兴趣、学习
效率、活学活用、急用先学、立杆见影,从而保证长期的学习方面的投资回报率,已经
保证不会被炮灰而血本无归。

:听上去大学录取形势严峻啊。俺那个年代国内进了TOP500基本上就保送了,单数学大
省能有四五十人吧。竞赛这个东西没有录取优惠是不太值得。AMC算是够基础了,但它那
:个时间限制也是需要一定的题海磨练的。
t******l
发帖数: 10908
197
我觉得跟哪怕跟竞赛级最终成绩直接相关的“专注力”,也是指那种坐下来解题的需要
attention span 的专注力。。。而我觉得普通娃长大以后学习解难题,不是靠专注力
,而是靠注意力的分配力和切换力。
对于你说的 “学习努力程度,更多的是对某一道难题持续思考的能力。这个其实个人观
察是很难看到。”,这是事实。。。但对普通娃而言,由于 attention span 的限制,
这个 “持续思考能力”,并不是通过 “专注力” 来解决。。。而是在娃长大以后
,反其道而行之,通过 “一边xx一边yy“(分配/切换注意力)和 mental work (心
算力),来解决的。
其实大人也是一样,比如我最近为了教娃要自己做题,昨天我就在聚众吃饭的时候,因
为吃饭前跟人聊天的时候正好看了一眼 2015 AIME I, Problem #3。。。在吃饭当中侃
大山当中就抽了十几分钟心算一把。第一次的心算解法不是最佳,但也解出来了。第一
次是这么心算的:
(n+1)^3 - 1
= n^3 + 3*(n^2) + 3*n
= n*(n^2 + 3*n + 3)
因为是 prime,所以取 n=16,心算:
16*16 + 3*16 + 3
= 256 + 48 + 3
= 256 + 51
= 307
答案是 307。


:你说的第一个情况在美国更普遍。由于教学要求低,很多成绩前列的孩子只是学习用
功有效率的孩子,不是智商最高的孩子。智商外的因素对成绩的影响更大。
t******l
发帖数: 10908
198
而我回家以后闲着吃东西的时候,心算发现更好的解法:
n^3 - 1
= (n-1) * (n^2 + n + 1)
因为是 prime,取 n = 17,得:
17*17 + 17 + 1
= 17*18 + 1
但到这里出来个心算方面的问题,17*18 不是马工特殊数,心算太烦,所以我就掏手机
按了下计算器验证下结果。。。于是干正事去,然后在给小娃刷完牙以后突然想到这
17*18 还是可以心算的,这么继续算:
17*18 + 1
= 17*9*2 + 1
= (170 - 17)*2 + 1
= 153*2 + 1
= 306 + 1
= 307
但这搞得代数上更好的办法,心算数字反而没遇上马工特殊数就慢了。

:我觉得跟哪怕跟竞赛级最终成绩直接相关的“专注力”,也是指那种坐下来解题的需
要 attention span 的专注力。。。而我觉得普通娃长大以后学习解难题,不是靠专注
力,而是靠注意力的分配力和切换力。
t******l
发帖数: 10908
199
但这样的普通娃学习办法虽然能避免 attention span 不够,同时也人生比较放松不紧
张。。。但这同时也导致普通娃在正式比赛的时候,很难跟 hypernumeracy /
Aspergers 竞争,因为正式比赛是坐下来全神贯注的,那普通娃平时的 “一边一边”
学习法,到正式比赛也要打一个很大的折扣的。
所以这也证实了前面 fac 的帖子,在没有入学优惠的情况下,普通娃做为竞赛备考目
的刷题战略,是得不偿失的。。。因为这太违背普通娃 “一边一边” 难题学习法的天
性了。。。普通娃的难题学习方式,一般还是切成能接受的时间小段,利用注意力的分
配力和切换力,来避免普通娃专注力不够(相对 hypernumeracy / Aspergers 而言)
的问题。

:而我回家以后闲着吃东西的时候,心算发现更好的解法:
f*c
发帖数: 687
200
上网看了一圈,感觉老兄可能过于悲观。过了AIME这一关对于UCLA这个层次的学校,甚
至CALTECH都是有点作用的。

它那

【在 t******l 的大作中提到】
: 我同意为了竞赛坐下来苦鼻刷题的话,没有录取优惠是不太值得。。。所以我觉得在这
: 种情况下,不要把竞赛当刷题拿成绩抓,而把竞赛当灵活数学学习/(相对而言的)趣
: 味数学来抓。。。这样虽然无法保证竞赛成绩高或者入围,但是能保证学习兴趣、学习
: 效率、活学活用、急用先学、立杆见影,从而保证长期的学习方面的投资回报率,已经
: 保证不会被炮灰而血本无归。
:
: :听上去大学录取形势严峻啊。俺那个年代国内进了TOP500基本上就保送了,单数学大
: 省能有四五十人吧。竞赛这个东西没有录取优惠是不太值得。AMC算是够基础了,但它那
: :个时间限制也是需要一定的题海磨练的。

相关主题
陶天才论天才 (转载)9岁, 如何报名考amc10
Explaining Your Math: Unnecessary at Best, Encumbering at W有明天考AMC 8的吗?
是不是任我儿子继续迷象棋[转载] 为什么说usamo 简单
进入Parenting版参与讨论
l*****c
发帖数: 1153
201
如果都是自己推导的,挺像我幼儿园的我,我当时也是自己推导出了乘除法,非十进制
,小数,负数这些。我觉得是有天赋的,可以引导一把,不要去培养。告诉他去学什么
想什么就行,用问题去诱惑他,让他自己去研究解决方案,不要去教。

【在 c********5 的大作中提到】
: 娃6岁了,一直都是普通娃,有些方面和同龄人比,还delay一年甚至更多。最近回国,
: 大家都说他数学有天分,可以多培养培养。当妈的我第一次有了飘飘然滴赶脚。上来一
: 是想问问,是不是大家忽悠我呢。二是想问问,下一步数学该如何推(无论是不是有天
: 赋)。真心讨论娃的问题,不喜欢,请右上角点X退出。万分谢谢。
: 1.我原本想给娃讲讲高斯定理,娃一看到我出的题,马上的反应是:
: 1+2+3+4+5+6+7+8+9+10
: =10+1+9+2+8+3+7+4+6+5
: =55
: (这个学校里倒是常常要求凑10,也可能有人问过他这道题)
: 2.

n******e
发帖数: 1046
202
应该还是有点数学天分,我儿子不到8岁
;,这些题也都会了,我儿子也是有点数学天
分的,对数字很敏感也很喜欢数学。
至于8*8 他会直接64出答案,因为乘法表他读K班5岁就hui
了。
高斯题,他已经会譬如5+。。。+25难度高一点的了
,自己会找多少个数,偶数还是奇数
个,中间值是多少。
当然他已经远远学到前面了,有点拔苗,不
guo这些要看孩子的个体差异,是否能吸收。

【在 c********5 的大作中提到】
: 娃6岁了,一直都是普通娃,有些方面和同龄人比,还delay一年甚至更多。最近回国,
: 大家都说他数学有天分,可以多培养培养。当妈的我第一次有了飘飘然滴赶脚。上来一
: 是想问问,是不是大家忽悠我呢。二是想问问,下一步数学该如何推(无论是不是有天
: 赋)。真心讨论娃的问题,不喜欢,请右上角点X退出。万分谢谢。
: 1.我原本想给娃讲讲高斯定理,娃一看到我出的题,马上的反应是:
: 1+2+3+4+5+6+7+8+9+10
: =10+1+9+2+8+3+7+4+6+5
: =55
: (这个学校里倒是常常要求凑10,也可能有人问过他这道题)
: 2.

f*******e
发帖数: 3433
203
那你长大后数学一直名列前茅吗?数学奥赛拿奖了吗?问这些就是想知道那些小时候是
“天才”的长大后怎样了。

【在 l*****c 的大作中提到】
: 如果都是自己推导的,挺像我幼儿园的我,我当时也是自己推导出了乘除法,非十进制
: ,小数,负数这些。我觉得是有天赋的,可以引导一把,不要去培养。告诉他去学什么
: 想什么就行,用问题去诱惑他,让他自己去研究解决方案,不要去教。

f*******e
发帖数: 3433
204
小孩子如果要拔苗的话除了太笨的都能拔上去。但小学拔苗拔高的大了不一定行。
因为小时候只要靠记忆,简单理解,加一点技巧就够了。大了容易成强弩
之末。这就是我为什么反对从小拔苗的原因。见过太多小时候拔苗拔出的
天才到高中哗哗掉的例子

【在 n******e 的大作中提到】
: 应该还是有点数学天分,我儿子不到8岁
: ;,这些题也都会了,我儿子也是有点数学天
: 分的,对数字很敏感也很喜欢数学。
: 至于8*8 他会直接64出答案,因为乘法表他读K班5岁就hui
: 了。
: 高斯题,他已经会譬如5+。。。+25难度高一点的了
: ,自己会找多少个数,偶数还是奇数
: 个,中间值是多少。
: 当然他已经远远学到前面了,有点拔苗,不
: guo这些要看孩子的个体差异,是否能吸收。

s***n
发帖数: 1280
205
一边xx一边yy分心做事绝大多数时候导致低效,不论是学习还是考试。这也是为什么很
多专注力不好的高智商孩子学习效率低成绩差,而专注力好的普通智商孩子可以成绩更
好。这种例子身边比比皆是。极端点的例子就是那些高智商ADHD的孩子。他们
Attention span太短,直接导致学习低效,你再怎么会转换分配也白搭。
专注力好不是ASD/Asperger/hypernumeracy特有的。大多数专注力好的孩子并没有ASD/
Asperger/hypernumeracy的问题。你不能因为身边一些普通孩子专注力不好,就认为专
注力好的是病态。对于那些专注力不好的孩子,解决专注力问题的首选办法应该是提高
专注力。我前文提到的兴趣,就是提高专注力的方法之一。如果孩子实在专注力太差,
也可以考虑检查ADHD以及相应的治疗措施。实在孩子专注力无法提高了,才应该把重点
放在适应和改进专注力差导致的种种低效上。你提到的那些方法属于最后一种,不是不
行,但不是最佳。

人观

【在 t******l 的大作中提到】
: 我觉得跟哪怕跟竞赛级最终成绩直接相关的“专注力”,也是指那种坐下来解题的需要
: attention span 的专注力。。。而我觉得普通娃长大以后学习解难题,不是靠专注力
: ,而是靠注意力的分配力和切换力。
: 对于你说的 “学习努力程度,更多的是对某一道难题持续思考的能力。这个其实个人观
: 察是很难看到。”,这是事实。。。但对普通娃而言,由于 attention span 的限制,
: 这个 “持续思考能力”,并不是通过 “专注力” 来解决。。。而是在娃长大以后
: ,反其道而行之,通过 “一边xx一边yy“(分配/切换注意力)和 mental work (心
: 算力),来解决的。
: 其实大人也是一样,比如我最近为了教娃要自己做题,昨天我就在聚众吃饭的时候,因
: 为吃饭前跟人聊天的时候正好看了一眼 2015 AIME I, Problem #3。。。在吃饭当中侃

o***n
发帖数: 2074
206
这个和后天兴趣,训练有关, 有个判断数学天才的简单标准: 2周岁以前能分清3个和
五个的区别, 越多越好, 那就一定有数学天分。
正常的孩子,2岁前只能分清一个, 两个, 和很多,对3以上的数没有概念。

【在 c********5 的大作中提到】
: 娃6岁了,一直都是普通娃,有些方面和同龄人比,还delay一年甚至更多。最近回国,
: 大家都说他数学有天分,可以多培养培养。当妈的我第一次有了飘飘然滴赶脚。上来一
: 是想问问,是不是大家忽悠我呢。二是想问问,下一步数学该如何推(无论是不是有天
: 赋)。真心讨论娃的问题,不喜欢,请右上角点X退出。万分谢谢。
: 1.我原本想给娃讲讲高斯定理,娃一看到我出的题,马上的反应是:
: 1+2+3+4+5+6+7+8+9+10
: =10+1+9+2+8+3+7+4+6+5
: =55
: (这个学校里倒是常常要求凑10,也可能有人问过他这道题)
: 2.

G*******s
发帖数: 10605
207
Ask your kid to play game of 24. Then try some hard word problem (like A
uses 3 hours to finish a distance, B uses 6 hours. What if they walk toward
each other to finish the distance. I think he should have never learnt
fraction, if he can figure this by himself then he is gifted.)
s***n
发帖数: 1280
208
"天才"哗哗掉的例子我也看到不少。很多是因为拔的那根苗就不是天才或是拔法不对,
归根是因为教育者对苗认识不足。对于真正的天才,脱离正规教育步骤的适当"拔"苗是
有利的。所以要不要拔苗,是因人因时而异的,不能一概而论。
我前面提到过,中学数学教育是抽象思维加逻辑思维。逻辑思维的成熟要到初高中。很
多小学数学天才只是抽象思维好(更小的数学天才可能只是记忆力好)。很多人缺省的认
为"天才"孩子什么都好,没有意识到很多天才只是某一两项能力有极高天分。不少抽象
思维好的孩子逻辑思维不一定好,这些孩子学到高中数理,就会吃力。而另一些孩子可
能抽象思维一般,但逻辑思维强,这些孩子的成绩到高中会上来。所以不少"天才"哗哗
掉的例子其实是正常人事变动,和推娃关系不大。

【在 f*******e 的大作中提到】
: 小孩子如果要拔苗的话除了太笨的都能拔上去。但小学拔苗拔高的大了不一定行。
: 因为小时候只要靠记忆,简单理解,加一点技巧就够了。大了容易成强弩
: 之末。这就是我为什么反对从小拔苗的原因。见过太多小时候拔苗拔出的
: 天才到高中哗哗掉的例子

f*******e
发帖数: 3433
209
所以我说的是“不一定行”,而不是“一定不行”。我只是想说明小时候小孩通过父母
提前教懂这些东西并不能说明小孩有数学天赋。长大后才能看出是不是天才。而且
国内那套小学教学方法很多是靠记忆力,比如乘法本身还没理解透就背乘法表,
加法教竖式加法却不教原理之内的。容易让小孩学了表却不知里。我小时候就
这样,算加法套竖式,为什么?不知道。

【在 s***n 的大作中提到】
: "天才"哗哗掉的例子我也看到不少。很多是因为拔的那根苗就不是天才或是拔法不对,
: 归根是因为教育者对苗认识不足。对于真正的天才,脱离正规教育步骤的适当"拔"苗是
: 有利的。所以要不要拔苗,是因人因时而异的,不能一概而论。
: 我前面提到过,中学数学教育是抽象思维加逻辑思维。逻辑思维的成熟要到初高中。很
: 多小学数学天才只是抽象思维好(更小的数学天才可能只是记忆力好)。很多人缺省的认
: 为"天才"孩子什么都好,没有意识到很多天才只是某一两项能力有极高天分。不少抽象
: 思维好的孩子逻辑思维不一定好,这些孩子学到高中数理,就会吃力。而另一些孩子可
: 能抽象思维一般,但逻辑思维强,这些孩子的成绩到高中会上来。所以不少"天才"哗哗
: 掉的例子其实是正常人事变动,和推娃关系不大。

f*******e
发帖数: 3433
210
拔苗不等于推娃。推娃有正确推法和错误推法。而“拔苗助长”这个词是指错误推法。读
初中高中的娃可以推猛点,5,6岁小娃推得过猛就变拔苗助长了。

【在 s***n 的大作中提到】
: "天才"哗哗掉的例子我也看到不少。很多是因为拔的那根苗就不是天才或是拔法不对,
: 归根是因为教育者对苗认识不足。对于真正的天才,脱离正规教育步骤的适当"拔"苗是
: 有利的。所以要不要拔苗,是因人因时而异的,不能一概而论。
: 我前面提到过,中学数学教育是抽象思维加逻辑思维。逻辑思维的成熟要到初高中。很
: 多小学数学天才只是抽象思维好(更小的数学天才可能只是记忆力好)。很多人缺省的认
: 为"天才"孩子什么都好,没有意识到很多天才只是某一两项能力有极高天分。不少抽象
: 思维好的孩子逻辑思维不一定好,这些孩子学到高中数理,就会吃力。而另一些孩子可
: 能抽象思维一般,但逻辑思维强,这些孩子的成绩到高中会上来。所以不少"天才"哗哗
: 掉的例子其实是正常人事变动,和推娃关系不大。

相关主题
[转载] 为什么说usamo 简单我招,我是猪
二年级女儿的report大学录取率
觉得美帝数学进度慢的进来看一下学校早晚要教的东西,早早学会了又如何?
进入Parenting版参与讨论
s***n
发帖数: 1280
211
"小孩通过父母提前教懂这些东西并不能说明小孩有数学天赋" 这一点我很赞同。我也
一直在跟LZ说,她孩子还小,现在还看不出孩子是不是真正有天赋。你对国内数学教育
方法的批评我也很同意。
有异议的地方是我觉得对小孩数学天赋还是要尽早判断尽早提供帮助,不要等到"长大
后",因为美国中小学数学教育的长处是拖数学牛娃后腿来减少Achievement Gap。

【在 f*******e 的大作中提到】
: 所以我说的是“不一定行”,而不是“一定不行”。我只是想说明小时候小孩通过父母
: 提前教懂这些东西并不能说明小孩有数学天赋。长大后才能看出是不是天才。而且
: 国内那套小学教学方法很多是靠记忆力,比如乘法本身还没理解透就背乘法表,
: 加法教竖式加法却不教原理之内的。容易让小孩学了表却不知里。我小时候就
: 这样,算加法套竖式,为什么?不知道。

n******e
发帖数: 1046
212
我不会去想那么多,他喜欢也有这个接受能力想学什
么都可以,那些学才艺的跟学数学没有什么不
同吧。 只是大多数的娃没有这个接受能力而
已。哪个学才艺的不是老师从小教出来的?
我儿子学其他的要没有兴趣了,他说他不要学了,
我马上停了,我不勉强,但是他喜欢的东西干吗不
jian持呢?我才不相信什?强弩之末,我早就研究了,那些数学
拿奖的10个9个都是提前学的,还有个天才吧。
怎么这么多乱码?

【在 f*******e 的大作中提到】
: 小孩子如果要拔苗的话除了太笨的都能拔上去。但小学拔苗拔高的大了不一定行。
: 因为小时候只要靠记忆,简单理解,加一点技巧就够了。大了容易成强弩
: 之末。这就是我为什么反对从小拔苗的原因。见过太多小时候拔苗拔出的
: 天才到高中哗哗掉的例子

t******l
发帖数: 10908
213
我觉得这 “专注力” 也要大概量化。。。如果是不严格目测的话,如果说人群平均的
专注力是 5,那我家小娃以前所在幼儿园的大部分娃的专注力可能是 8,我家小娃幼儿
园时老师说她专注力还不错,我估计可能是 9。。。但相比而言,楼主娃能玩一枯燥计
算器玩上三个小时,那个专注力没有 1000 也有 800,跟普通娃不是一个概念的专注力。

:一边xx一边yy分心做事绝大多数时候导致低效,不论是学习还是考试。这也是为什么
很多专注力不好的高智商孩子学习效率低成绩差,而专注力好的普通智商孩子可以成绩
更好。这种例子身边比比皆是。极端点的例子就是那些高智商ADHD的孩子。他们
s***n
发帖数: 1280
214
赞同。我说的"拔"是说提前学和推,不特指错误推法。所以我觉得我们要表达的意思其
实不矛盾。

。读

【在 f*******e 的大作中提到】
: 拔苗不等于推娃。推娃有正确推法和错误推法。而“拔苗助长”这个词是指错误推法。读
: 初中高中的娃可以推猛点,5,6岁小娃推得过猛就变拔苗助长了。

f*******e
发帖数: 3433
215
我们说的不矛盾啊。我是同意推的,但反对5,6岁小娃推得太猛(就是拔苗)。
而且应该有针对性的推。美国小学教育确实不适合牛蛙,所以我每天有时间的话
教他7-8分钟数学,根据美国教钢教(之前用中国教法教,结果上K后和
common-core教法冲突,娃很不适应。比如这边用number bond教加法,
小孩习惯了1+1=2的表示方法后很不适应)。所以我加法就教他number bond,
make 10, skip counting, 竖式加法,并且在每教一个的时候都会解释为为什么
这么做。同时教他不需要靠背来解决加减法问题。现在小孩如鱼得水,既可以
适应学校,又比学校教得简洁,而且因为推的比较GENTLE,小孩没感觉到
任何压力。今年还跑过来说要我教他乘法因为他summer camp的老师提了
一下乘法。然后就教了几分钟,学得津津有味。

【在 s***n 的大作中提到】
: "小孩通过父母提前教懂这些东西并不能说明小孩有数学天赋" 这一点我很赞同。我也
: 一直在跟LZ说,她孩子还小,现在还看不出孩子是不是真正有天赋。你对国内数学教育
: 方法的批评我也很同意。
: 有异议的地方是我觉得对小孩数学天赋还是要尽早判断尽早提供帮助,不要等到"长大
: 后",因为美国中小学数学教育的长处是拖数学牛娃后腿来减少Achievement Gap。

t******l
发帖数: 10908
216
另外 “一边xx一边yy” 的目的不是为了提高效率,而是为了在专注力不够的情况下提
高为了练习目的的有效的持久力。。。这就好比短跑队的分组训练/循环训练法。。。
短跑队都是一批没耐力的娃。。。分组训练法就是每个 100 米只上 80% 的力气,这样
休息恢复快,休息恢复后再来一组。。。循环训练法就是普通跑、后蹬跑、高抬腿跑、
俯卧撑、叉腰肌等等各种循环,让不同肌群得到间歇休息,以对付肌肉耐力不够的问题
。。。古人云:HIIT -- High Intensity Interval Training。。。

:一边xx一边yy分心做事绝大多数时候导致低效,不论是学习还是考试。这也是为什么
很多专注力不好的高智商孩子学习效率低成绩差,而专注力好的普通智商孩子可以成绩
更好。这种例子身边比比皆是。极端点的例子就是那些高智商ADHD的孩子。他们
f*******e
发帖数: 3433
217
我们实际上很多观点是一样的,表述上的不同:-)

【在 s***n 的大作中提到】
: 赞同。我说的"拔"是说提前学和推,不特指错误推法。所以我觉得我们要表达的意思其
: 实不矛盾。
:
: 。读

s***n
发帖数: 1280
218
那些研究教育,心理,自闭多动的应该已经把专注力量化了吧。所以才有结论,HGT的
专注力接近AS和ASD。
如果LZ孩子是几年如一日的每天玩三个小时计算器,或三个小时只玩计算器一个键,那
可以考虑他的专注力可能超出正常区间。如果只是一两次,两三次,我不觉得有啥问题
。很多小孩一打游戏几个小时。无它,兴趣而已。

力。

【在 t******l 的大作中提到】
: 我觉得这 “专注力” 也要大概量化。。。如果是不严格目测的话,如果说人群平均的
: 专注力是 5,那我家小娃以前所在幼儿园的大部分娃的专注力可能是 8,我家小娃幼儿
: 园时老师说她专注力还不错,我估计可能是 9。。。但相比而言,楼主娃能玩一枯燥计
: 算器玩上三个小时,那个专注力没有 1000 也有 800,跟普通娃不是一个概念的专注力。
:
: :一边xx一边yy分心做事绝大多数时候导致低效,不论是学习还是考试。这也是为什么
: 很多专注力不好的高智商孩子学习效率低成绩差,而专注力好的普通智商孩子可以成绩
: 更好。这种例子身边比比皆是。极端点的例子就是那些高智商ADHD的孩子。他们

k**n
发帖数: 6198
219
你怎么找的美国教纲?我在我娃学校看了,除了教学目标,没看到他们乌七八糟的教法阿
惭愧的问,数学怎么教?俺的直觉是,数学没法教,就是拿着题讲一遍。讲完了,懂了
就懂了,没懂就没懂。每个题型都做过,那就八九不离十。 普通娃,不就是题海战吗?

【在 f*******e 的大作中提到】
: 我们说的不矛盾啊。我是同意推的,但反对5,6岁小娃推得太猛(就是拔苗)。
: 而且应该有针对性的推。美国小学教育确实不适合牛蛙,所以我每天有时间的话
: 教他7-8分钟数学,根据美国教钢教(之前用中国教法教,结果上K后和
: common-core教法冲突,娃很不适应。比如这边用number bond教加法,
: 小孩习惯了1+1=2的表示方法后很不适应)。所以我加法就教他number bond,
: make 10, skip counting, 竖式加法,并且在每教一个的时候都会解释为为什么
: 这么做。同时教他不需要靠背来解决加减法问题。现在小孩如鱼得水,既可以
: 适应学校,又比学校教得简洁,而且因为推的比较GENTLE,小孩没感觉到
: 任何压力。今年还跑过来说要我教他乘法因为他summer camp的老师提了
: 一下乘法。然后就教了几分钟,学得津津有味。

t******l
发帖数: 10908
220
我实话实说觉得心理学还不如体育学。。。心理学都不区分 “专注力” vs “持久注
意力”,体育系上把 sprinter 和 endurance runner 分开讨论的。
另外心理学是理想环境,但实践上马工上班码 code 的专注力肯定比做 AIME 强太多,
因为来支票。。。而如果不是要教娃,估计根本就不会去做 AIME 题,专注力降到零。
另外的问题我觉得 HGT 可能也有 selection bias,或者说 HGT 其实也不是普通正常
范围的娃。

:那些研究教育,心理,自闭多动的应该已经把专注力量化了吧。所以才有结论,HGT的
相关主题
amc 10 的A, B卷有啥区别转载: 从首个IMO季军谈起 by 付云皓
来点美好的时光新加坡数学对小学生帮助大吗?
有些小学四年级的数学题不太会做WISC-V 智商测试
进入Parenting版参与讨论
t******l
发帖数: 10908
221
或者说,如果不是 hypernumeracy,同时也没有职业需求,那普通正常范围的大众,对
数学题的专注力通常很低。。。心理学研究很难创建 County 烧烤大会的真实氛围,除
非上大卫科波菲尔。。。

:我实话实说觉得心理学还不如体育学。。。心理学都不区分 “专注力” vs “持久注
:意力”,体育系上把 sprinter 和 endurance runner 分开讨论的。
f*******e
发帖数: 3433
222
我们这里是根据一个网站给的common core大纲教数学,上面给得非常详细,很多例子
告诉怎么教。不过不同学校不同州教纲不同,如果你小孩不用number bond学加减,那教
纲肯定不同. number bond好像是新加坡数学教法。所以建议你问问学校老师。
我看了教纲后,发现小学1,2年级都在学加减法(make 10, skip counting, 三位数加
减)。3年级才学乘除,但common core3年级考试已经很多乘除,和小数的题了,所以
提前一年学是必要的。我家5岁半,现在还在教他2年级的加减(除了今天他要我教的三
年级简单乘法入门)。
自从娃上K了,我一直在考虑如何结合中美教学教小小孩数学,既有美国的理解,有能
快速计算,而且还不和娃学校学的冲突。现在至少知道怎么教加法,减法和乘法了。
除法还没研究。

法阿
吗?

【在 k**n 的大作中提到】
: 你怎么找的美国教纲?我在我娃学校看了,除了教学目标,没看到他们乌七八糟的教法阿
: 惭愧的问,数学怎么教?俺的直觉是,数学没法教,就是拿着题讲一遍。讲完了,懂了
: 就懂了,没懂就没懂。每个题型都做过,那就八九不离十。 普通娃,不就是题海战吗?

n******e
发帖数: 1046
223
你这不也是在拔么就不怕强弩之末啦? 愿学愿教,就那?简
;单,哪有那么多大道理。

那教

【在 f*******e 的大作中提到】
: 我们这里是根据一个网站给的common core大纲教数学,上面给得非常详细,很多例子
: 告诉怎么教。不过不同学校不同州教纲不同,如果你小孩不用number bond学加减,那教
: 纲肯定不同. number bond好像是新加坡数学教法。所以建议你问问学校老师。
: 我看了教纲后,发现小学1,2年级都在学加减法(make 10, skip counting, 三位数加
: 减)。3年级才学乘除,但common core3年级考试已经很多乘除,和小数的题了,所以
: 提前一年学是必要的。我家5岁半,现在还在教他2年级的加减(除了今天他要我教的三
: 年级简单乘法入门)。
: 自从娃上K了,我一直在考虑如何结合中美教学教小小孩数学,既有美国的理解,有能
: 快速计算,而且还不和娃学校学的冲突。现在至少知道怎么教加法,减法和乘法了。
: 除法还没研究。

f*******e
发帖数: 3433
224
我这哪是拔苗啊,只是每天教他最多6-7分钟而已,属于很gentle的推。我前面说过了
,我理解的拔苗是猛推。我又没叫他幼儿园背99乘法表,做乘法。他K都学完了我还在
教他加减法呢。每天最多教6-7分钟能拔到哪去。而且我都是等他前面的东西学扎实了
才往下教,所以我一直没教他乘法(直到今天他跑过来叫我教)。

【在 n******e 的大作中提到】
: 你这不也是在拔么就不怕强弩之末啦? 愿学愿教,就那?简
: ;单,哪有那么多大道理。
:
: 那教

t******l
发帖数: 10908
225
计算器,平板电脑 chess,娃版电脑游戏,这三个对专注力本来就不能相提并论。。。
或者说在这三者之间的相对专注力的大小,本身可能正常范围 vs 超出正常范围的指标
之一。


:那些研究教育,心理,自闭多动的应该已经把专注力量化了吧。所以才有结论,HGT的
t******l
发帖数: 10908
226
这个没有统一标准。。。首先各家娃都不太一样,其次 “成王败寇” 还是 “旱涝保
收” 也是各家自己的选择。

:我这哪是拔苗啊,只是每天教他最多6-7分钟而已,属于很gentle的推。我前面说过了
:,我理解的拔苗是猛推。我又没叫他幼儿园背99乘法表,做乘法。他K都学完了我还
n******e
发帖数: 1046
227
我的理解,父母额外提前教都是拔,否则学校的老师教的也
足够了。推,只是督促做份内的事情。

【在 f*******e 的大作中提到】
: 我这哪是拔苗啊,只是每天教他最多6-7分钟而已,属于很gentle的推。我前面说过了
: ,我理解的拔苗是猛推。我又没叫他幼儿园背99乘法表,做乘法。他K都学完了我还在
: 教他加减法呢。每天最多教6-7分钟能拔到哪去。而且我都是等他前面的东西学扎实了
: 才往下教,所以我一直没教他乘法(直到今天他跑过来叫我教)。

t******l
发帖数: 10908
228
这种术语上都叫 enrichments 而不是 requirements,也就是都不是必须的。。。至于
是早点 enrich 还是晚点 enrich,很多时候偶然因子也很大。。。不过说白了绝大多
数普通
娃最终 enrich 到 AIME 第 8 题基本都洗手不干了,所以早点 enrich 就后面轻松点
,晚点 enrich 就后面推狠点。。。最终结果区别不大,前浪后浪最后都一样,统统都
被拍 S 在 AIME 第 8 题的沙滩上。。。

:我的理解,父母额外提前教都是拔,否则学校的老师教的
也足够了。
k**n
发帖数: 6198
229
我们好后进,感觉我们一年级的老师啥都不教。每天在脸书上就贴照片玩来着。
number bond这词今天才听说。看了一下,和国内一年级的数学书一致。看了一下,觉
得小娃不会这样也没啥关系,俺是不是理论完全错了?
我也不知道怎么教,就给她加减法的竖式,横式,方块,让她自己写。 然后俺在她对
面看自己的书。 她要问,几加几等于几。我也不知道怎么教,直觉是把数拆开来是不
对的,只好直接把答案告诉她。
为什么提前一年学有必要?

那教

【在 f*******e 的大作中提到】
: 我们这里是根据一个网站给的common core大纲教数学,上面给得非常详细,很多例子
: 告诉怎么教。不过不同学校不同州教纲不同,如果你小孩不用number bond学加减,那教
: 纲肯定不同. number bond好像是新加坡数学教法。所以建议你问问学校老师。
: 我看了教纲后,发现小学1,2年级都在学加减法(make 10, skip counting, 三位数加
: 减)。3年级才学乘除,但common core3年级考试已经很多乘除,和小数的题了,所以
: 提前一年学是必要的。我家5岁半,现在还在教他2年级的加减(除了今天他要我教的三
: 年级简单乘法入门)。
: 自从娃上K了,我一直在考虑如何结合中美教学教小小孩数学,既有美国的理解,有能
: 快速计算,而且还不和娃学校学的冲突。现在至少知道怎么教加法,减法和乘法了。
: 除法还没研究。

f*******e
发帖数: 3433
230
那就是理解上的不同了。

【在 n******e 的大作中提到】
: 我的理解,父母额外提前教都是拔,否则学校的老师教的也
: 足够了。推,只是督促做份内的事情。

相关主题
今天和幼儿园老师开会review娃的progress,非常upsetExplaining Your Math: Unnecessary at Best, Encumbering at W
好学生去普通公立高中真的会被耽误吗?是不是任我儿子继续迷象棋
陶天才论天才 (转载)9岁, 如何报名考amc10
进入Parenting版参与讨论
f*******e
发帖数: 3433
231
我们老师每个月也会给我们发每个月要交的,所以我们做家长的比较清楚小孩学了什么
。当时学number bond时小孩病了,school work就带回家做。我看得一头雾水,问老师
这个图是什么东东。老师说是number bond, 还说我如果不清楚她可以和我meeting.
然后我就google了number bond, 又看了看教纲,发现整个小学数学都是以这个number
bond为基础,才改变教学策略根据教纲教。开始我对这个common core颇有微词,不过
在教的过程中发现里面的很多东西实际上是加减法的技巧(毕竟是根据新加坡数学制定
的),里面光加减发就有好几个不同的做法,能让小娃在理解的基础上快速解题,但学
校教的速度又慢得像蜗牛,所以我在教他的时候就扬长避短了。
提前一年学乘法有必要时因为这边common core考试很难,每次考试整个州不到一半学
生能达到标准,所以教的东西在3年级突然增多增难,很多学生适应不了。提前一年教
小孩3年级就比较容易适应了。

【在 k**n 的大作中提到】
: 我们好后进,感觉我们一年级的老师啥都不教。每天在脸书上就贴照片玩来着。
: number bond这词今天才听说。看了一下,和国内一年级的数学书一致。看了一下,觉
: 得小娃不会这样也没啥关系,俺是不是理论完全错了?
: 我也不知道怎么教,就给她加减法的竖式,横式,方块,让她自己写。 然后俺在她对
: 面看自己的书。 她要问,几加几等于几。我也不知道怎么教,直觉是把数拆开来是不
: 对的,只好直接把答案告诉她。
: 为什么提前一年学有必要?
:
: 那教

P******e
发帖数: 1325
232
哈哈,照你这个标准,我小娃是数学天才,她23个月就能数9个小球,2周岁能数20个了。
请问你这个标准的来源是?您老一拍脑门想的?

【在 o***n 的大作中提到】
: 这个和后天兴趣,训练有关, 有个判断数学天才的简单标准: 2周岁以前能分清3个和
: 五个的区别, 越多越好, 那就一定有数学天分。
: 正常的孩子,2岁前只能分清一个, 两个, 和很多,对3以上的数没有概念。

f*******e
发帖数: 3433
233
简单的说个教法吧:
9+5:
-典型K解法:画9个圈再画5个圈,数一共多少
-改进一点解法:画5个圈,从9开始往上数:10,11,12,13,14
-快一点的解法: 脑子里,从9往上数: 10, 11, 12, 13, 14
-make 10 (快速解法,需要背哪两个单词maket 10): 9+5 = 9 + 1 + 4 = 14
- 纯记忆背法:9+5=14,

【在 k**n 的大作中提到】
: 我们好后进,感觉我们一年级的老师啥都不教。每天在脸书上就贴照片玩来着。
: number bond这词今天才听说。看了一下,和国内一年级的数学书一致。看了一下,觉
: 得小娃不会这样也没啥关系,俺是不是理论完全错了?
: 我也不知道怎么教,就给她加减法的竖式,横式,方块,让她自己写。 然后俺在她对
: 面看自己的书。 她要问,几加几等于几。我也不知道怎么教,直觉是把数拆开来是不
: 对的,只好直接把答案告诉她。
: 为什么提前一年学有必要?
:
: 那教

t******l
发帖数: 10908
s**********y
发帖数: 509
235
赞多样性

【在 f*******e 的大作中提到】
: 简单的说个教法吧:
: 9+5:
: -典型K解法:画9个圈再画5个圈,数一共多少
: -改进一点解法:画5个圈,从9开始往上数:10,11,12,13,14
: -快一点的解法: 脑子里,从9往上数: 10, 11, 12, 13, 14
: -make 10 (快速解法,需要背哪两个单词maket 10): 9+5 = 9 + 1 + 4 = 14
: - 纯记忆背法:9+5=14,

s**********y
发帖数: 509
236
好的数学从不备九九表开始。

【在 n******e 的大作中提到】
: 我的理解,父母额外提前教都是拔,否则学校的老师教的也
: 足够了。推,只是督促做份内的事情。

d******e
发帖数: 2265
237
数学就是建立逻辑体系。
加法由数数得到。
乘法是加法的上层
然后出现除法
出现分数,无理数,代数。
至于解题我认为很多都是抽象语言能力和语言理解能力。

法阿
吗?

【在 k**n 的大作中提到】
: 你怎么找的美国教纲?我在我娃学校看了,除了教学目标,没看到他们乌七八糟的教法阿
: 惭愧的问,数学怎么教?俺的直觉是,数学没法教,就是拿着题讲一遍。讲完了,懂了
: 就懂了,没懂就没懂。每个题型都做过,那就八九不离十。 普通娃,不就是题海战吗?

d******e
发帖数: 2265
238
我认为还是要背的。
数学吗,本质就是提出一堆公里,然后开始推。
对于小孩,根本不需要理解最本质的东西。
从定理直接开始推理,计算这就是能力。
真正要理解这后面的东西要上元计算了。这是计算机系研究生的课了。

【在 s**********y 的大作中提到】
: 好的数学从不备九九表开始。
s**********y
发帖数: 509
239
赞一句 “我认为”, 至少有了主语。
不备九九表是 苏菲云说的。 对于不对, 大家自己判断。

【在 d******e 的大作中提到】
: 我认为还是要背的。
: 数学吗,本质就是提出一堆公里,然后开始推。
: 对于小孩,根本不需要理解最本质的东西。
: 从定理直接开始推理,计算这就是能力。
: 真正要理解这后面的东西要上元计算了。这是计算机系研究生的课了。

t******l
发帖数: 10908
240
我觉得这边版上很多娃确实 numeracy 很好。。。很多父母也对 numeracy 有兴趣,而
那些父母很多也是满分型的父母。
但我好像不是(我小学数学被红叉追屁股),我家的两娃好像都不是。。。这也可能就
是为啥我家对数学题的兴趣和专注力都不是太行的原因。。。我觉得每家的个性化家庭
教育,最重要的是符合自身的特点。。。我看着我家的特点,我觉得我教两娃数学基于
space-time pattern 的出发点还是对的。。。我家肯定不能上基于 numeracy 的数学
教育,否则多半我家都数学挂科不及格了。。。实话实说。

:数学就是建立逻辑体系。
:加法由数数得到。
相关主题
9岁, 如何报名考amc10二年级女儿的report
有明天考AMC 8的吗?觉得美帝数学进度慢的进来看一下
[转载] 为什么说usamo 简单我招,我是猪
进入Parenting版参与讨论
s**********y
发帖数: 509
241
number sense 和 算数 有关系。 有算数培养number sense, 倒也make sense。 以速
度和准确率为目标, 老让人觉得是在培养卖菜员工哈。

【在 t******l 的大作中提到】
: 我觉得这边版上很多娃确实 numeracy 很好。。。很多父母也对 numeracy 有兴趣,而
: 那些父母很多也是满分型的父母。
: 但我好像不是(我小学数学被红叉追屁股),我家的两娃好像都不是。。。这也可能就
: 是为啥我家对数学题的兴趣和专注力都不是太行的原因。。。我觉得每家的个性化家庭
: 教育,最重要的是符合自身的特点。。。我看着我家的特点,我觉得我教两娃数学基于
: space-time pattern 的出发点还是对的。。。我家肯定不能上基于 numeracy 的数学
: 教育,否则多半我家都数学挂科不及格了。。。实话实说。
:
: :数学就是建立逻辑体系。
: :加法由数数得到。

t******l
发帖数: 10908
242
现在我其实也理解了为啥我家大娃五年级加料班里纯计算考试排倒数了。。。如果借用
一下 Aspergers 的 hypernumeracy 的说法,我大胆猜测硅谷友谊村的平均线是
supernumeracy,而我家这种 normalnumeracy 的就成为 numeracy 方面的落后生了。

:我觉得这边版上很多娃确实 numeracy 很好。。。很多父母也对 numeracy 有兴趣,
而那些父母很多也是满分型的父母。
s**********y
发帖数: 509
243
点赞。 听起来比几岁会什么靠谱啊。

【在 l*****c 的大作中提到】
: 如果都是自己推导的,挺像我幼儿园的我,我当时也是自己推导出了乘除法,非十进制
: ,小数,负数这些。我觉得是有天赋的,可以引导一把,不要去培养。告诉他去学什么
: 想什么就行,用问题去诱惑他,让他自己去研究解决方案,不要去教。

t******l
发帖数: 10908
244
从这个讨论线程看过来,其实 99 表只是表象,用不用 99 表都是在讨论 number
sense,也就是 numeracy。
但是有争议的说,无论是 1983 AIME Problem #4,还是 2015 AIME-I Problem #3,
numeracy 除了让解题中计算速度快一点以外,没有其他的帮助。。。但普通娃的问题
是在根本就解不出来,completely clueless,totally lost,而不是 numeracy 好点
差点算得快点慢点。。。。

:number sense 和 算数 有关系。 有算数培养number sense, 倒也make sense。 以速
n******e
发帖数: 1046
245
会乘法表的小孩数学不一定好,不会乘法表的小孩数学一定不会好,因为速度摆在那里。
别告诉我你让你孩子做9*9让他去算9个9相加就好了
要不要背乘法表,自己读过书来的,我不会人云亦云,为突出自己孩子与众不同不是死
背那就不背,很简单


: 好的数学从不备九九表开始。



【在 s**********y 的大作中提到】
: 点赞。 听起来比几岁会什么靠谱啊。
t******l
发帖数: 10908
246
对于 2015 AIME-I Problem #3,我和娃都还没看网上的 solution。。。我不知道会不
会有人直接去猜那个三次方数,或者其他天外飞仙的纯数解法。。。但我觉得普通娃的
intuition 是 algebraic manipulation,在这里就是 algebraic factorization。。
。而我跟娃说的两个解法一个是 “make 1 then cancel 1",另一个是 guess a
root then polynomial division。
不过我发现计算时也不一定算表达式,可以反过来算,能避免不必要的代数运算,不过
本来的目的是学代数运算。

:从这个讨论线程看过来,其实 99 表只是表象,用不用 99 表都是在讨论 number
:sense,也就是 numeracy。
t******l
发帖数: 10908
247
美帝学校二年级或三年级会教背 99 表的,当然比这边大多数父母期望的年龄要晚不少。

:会乘法表的小孩数学不一定好,不会乘法表的小孩数学一定不会好,因为速度摆在那
里。
:别告诉我你让你孩子做9*9让他去算9个9相加就好了
t******l
发帖数: 10908
248
刚才跟娃提了一下,但发现计算 (17^3 - 1) / 16 反而比计算 (17*18+1) 更慢。。。
说明 intuition 在这种题目上也许能从 supernumeracy 那边找回一些场子。

:对于 2015 AIME-I Problem #3,我和娃都还没看网上的 solution。。。我不知道会
不会有人直接去猜那个三次方数,或者其他天外飞仙的纯数解法。。。但我觉得普通娃
的 intuition 是 algebraic manipulation,在这里就是 algebraic factorization。
。。而我跟娃说的两个解法一个是 “make 1 then cancel 1",另一个是 guess a
:root then polynomial division。
s**********y
发帖数: 509
249
2015 AIME-I Problem #3 两种解法中规中矩。 但是不能要求娃一上来就会。 应该有
一些例如, 勾股炫均为整数,求证其中一个必为偶数之类的题目上手。
1983 AIME Problem #4 的两种解法太过纤细。 现提供一种简单暴力解法。
以AC为X, AC种点为原点,建立坐标系。
O 的坐标为: (0, 2sqrt(10))
B 的坐标为: (-4/5sqrt(10), 3/5sqrt(10))
直接算距离即可。

以速

【在 t******l 的大作中提到】
: 从这个讨论线程看过来,其实 99 表只是表象,用不用 99 表都是在讨论 number
: sense,也就是 numeracy。
: 但是有争议的说,无论是 1983 AIME Problem #4,还是 2015 AIME-I Problem #3,
: numeracy 除了让解题中计算速度快一点以外,没有其他的帮助。。。但普通娃的问题
: 是在根本就解不出来,completely clueless,totally lost,而不是 numeracy 好点
: 差点算得快点慢点。。。。
:
: :number sense 和 算数 有关系。 有算数培养number sense, 倒也make sense。 以速

s**********y
发帖数: 509
250
9 * 9 难道不从 九个九 相加开始算?
9^1, 9^2, 9^3, 难道也被?
9^{1/2}, 9^{1/4}, 难道也被?

里。

【在 n******e 的大作中提到】
: 会乘法表的小孩数学不一定好,不会乘法表的小孩数学一定不会好,因为速度摆在那里。
: 别告诉我你让你孩子做9*9让他去算9个9相加就好了
: 要不要背乘法表,自己读过书来的,我不会人云亦云,为突出自己孩子与众不同不是死
: 背那就不背,很简单
:
:
: 好的数学从不备九九表开始。
:

相关主题
大学录取率来点美好的时光
学校早晚要教的东西,早早学会了又如何?有些小学四年级的数学题不太会做
amc 10 的A, B卷有啥区别转载: 从首个IMO季军谈起 by 付云皓
进入Parenting版参与讨论
n******e
发帖数: 1046
251
理解与运算是两回事,你就教你孩子去9个9相加吧,挺好,跟你一样出色,其他说多了
都是多余


: 9 * 9 难道不从 九个九 相加开始算?

: 9^1, 9^2, 9^3, 难道也被?

: 9^{1/2}, 9^{1/4}, 难道也被?

: 里。



【在 s**********y 的大作中提到】
: 9 * 9 难道不从 九个九 相加开始算?
: 9^1, 9^2, 9^3, 难道也被?
: 9^{1/2}, 9^{1/4}, 难道也被?
:
: 里。

s**********y
发帖数: 509
252
competition math 与一般math 不同。 到了这一步, 天赋有用, 只凭天赋是好像是
断断不可得。

【在 t******l 的大作中提到】
: 对于 2015 AIME-I Problem #3,我和娃都还没看网上的 solution。。。我不知道会不
: 会有人直接去猜那个三次方数,或者其他天外飞仙的纯数解法。。。但我觉得普通娃的
: intuition 是 algebraic manipulation,在这里就是 algebraic factorization。。
: 。而我跟娃说的两个解法一个是 “make 1 then cancel 1",另一个是 guess a
: root then polynomial division。
: 不过我发现计算时也不一定算表达式,可以反过来算,能避免不必要的代数运算,不过
: 本来的目的是学代数运算。
:
: :从这个讨论线程看过来,其实 99 表只是表象,用不用 99 表都是在讨论 number
: :sense,也就是 numeracy。

t******l
发帖数: 10908
253
我觉得你说的 1983 AIME #4 的解几解法挺好的。。。两种解几解法的复杂度差不多。
。。我觉得欧几是 chase triangles,而解几是 chase coordinates,在三角形数量多
到一定程度的时候,解几解法实际上更清晰。


:2015 AIME-I Problem #3 两种解法中规中矩。 但是不能要求娃一上来就会。 应该有
t******l
发帖数: 10908
254
我觉得对待 Competition Math 有两条路。
正规的道路就是要出成绩,但对普通娃非真爱娃,就算有一定天赋,我觉得这条路也会
挺苦的,因为要大量练习速度题型正确率,有些违背天性。好处是 “成者为王”。
另一条就是 AlphaGo 型 “解题基本靠猜” 的利用 intuition 的偷懒酱油竞赛之路。
这条路基本很难拿牌,但就好比快乐 chess 练练身体,走 “旱涝保收” 之路 。

:competition math 与一般math 不同。 到了这一步, 天赋有用, 只凭天赋是好像
s**********y
发帖数: 509
255
赞旱涝保收
第一条路太难了。 史上仿佛也没听说过能坚持到最后的。

【在 t******l 的大作中提到】
: 我觉得对待 Competition Math 有两条路。
: 正规的道路就是要出成绩,但对普通娃非真爱娃,就算有一定天赋,我觉得这条路也会
: 挺苦的,因为要大量练习速度题型正确率,有些违背天性。好处是 “成者为王”。
: 另一条就是 AlphaGo 型 “解题基本靠猜” 的利用 intuition 的偷懒酱油竞赛之路。
: 这条路基本很难拿牌,但就好比快乐 chess 练练身体,走 “旱涝保收” 之路 。
:
: :competition math 与一般math 不同。 到了这一步, 天赋有用, 只凭天赋是好像
: 是

t******l
发帖数: 10908
256
我觉得奥赛奖牌都是第一条路坚持到最后的。。。只不过我觉得这条路更像竞技体育(
也包括竞技国象围棋),成功的凤毛麟角,不成的代价不小。


:赞旱涝保收
s**********y
发帖数: 509
257
好像都不是扑通蛙吧?

【在 t******l 的大作中提到】
: 我觉得奥赛奖牌都是第一条路坚持到最后的。。。只不过我觉得这条路更像竞技体育(
: 也包括竞技国象围棋),成功的凤毛麟角,不成的代价不小。
:
: :
: :赞旱涝保收

t******l
发帖数: 10908
258
竞赛拿牌的都不是普通娃,是天赋+真爱娃。

:好像都不是扑通蛙吧?
t******l
发帖数: 10908
259
另外我们这边确实有 “快乐酱油 chess班”、“快乐酱油花滑班”、“快乐酱油游泳
班”、“快乐酱油田径班”。。。所以我在想 “快乐酱油奥外数学” 的可能性,不求
玩耍的那种快乐,但求比作为学校 math 的补充,但也不会鸡飞狗跳。
当然这种也会有很多问题,一个是娃往往最终会 quit。。。另一个是没有竞争驱动,
娃的兴趣度也低一些。有时候为了娃不翘课也要跟娃争执。。。但其实竞争驱动也是双
刃剑。竞争的时候无比兴奋,但一旦持续竞争不过也可能无比空虚沮丧,这时候发生的
争执可能更大更不可调和。

:赞旱涝保收
d****r
发帖数: 300
260
这么说吧,申请MIT的usamo qualifier有85%以上的录取统计。MIT会让你填你aime的分
数即使你没进usamo。数学在美国地位还是很高的。但进数学top500的确不容易。

【在 f*c 的大作中提到】
: 俺的意思是,AIME或者USAMO qualifier这些级别对进大学能帮到什么程度,比如说进
: 个UCLA/UCB什么的。个人觉得近年来数学竞赛有正规化,职业化的倾向。真想做数学家
: 的话,16/7岁这个年纪往竞赛这个方向走得太厉害多少有些浪费的感觉。
:
: 数的

相关主题
新加坡数学对小学生帮助大吗?好学生去普通公立高中真的会被耽误吗?
WISC-V 智商测试陶天才论天才 (转载)
今天和幼儿园老师开会review娃的progress,非常upsetExplaining Your Math: Unnecessary at Best, Encumbering at W
进入Parenting版参与讨论
s**********y
发帖数: 509
261
快乐酱油多了, 也是成就吧。

【在 t******l 的大作中提到】
: 另外我们这边确实有 “快乐酱油 chess班”、“快乐酱油花滑班”、“快乐酱油游泳
: 班”、“快乐酱油田径班”。。。所以我在想 “快乐酱油奥外数学” 的可能性,不求
: 玩耍的那种快乐,但求比作为学校 math 的补充,但也不会鸡飞狗跳。
: 当然这种也会有很多问题,一个是娃往往最终会 quit。。。另一个是没有竞争驱动,
: 娃的兴趣度也低一些。有时候为了娃不翘课也要跟娃争执。。。但其实竞争驱动也是双
: 刃剑。竞争的时候无比兴奋,但一旦持续竞争不过也可能无比空虚沮丧,这时候发生的
: 争执可能更大更不可调和。
:
: :赞旱涝保收

n******g
发帖数: 2201
262
what about Cal Tech

【在 d****r 的大作中提到】
: 这么说吧,申请MIT的usamo qualifier有85%以上的录取统计。MIT会让你填你aime的分
: 数即使你没进usamo。数学在美国地位还是很高的。但进数学top500的确不容易。

d****r
发帖数: 300
263
问的好,Cal Tech和MIT都很看重这个。哈佛不是那么明显。

【在 n******g 的大作中提到】
: what about Cal Tech
t******l
发帖数: 10908
264
我觉得 “快乐酱油课后加料”(当然也不可能是电脑打游戏的那种快乐,是相对 “吃
苦刷题求名次” 而言的)就是父母该干的事,就好比古人说的 “师傅领进门”。
而 “真爱竞争拿牌” 是娃自己的选择自己的事,就好比古人说的 “修行在自身”。

:快乐酱油多了, 也是成就吧。
s**********y
发帖数: 509
265
还是潮老大站得高, 看得远, 说得准。

【在 t******l 的大作中提到】
: 我觉得 “快乐酱油课后加料”(当然也不可能是电脑打游戏的那种快乐,是相对 “吃
: 苦刷题求名次” 而言的)就是父母该干的事,就好比古人说的 “师傅领进门”。
: 而 “真爱竞争拿牌” 是娃自己的选择自己的事,就好比古人说的 “修行在自身”。
:
: :快乐酱油多了, 也是成就吧。

f*******e
发帖数: 3433
266
对于愿意被99表的小孩,在理解的基础上背熟了肯定有帮助。理解也很重要因为考试考
的都是wording problem. 对于不愿意背全99表的可以用下面的方法减少背的东西同时
不对速度有太大影响:
背诵
2*2 。。。 2*5
3*2.。。。3*5
。。。
9*2 。。。 9*5
10*2 。。。10*9
其他的可以这么算
9*9 = 9*(10-1) = 90-9 = 81
7*8 = 7*(10-2) = 70 - 14 = 60 - 4 = 56

【在 n******e 的大作中提到】
: 理解与运算是两回事,你就教你孩子去9个9相加吧,挺好,跟你一样出色,其他说多了
: 都是多余
:
:
: 9 * 9 难道不从 九个九 相加开始算?
:
: 9^1, 9^2, 9^3, 难道也被?
:
: 9^{1/2}, 9^{1/4}, 难道也被?
:
: 里。
:

t******l
发帖数: 10908
267
而我自己有时做个 AIME 题目再教娃,就是出于这个目的。。。因为我觉得不少答案解
法并不是针对 “快乐酱油奥外数学” 而写的。
我其实对这边的 “快乐酱油 chess 课” 略知一二。。。我觉得那种首先是侧重 ”
play the middle game like a magician (intuition)” 那块,我觉得原因是那是普
通娃觉得最有趣的部分。。。其次是侧重 “play the end-game like a machine“,
原因可能是简洁明了、急用先学、立杆见影。符合普通娃的心理特点。。。而 ”play
the opening game like a book” 是相对而言避免的部分,因为普通娃可能最烦死记
硬背也不知道将来有没有用的东西。
我觉得以上这个思路可能也适合 “酱油奥外数学”。

:快乐酱油多了, 也是成就吧。
d****r
发帖数: 300
268
说的好,父母只是jump start一下,剩下的全靠娃自己了。你还化时间研究aime题,已
经不容易了。

【在 t******l 的大作中提到】
: 我觉得 “快乐酱油课后加料”(当然也不可能是电脑打游戏的那种快乐,是相对 “吃
: 苦刷题求名次” 而言的)就是父母该干的事,就好比古人说的 “师傅领进门”。
: 而 “真爱竞争拿牌” 是娃自己的选择自己的事,就好比古人说的 “修行在自身”。
:
: :快乐酱油多了, 也是成就吧。

n******g
发帖数: 2201
269
他也许小时候喜欢数学

【在 d****r 的大作中提到】
: 说的好,父母只是jump start一下,剩下的全靠娃自己了。你还化时间研究aime题,已
: 经不容易了。

t******l
发帖数: 10908
270
其实一来是因为 AoPS 有些题目的 solution 我实在是不能忍。。。其次网上那些
solution 只告诉其然,也不告诉其所以然,也就是咋想出来的,这个 intuition /
wishful thinking 的思路过程。。。再说我自己也懒得看砖头数学书以后教,还不如
自己做几题当例题教娃。

:说的好,父母只是jump start一下,剩下的全靠娃自己了。你还化时间研究aime题,
已经不容易了。
相关主题
Explaining Your Math: Unnecessary at Best, Encumbering at W有明天考AMC 8的吗?
是不是任我儿子继续迷象棋[转载] 为什么说usamo 简单
9岁, 如何报名考amc10二年级女儿的report
进入Parenting版参与讨论
f*******e
发帖数: 3433
271
你对数学也是真爱啊。我高中时还能做做AIME,现在这么多年不怎么用数学早交回给老
师了(虽然你给的那个AIME83年第4题我现在还能做做)。等娃长大了,做AIME时也许
把他丢给补习班更省事。

play

【在 t******l 的大作中提到】
: 而我自己有时做个 AIME 题目再教娃,就是出于这个目的。。。因为我觉得不少答案解
: 法并不是针对 “快乐酱油奥外数学” 而写的。
: 我其实对这边的 “快乐酱油 chess 课” 略知一二。。。我觉得那种首先是侧重 ”
: play the middle game like a magician (intuition)” 那块,我觉得原因是那是普
: 通娃觉得最有趣的部分。。。其次是侧重 “play the end-game like a machine“,
: 原因可能是简洁明了、急用先学、立杆见影。符合普通娃的心理特点。。。而 ”play
: the opening game like a book” 是相对而言避免的部分,因为普通娃可能最烦死记
: 硬背也不知道将来有没有用的东西。
: 我觉得以上这个思路可能也适合 “酱油奥外数学”。
:

d****r
发帖数: 300
272
我见过象你这样的家长推孩成功后就开班了,一箭双雕。看好你,哈哈。

【在 t******l 的大作中提到】
: 其实一来是因为 AoPS 有些题目的 solution 我实在是不能忍。。。其次网上那些
: solution 只告诉其然,也不告诉其所以然,也就是咋想出来的,这个 intuition /
: wishful thinking 的思路过程。。。再说我自己也懒得看砖头数学书以后教,还不如
: 自己做几题当例题教娃。
:
: :说的好,父母只是jump start一下,剩下的全靠娃自己了。你还化时间研究aime题,
: 已经不容易了。

t******l
发帖数: 10908
273
你应该说我对俩娃是真爱 :-P 。。。这 AIME 是因为要拿例题教娃的时候,我才去做
的。。。否则我看也不看。
另外我早就被那位 id 叫马克的哥们嘲笑除了集合论啥都记不住的感觉。。。所以我反
正高中时除了集合论以外其他啥都记不住,现在也差不多就是了。。。剩下的就一个字
:“猜”!。。。如果还要加上一个修饰语:“蒙”!

:你对数学也是真爱啊。我高中时还能做做AIME,现在这么多年不怎么用数学早交回给
老师了(虽然你给的那个AIME83年第4题我现在还能做做)。等娃长大了,做AIME时也许
:把他丢给补习班更省事。
t******l
发帖数: 10908
274
另外老实说其实我跟着娃是从 MOEMS / AMC 8 题目做起的。。。当然一开始的原因是
翻答案读一遍要花 30 秒,心算 15 秒就够了。所以心算了干脆不读答案了,省了 15
秒。。。// run

:你对数学也是真爱啊。我高中时还能做做AIME,现在这么多年不怎么用数学早交回给
老师了(虽然你给的那个AIME83年第4题我现在还能做做)。等娃长大了,做AIME时也许
:把他丢给补习班更省事。
t******l
发帖数: 10908
275
另外我前面做的那个AIME83年第4题的解几心算解法,我前天也给娃教了一遍,帮娃复
习一遍直线的 rise-over-run 的概念,还用 Documentary 那个 Air Disasters --
The final blow 里的 Airbus A320 里一个镜头,理论联系一下实际,说明实践上为啥
更实用的是笛卡尔的坐标几何,而不是欧几里德的追逐三角形几何(因为太饶舌)。。
。反正我跟娃说过了,AIME 我花时间做了就不能白做,你就得学。这是 respect my
time and effort。。。哼哼。。。

:你对数学也是真爱啊。我高中时还能做做AIME,现在这么多年不怎么用数学早交回给
老师了(虽然你给的那个AIME83年第4题我现在还能做做)。等娃长大了,做AIME时也许
:把他丢给补习班更省事。
f*******e
发帖数: 3433
276
你娃上高中了吗,就开始AIME了?我怎么记得你娃才上初中啊。

也许

【在 t******l 的大作中提到】
: 另外我前面做的那个AIME83年第4题的解几心算解法,我前天也给娃教了一遍,帮娃复
: 习一遍直线的 rise-over-run 的概念,还用 Documentary 那个 Air Disasters --
: The final blow 里的 Airbus A320 里一个镜头,理论联系一下实际,说明实践上为啥
: 更实用的是笛卡尔的坐标几何,而不是欧几里德的追逐三角形几何(因为太饶舌)。。
: 。反正我跟娃说过了,AIME 我花时间做了就不能白做,你就得学。这是 respect my
: time and effort。。。哼哼。。。
:
: :你对数学也是真爱啊。我高中时还能做做AIME,现在这么多年不怎么用数学早交回给
: 老师了(虽然你给的那个AIME83年第4题我现在还能做做)。等娃长大了,做AIME时也许
: :把他丢给补习班更省事。

t******l
发帖数: 10908
277
我不是让我娃做 AIME。。。我就是利用 AIME 里,比较容易但同时稍有挑战的前几题
,作为兴趣诱饵,来提高教娃数学的效率和娃娃的兴趣。。。正如林彪同志在毛主席语
录里说的,要 “急用先学”,才能 “立竿见影”。。。我就是用 AIME 前几题的稍有
挑战性,来人工创建 “急用“ 的需求和环境,希望达到 “立竿见影” 的目的。。。

:你娃上高中了吗,就开始AIME了?我怎么记得你娃才上初中啊。
t******l
发帖数: 10908
278
比如 2015 AIME-I Problem 3 的一个解法里,用到 (n+1)^3 的展开。。。这公式娃肯
定背不出来,用分配率死算又太烦,“急用” 了不是,那就正好复习 binomial
theorem。。。但 binomial theorem 也背不出来不是?又 “急用” 了不是?正好临
场概念级不严格椅子号建模用组合理论证明一下,得出展开成 n^3 + C(3,2)*n^2 + C(
3,1)*n + 1
急用先学降低枯燥度,所以降低教学阻力。

:我不是让我娃做 AIME。。。我就是利用 AIME 里,比较容易但同时稍有挑战的前几题
:,作为兴趣诱饵,来提高教娃数学的效率和娃娃的兴趣。。。正如林彪同志在毛主席
语录里说的,要 “急用先学”,才能 “立竿见影”。。。我就是用 AIME 前几题的稍
有挑战性,来人工创建 “急用“ 的需求和环境,希望达到 “立竿见影” 的目的。。。
t******l
发帖数: 10908
279
比如另一个解法里,需要用到 x^3 - 1 = (x-1) *(x^2 + x + 1)。。。这公式娃不要
说背不出来,连听都没听说过(play with algebraic cubes)。。。“急用” 了不是
?。。。
所以就先观察该多项式对应的方程 x^3 - 1 = 0,可知有一个根为 x=1。。。根据多项
式理论,该多项式必然能被多项式 x-1 整除。。。那剩下的咋办?。。。又急用了不
是?。。。办法一,多项式长除法,正好复习一遍。。。这长除法做完了,但是好像有
点烦。。。OK 还有诡异的办法二:利用指数的 “移位对称性”(algebraic
telescope tool) 凑正负抵消,反凑一个多项式 x^2 + x + 1 出来。
这样通过 “急用”,把枯燥变得稍微有趣一些,降低 normalnumeracy 娃学数学的最
大阻力 -- 枯燥。

:比如 2015 AIME-I Problem 3 的一个解法里,用到 (n+1)^3 的展开。。。这公式娃
肯定背不出来,用分配率死算又太烦,“急用” 了不是,那就正好复习 binomial
:theorem。。。但 binomial theorem 也背不出来不是?又 “急用” 了不是?正好临
s**********y
发帖数: 509
280
方法倒是好的。 只是这种数学题, 设计时就是要把几层套道一块。 能看出这几层,
是考眼力的地方。 我倒是觉得, 应该把单独的层搞了, 再来做题。 好题不再哈。

【在 t******l 的大作中提到】
: 比如另一个解法里,需要用到 x^3 - 1 = (x-1) *(x^2 + x + 1)。。。这公式娃不要
: 说背不出来,连听都没听说过(play with algebraic cubes)。。。“急用” 了不是
: ?。。。
: 所以就先观察该多项式对应的方程 x^3 - 1 = 0,可知有一个根为 x=1。。。根据多项
: 式理论,该多项式必然能被多项式 x-1 整除。。。那剩下的咋办?。。。又急用了不
: 是?。。。办法一,多项式长除法,正好复习一遍。。。这长除法做完了,但是好像有
: 点烦。。。OK 还有诡异的办法二:利用指数的 “移位对称性”(algebraic
: telescope tool) 凑正负抵消,反凑一个多项式 x^2 + x + 1 出来。
: 这样通过 “急用”,把枯燥变得稍微有趣一些,降低 normalnumeracy 娃学数学的最
: 大阻力 -- 枯燥。

相关主题
觉得美帝数学进度慢的进来看一下学校早晚要教的东西,早早学会了又如何?
我招,我是猪amc 10 的A, B卷有啥区别
大学录取率来点美好的时光
进入Parenting版参与讨论
n*******l
发帖数: 2911
281
你说的这些,基本都是“事后诸葛亮”。我觉得,如果一个学生知道x^3-1有一个根
是x=1,从而x-1必然是一个因子,那么十有八九这个学生知道立方差公式。后面那
个所谓的反凑,基本也是知道了这些公式之后写出来唬人的。
很多时候,一个题目的若干“巧妙”的解法,是基于很多基础知识和经验的。

【在 t******l 的大作中提到】
: 比如另一个解法里,需要用到 x^3 - 1 = (x-1) *(x^2 + x + 1)。。。这公式娃不要
: 说背不出来,连听都没听说过(play with algebraic cubes)。。。“急用” 了不是
: ?。。。
: 所以就先观察该多项式对应的方程 x^3 - 1 = 0,可知有一个根为 x=1。。。根据多项
: 式理论,该多项式必然能被多项式 x-1 整除。。。那剩下的咋办?。。。又急用了不
: 是?。。。办法一,多项式长除法,正好复习一遍。。。这长除法做完了,但是好像有
: 点烦。。。OK 还有诡异的办法二:利用指数的 “移位对称性”(algebraic
: telescope tool) 凑正负抵消,反凑一个多项式 x^2 + x + 1 出来。
: 这样通过 “急用”,把枯燥变得稍微有趣一些,降低 normalnumeracy 娃学数学的最
: 大阻力 -- 枯燥。

t******l
发帖数: 10908
282
这个我也教了。
16*p + 1 = n^3,我就问娃如果是 MOEMS 咋 guess & check,娃说随便猜个 n,计算
其三次方,然后减一,然后做 factorization。
OK 我告诉娃,大名词上,你实际上找到了这个 search problem 所对应的 decision
problem 的算术解法,也就是 guess & check 里的 check。。。所以现在我们要做的
是两件事。。。第一,把这个算术 check 换成代数 check。。。第二,看看根据换出
来的代数 check 能不能找到代数 guess,也就是 algebraic wishful thinking。
所以闲话少说,第一步就是如何 algebraic factorize 多项式 n^3 - 1,并且保证分
开的两项都是整数的问题。。。两种方法(1)凑1 消 1,(2)猜测该多项式对应的方
程的一个整数根。。。然后就是后面了。

:方法倒是好的。 只是这种数学题, 设计时就是要把几层套道一块。 能看出这几层
, 是考眼力的地方。 我倒是觉得, 应该把单独的层搞了, 再来做题。 好题不再哈
t******l
发帖数: 10908
283
你的假设是错误的,至少我背不出立方差公式,吃饭喝酒后肯定背不出来。。。另外你
背得出 x^5 - 1?。。。x^7 - 1?。。。再咋地 x^307 - 1 总背不出来了吧。
其实你这个确实说明了自己心算一遍很重要。。。否则看答案后教娃,更容易事后猪哥
亮。。。

:你说的这些,基本都是“事后诸葛亮”。我觉得,如果一个学生知道x^3-1有一个根
:是x=1,从而x-1必然是一个因子,那么十有八九这个学生知道立方差公式。后面那
n*******l
发帖数: 2911
284
其实,第一步你应该看出来16p+1是个奇数,所以右侧应该是(2n+1)^3。不过对于
这道题这一步不重要就是了。但是我觉得这是基本要求。

【在 t******l 的大作中提到】
: 这个我也教了。
: 16*p + 1 = n^3,我就问娃如果是 MOEMS 咋 guess & check,娃说随便猜个 n,计算
: 其三次方,然后减一,然后做 factorization。
: OK 我告诉娃,大名词上,你实际上找到了这个 search problem 所对应的 decision
: problem 的算术解法,也就是 guess & check 里的 check。。。所以现在我们要做的
: 是两件事。。。第一,把这个算术 check 换成代数 check。。。第二,看看根据换出
: 来的代数 check 能不能找到代数 guess,也就是 algebraic wishful thinking。
: 所以闲话少说,第一步就是如何 algebraic factorize 多项式 n^3 - 1,并且保证分
: 开的两项都是整数的问题。。。两种方法(1)凑1 消 1,(2)猜测该多项式对应的方
: 程的一个整数根。。。然后就是后面了。

t******l
发帖数: 10908
285
另外全唐诗说了,AIME 题开须折直须折,莫等题过空折枝。。。哥们您太怜香惜玉了
。。。

:方法倒是好的。 只是这种数学题, 设计时就是要把几层套道一块。 能看出这几层
, 是考眼力的地方。 我倒是觉得, 应该把单独的层搞了, 再来做题。 好题不再哈
n*******l
发帖数: 2911
286
你背不出那是因为你已经不是中学生了。

【在 t******l 的大作中提到】
: 你的假设是错误的,至少我背不出立方差公式,吃饭喝酒后肯定背不出来。。。另外你
: 背得出 x^5 - 1?。。。x^7 - 1?。。。再咋地 x^307 - 1 总背不出来了吧。
: 其实你这个确实说明了自己心算一遍很重要。。。否则看答案后教娃,更容易事后猪哥
: 亮。。。
:
: :你说的这些,基本都是“事后诸葛亮”。我觉得,如果一个学生知道x^3-1有一个根
: :是x=1,从而x-1必然是一个因子,那么十有八九这个学生知道立方差公式。后面那

t******l
发帖数: 10908
287
想到了,但是这条路我没 factor 出来。wishful thinking 不总是 work。

:其实,第一步你应该看出来16p+1是个奇数,所以右侧应该是(2n+1)^3。不过对于
:这道题这一步不重要就是了。但是我觉得这是基本要求。
c***x
发帖数: 1826
288

总算来了个严正视听的。

【在 s***n 的大作中提到】
: 接着谈。
: 因为Asperger不必然导致逻辑思维强或数学好,所以当一个Asperger孩子在小时候表现
: 出“数学天分“,家长需要判断这种天分是数学Gifted,还是hypernumeracy。回到楼主
: 孩子的例子,我前文已经说了,由于孩子的逻辑思维能力还不明,无法判断他的数学天
: 分。如果楼主的孩子有ASD/Asperger方面的担心,那对他表现出来的数学“天分",要
: 有更多一层的考量。Asperger/hypernumeracy的孩子由于对数字的喜爱和记忆力强,他
: 们中一些孩子在幼儿园和小学低年级可以表现出很强的数学水平。但哪怕是这些孩子中
: 间也只有小部分是真正的数学Gifted。
: 对那些表现出数学天分的孩子,区分数学Gifted,hypernumeracy对教育有一定的指导
: 意义。对于前者,要提高兴趣,拓展数学教育的广度和深度;对后者重点要放在对已学

s**********y
发帖数: 509
289
哈哈

【在 t******l 的大作中提到】
: 另外全唐诗说了,AIME 题开须折直须折,莫等题过空折枝。。。哥们您太怜香惜玉了
: 。。。
:
: :方法倒是好的。 只是这种数学题, 设计时就是要把几层套道一块。 能看出这几层
: , 是考眼力的地方。 我倒是觉得, 应该把单独的层搞了, 再来做题。 好题不再哈
: 。

n*******l
发帖数: 2911
290
(2n+1)^3展开呗。我现在基本就是不动纸笔不读书啊。

【在 t******l 的大作中提到】
: 想到了,但是这条路我没 factor 出来。wishful thinking 不总是 work。
:
: :其实,第一步你应该看出来16p+1是个奇数,所以右侧应该是(2n+1)^3。不过对于
: :这道题这一步不重要就是了。但是我觉得这是基本要求。

相关主题
有些小学四年级的数学题不太会做WISC-V 智商测试
转载: 从首个IMO季军谈起 by 付云皓今天和幼儿园老师开会review娃的progress,非常upset
新加坡数学对小学生帮助大吗?好学生去普通公立高中真的会被耽误吗?
进入Parenting版参与讨论
t******l
发帖数: 10908
291
你这么说也对。。。但如果目的不是大学入学高分,而是人生相对轻松的话。。。那中
学背得出,大学背不出,其实反而不好。
另外 x^7 - 1 总是背不出来的。。。当然其实数学竞赛那波确实背诵很多分解和级数
求和公式。。。这也算很多人放弃数学竞技,把数学竞赛就是当初学数学的工具的原因
之一。。。大部分普通娃确实不是 supernumeracy,对背公式兴趣不大。

:你背不出那是因为你已经不是中学生了。
n*******l
发帖数: 2911
292
x^7-1 = (x-1)(x^6+x^5+...+x+1)
这个玩意儿根本就不需要背,是个基本规律啊。

【在 t******l 的大作中提到】
: 你这么说也对。。。但如果目的不是大学入学高分,而是人生相对轻松的话。。。那中
: 学背得出,大学背不出,其实反而不好。
: 另外 x^7 - 1 总是背不出来的。。。当然其实数学竞赛那波确实背诵很多分解和级数
: 求和公式。。。这也算很多人放弃数学竞技,把数学竞赛就是当初学数学的工具的原因
: 之一。。。大部分普通娃确实不是 supernumeracy,对背公式兴趣不大。
:
: :你背不出那是因为你已经不是中学生了。

t******l
发帖数: 10908
293
(2*n + 1)^3 - 1
= 8*n^3 + 12*n^2 + 6*n
= (2*n) * (4*n^2 + 6*n + 3)
接下去怎么做?跟其他办法一个套路?


:(2n+1)^3展开呗。我现在基本就是不动纸笔不读书啊。
n*******l
发帖数: 2911
294
16p+1 = (2n+1)^3 = 8n^3 + 12n^2 + 6n+1
16p = 8n^3 +12n^2 +6n
8p = 4n^3 + 6n^2 +3n = n(4n^2 + 6n+3)
要么 n=8, p = 4n^2 + 6n+3 = 307
要么 p=n, 8= 4n^2 +6n+3, 无解。

【在 t******l 的大作中提到】
: (2*n + 1)^3 - 1
: = 8*n^3 + 12*n^2 + 6*n
: = (2*n) * (4*n^2 + 6*n + 3)
: 接下去怎么做?跟其他办法一个套路?
:
: :
: :(2n+1)^3展开呗。我现在基本就是不动纸笔不读书啊。

t******l
发帖数: 10908
295
这 pattern 我确实不知道。。。我确实只知道能被 x-1 整除,剩下的要么做长除法发
现 pattern,要么指数移位抵消反凑。
这个的确说明每个人的思路差别很大,因材施教才是正道。。。所以父母自己解题可能
还是挺重要的,毕竟有相当的相似性。

:x^7-1 = (x-1)(x^6+x^5+...+x+1)
s**********y
发帖数: 509
296
那条路还不如分解q^3-1快,let 16p+1=q^3。

【在 t******l 的大作中提到】
: 想到了,但是这条路我没 factor 出来。wishful thinking 不总是 work。
:
: :其实,第一步你应该看出来16p+1是个奇数,所以右侧应该是(2n+1)^3。不过对于
: :这道题这一步不重要就是了。但是我觉得这是基本要求。

t******l
发帖数: 10908
297
那如果题目改成 17*p + 1 = k^3 ?


:16p+1 = (2n+1)^3 = 8n^3 + 12n^2 + 6n+1
t******l
发帖数: 10908
298
我娃的反馈认为直接分解 q^3 - 1 是她目前看到的最佳解法。

:那条路还不如分解q^3-1快,let 16p+1=q^3。
s**********y
发帖数: 509
299
一个数如果是两个(不相同的)质数的乘积, 这个数只有四个印子。 17 makes it
easier.

【在 t******l 的大作中提到】
: 那如果题目改成 17*p + 1 = k^3 ?
:
: :
: :16p+1 = (2n+1)^3 = 8n^3 + 12n^2 + 6n+1

n*******l
发帖数: 2911
300
难道不是首先看到17p+1必须是个偶数,然后右端项应该是(2k)^3?
然后17p = (2k)^3-1,分解因式?

【在 t******l 的大作中提到】
: 那如果题目改成 17*p + 1 = k^3 ?
:
: :
: :16p+1 = (2n+1)^3 = 8n^3 + 12n^2 + 6n+1

相关主题
陶天才论天才 (转载)9岁, 如何报名考amc10
Explaining Your Math: Unnecessary at Best, Encumbering at W有明天考AMC 8的吗?
是不是任我儿子继续迷象棋[转载] 为什么说usamo 简单
进入Parenting版参与讨论
t******l
发帖数: 10908
301
哦,我的原意是想给个奇数来对付一下那个 (2*n + 1) 奇偶性的解法。。。生造数字的
时候我没注意到 17 质数。// blush


:一个数如果是两个(不相同的)质数的乘积, 这个数只有四个印子。 17 makes it
s**********y
发帖数: 509
302
haha 题不好出的

it

【在 t******l 的大作中提到】
: 哦,我的原意是想给个奇数来对付一下那个 (2*n + 1) 奇偶性的解法。。。生造数字的
: 时候我没注意到 17 质数。// blush
:
: :
: :一个数如果是两个(不相同的)质数的乘积, 这个数只有四个印子。 17 makes it

t******l
发帖数: 10908
303
属实。。。另外小娃正拖着我陪她 reading。。。结果我 “一边xx一边yy” 两边闹笑
话,小娃还怀疑了我给她说错的词。。。充分说明了 syuan 专注力理论。。。

:haha 题不好出的
o***n
发帖数: 2074
304
这个标准是专家提出的。至于多大天分, 你可以参考Terence Tao 的童年。

了。

【在 P******e 的大作中提到】
: 哈哈,照你这个标准,我小娃是数学天才,她23个月就能数9个小球,2周岁能数20个了。
: 请问你这个标准的来源是?您老一拍脑门想的?

t******l
发帖数: 10908
305
我个人猜测这个可能确实可以大概率地预测娃的数学的上限。。。不管其背后的原因是
能力限制、还是小时数限制、还是兴趣限制。

:这个标准是专家提出的。至于多大天分, 你可以参考Terence Tao 的童年。
t******l
发帖数: 10908
306
对付完小娃回来。。。我觉得上面确实说明因材施教的重要性。。。确实有人/娃更记
得住公式或公式的 pattern。。。大家相互都觉得对方是反凑,但其实双方都是自然思
考。。。区别就是同一张图片,但不同的人看到的热点/侧重点不同。
这个也跟我自己过去的经验,和我家大娃在学校的经验,是吻合的。。。我觉得我们这
边很多 excellent-numeracy 甚至 supernumeracy,数感公式感好,记得住算得准。。
。而我和我娃平均水平的 normal numeracy 就容易被红叉追屁股,但最终到高中时,
intuition 把 numeracy 抵平一部分而找回点场子,最终大部分打成基本平手。。。也
属于合理。

:haha 题不好出的
n*******l
发帖数: 2911
307
反正你闲着也是闲着,算一下 18p+1 = n^3, p=?

【在 t******l 的大作中提到】
: 对付完小娃回来。。。我觉得上面确实说明因材施教的重要性。。。确实有人/娃更记
: 得住公式或公式的 pattern。。。大家相互都觉得对方是反凑,但其实双方都是自然思
: 考。。。区别就是同一张图片,但不同的人看到的热点/侧重点不同。
: 这个也跟我自己过去的经验,和我家大娃在学校的经验,是吻合的。。。我觉得我们这
: 边很多 excellent-numeracy 甚至 supernumeracy,数感公式感好,记得住算得准。。
: 。而我和我娃平均水平的 normal numeracy 就容易被红叉追屁股,但最终到高中时,
: intuition 把 numeracy 抵平一部分而找回点场子,最终大部分打成基本平手。。。也
: 属于合理。
:
: :haha 题不好出的

t******l
发帖数: 10908
308
这种本身都是 algebraic wishful thinking。。。照前面的做法就是先看一下 19*20
+ 1 = 381 是不是质数。

:反正你闲着也是闲着,算一下 18p+1 = n^3, p=?
t******l
发帖数: 10908
309
381 不是质数,因为能被 3 整除。。。得找其他的分解办法。

:这种本身都是 algebraic wishful thinking。。。照前面的做法就是先看一下 19*
20 + 1 = 381 是不是质数。
n*******l
发帖数: 2911
310
There is a reason why your method does not work this time.
还是以16p+1=n^3为例。纯粹从逻辑推理的完整性上来说,从
16p= n^3 - 1= (n-1) * (n^2 + n + 1)
不能直接推出n=17,因为右端项可能有其他的因数分解形式。
不过如果你注意到n^2+n+1是个奇数, 就能推导出出唯一的可
能性是n-1=16。
另外,对于一个题目,就算你找到了一个解,还有一个问题是解的唯
一性,有可能存在其他的解。一个题目的完整解答要么把所有解都找
出来,要么证明为什么除了你找到的那个之外不存在另外的解。

20

【在 t******l 的大作中提到】
: 这种本身都是 algebraic wishful thinking。。。照前面的做法就是先看一下 19*20
: + 1 = 381 是不是质数。
:
: :反正你闲着也是闲着,算一下 18p+1 = n^3, p=?

相关主题
[转载] 为什么说usamo 简单我招,我是猪
二年级女儿的report大学录取率
觉得美帝数学进度慢的进来看一下学校早晚要教的东西,早早学会了又如何?
进入Parenting版参与讨论
t******l
发帖数: 10908
311
你这个奇偶性判断是对的。。。我根本就没有去判断,猜完了就算了。。。反正是
AIME 填空题,快糙猛先。。。
更推广的说,根据 Unique Prime Factorization,对于 16p 问题,(n-1) 是 2*2*2*2
*p 的所有可能的因数里的某一个,一个个试过去即可。。。奇偶性在这题里面,我觉
得是 Unique Prime Factorization 的一种特例。

:There is a reason why your method does not work this time.
t******l
发帖数: 10908
312
所以同理,对于
18*p = n^3 - 1
18*p = (n-1)*(n^2 + n + 1)
根据 unique prime factorization,n - 1 必然是 2*3*3*p 的一个因数(质因数或
合数因数都可以),把所有因数组合一个一个试过去,如果有,就是解;如果没有,就
证明了无解。。。对于 18p 这个问题,奇偶性(特例)就用不上了,只能用基本的
unique prime factorization 理论(通用基本理论)了。
剩下的体力活我不干了。

:你这个奇偶性判断是对的。。。我根本就没有去判断,猜完了就算了。。。反正是
:AIME 填空题,快糙猛先。。。
t******l
发帖数: 10908
313
另外楼上哥们太狠了。。。这题本来就是个 AIME 快糙猛填空题,你们给生生掰成了
USAMO 证明题。。。我得想想是不是要穷追猛打继续教娃一步。。。
t******l
发帖数: 10908
314
根据上面的帖子,my method does work for all the time。。。只是要加上 unique
prime factorization。
或者简单的说个算法概念:
首先把比如 18*p,给prime factorization 后,重新组合成比如 u*(v*p) (u 和 v 是
已知整数)。组合理论遍历所有组合。
对于每一种 u*(v*p)。要么 n-1=u,要么 n^2 + n + 1 = u。。。如果整数解存在,则
把解出的 n 代入可解得 p,验证 p 是不是质数。
遍历所有组合,即得到所有解。计算一下时空复杂度,改追美剧 “美国高中小马工竞
赛 USACO”。。。// run

:There is a reason why your method does not work this time.
t******l
发帖数: 10908
315
或者这么说可能更合适一些。。。所谓的 “反凑”,并不是从无到有不建立在前人研
究及自己过去学习的基础上的全新发现,这是不可能的。。。而是把其看成一种有机的
信息压缩存储方式,占更少的大脑内存,也相对不容易忘记,更容易保持大脑存储信息
的内在一致性。
所以说是 “事后诸葛亮” 也并不错,但这个 “事后” 是指 “知道概念之后”, 而
不是指 “把对于公式完全记忆下来能够默写之后”。
另外这也可能取决于思维方式。。。supernumeracy 因为记忆力强大,对这种 “事后
诸葛亮” 的需求可能不高。。。但 normal numeracy 就可能需要这种 “事后诸葛亮
”,用 intuition 的能力来补偿 numeracy 的能力和记忆力。
所谓因材施教可能也需要考虑这些特点。

:你说的这些,基本都是“事后诸葛亮”。我觉得,如果一个学生知道x^3-1有一个根
:是x=1,从而x-1必然是一个因子,那么十有八九这个学生知道立方差公式。后面那
k**n
发帖数: 6198
316
你解释的我茅厕顿开。这么说我完全明白了。
俺娃是第三个解法。我前面就奇怪,她挥舞着手掌,从掌根处向我发力是啥意思。我完
全没理解她的降龙十八掌阿。
我想和她解释是第四种解法,就是找离10近的数。
可是反映到自己的思维,自己貌似是背出来的。所以我就是不知道,小孩子从解到背,
该需要多久。按照作业题的时间要求,我感觉不会背,规定的那么短的时间是做不完的。
所以,所有的前面的解法,还是为背打基础的。只要是理解以后的背,就没有任何关系。
重新解析小娃的思路,真是不容易啊

【在 f*******e 的大作中提到】
: 简单的说个教法吧:
: 9+5:
: -典型K解法:画9个圈再画5个圈,数一共多少
: -改进一点解法:画5个圈,从9开始往上数:10,11,12,13,14
: -快一点的解法: 脑子里,从9往上数: 10, 11, 12, 13, 14
: -make 10 (快速解法,需要背哪两个单词maket 10): 9+5 = 9 + 1 + 4 = 14
: - 纯记忆背法:9+5=14,

f*******e
发帖数: 3433
317
是这样的,理解完之后就教小孩快速计算方法,到最后题目做多了就都记在脑中了(被
动的背下来了)。如果叫小孩主动背,有些小孩不愿意背。而且教不同方法对心算速度
有帮助。比如说99+99,既可以用竖式加法,也可以转成(100-1)+(100-1)=200-2

还有19*19,如果转成19*(20-1)=380-19=370-9=361也会快点。所以我教得比较
慢,不仅仅是教如何做,而且教如何快速做。

的。
系。

【在 k**n 的大作中提到】
: 你解释的我茅厕顿开。这么说我完全明白了。
: 俺娃是第三个解法。我前面就奇怪,她挥舞着手掌,从掌根处向我发力是啥意思。我完
: 全没理解她的降龙十八掌阿。
: 我想和她解释是第四种解法,就是找离10近的数。
: 可是反映到自己的思维,自己貌似是背出来的。所以我就是不知道,小孩子从解到背,
: 该需要多久。按照作业题的时间要求,我感觉不会背,规定的那么短的时间是做不完的。
: 所以,所有的前面的解法,还是为背打基础的。只要是理解以后的背,就没有任何关系。
: 重新解析小娃的思路,真是不容易啊

s**********y
发帖数: 509
318
你这是登堂了。 你前面加法的帖子总结的很好, 这个说的也很对。 还有好多trick,
可以慢慢总结。
其实就是两条:1, 不备九九表, 2, 不列竖式计算。
算数就是用来培养number sense的。所以一不靠鸡翼, 二不靠procedure 。

-2

【在 f*******e 的大作中提到】
: 是这样的,理解完之后就教小孩快速计算方法,到最后题目做多了就都记在脑中了(被
: 动的背下来了)。如果叫小孩主动背,有些小孩不愿意背。而且教不同方法对心算速度
: 有帮助。比如说99+99,既可以用竖式加法,也可以转成(100-1)+(100-1)=200-2
: 。
: 还有19*19,如果转成19*(20-1)=380-19=370-9=361也会快点。所以我教得比较
: 慢,不仅仅是教如何做,而且教如何快速做。
:
: 的。
: 系。

t******l
发帖数: 10908
319
我看了前面帖子,我觉得如果娃不是 supernumeracy 的话(也就是反正将来计算不行
,MOEMS 奇偶性也就一般般),number sense 可以主要留给公立学校和课后班大路货
教就好了。。。我打算主攻 arithmetic-property sense,争取小娃能够直接跑步掌握
unique prime factorization。。。// run

:你这是登堂了。 你前面加法的帖子总结的很好, 这个说的也很对。 还有好多trick
, 可以慢慢总结。
t******l
发帖数: 10908
320
干了一下体力活,发现这 18p 问题确实有解。
n-1 = 2*3
n = 7
(n^2 + n + 1) / 3 = 57/3 = 19,是质数
所以 18p 问题的答案是 019。

所以同理,对于
18*p = n^3 - 1
18*p = (n-1)*(n^2 + n + 1)
根据 unique prime factorization,n - 1 必然是 2*3*3*p 的一个因数(质因数或
合数因数都可以),把所有因数组合一个一个试过去,如果有,就是解;如果没有,就
证明了无解。。。对于 18p 这个问题,奇偶性(特例)就用不上了,只能用基本的
unique prime factorization 理论(通用基本理论)了。
相关主题
amc 10 的A, B卷有啥区别转载: 从首个IMO季军谈起 by 付云皓
来点美好的时光新加坡数学对小学生帮助大吗?
有些小学四年级的数学题不太会做WISC-V 智商测试
进入Parenting版参与讨论
f*******e
发帖数: 3433
321
我之前和我大学同学说我在教小孩make 10做加法. 她说她小孩是奥数才学make 10, 国
内小学不教。

trick

【在 t******l 的大作中提到】
: 我看了前面帖子,我觉得如果娃不是 supernumeracy 的话(也就是反正将来计算不行
: ,MOEMS 奇偶性也就一般般),number sense 可以主要留给公立学校和课后班大路货
: 教就好了。。。我打算主攻 arithmetic-property sense,争取小娃能够直接跑步掌握
: unique prime factorization。。。// run
:
: :你这是登堂了。 你前面加法的帖子总结的很好, 这个说的也很对。 还有好多trick
: , 可以慢慢总结。

f*******e
发帖数: 3433
322
小娃读K,我也跟着读了一遍。希望他上高中时我不用跟着重读一遍高中。LOL

【在 s**********y 的大作中提到】
: 你这是登堂了。 你前面加法的帖子总结的很好, 这个说的也很对。 还有好多trick,
: 可以慢慢总结。
: 其实就是两条:1, 不备九九表, 2, 不列竖式计算。
: 算数就是用来培养number sense的。所以一不靠鸡翼, 二不靠procedure 。
:
: -2

t******l
发帖数: 10908
323
make 10 也看观察角度,make 10 也可以认为是 Piaget 老先生 Concrete Operation
里的 conserve (invariant)。
当然我目前的看法是直接教 unique prime factorization 一了百了,反正 unique
prime factorization 也是 conserve,因为是 invariant on factorization
structure。。。之前都可以直接跳过。。。// run

:我之前和我大学同学说我在教小孩make 10做加法. 她说她小孩是奥数才学make 10,
国内小学不教。
d******e
发帖数: 2265
324
x: x * x^ 6 + x * x^5 + ..... x* 1
1: - x^6 - ..... -x - 1
= x^7 - 1
这个不需要pattern,简单技巧而已。我记得当年公式都是现场推出。

【在 t******l 的大作中提到】
: 这 pattern 我确实不知道。。。我确实只知道能被 x-1 整除,剩下的要么做长除法发
: 现 pattern,要么指数移位抵消反凑。
: 这个的确说明每个人的思路差别很大,因材施教才是正道。。。所以父母自己解题可能
: 还是挺重要的,毕竟有相当的相似性。
:
: :x^7-1 = (x-1)(x^6+x^5+...+x+1)

t******l
发帖数: 10908
325
如果不是记忆 pattern 的话,那你说的跟我前面说的 “指数移位抵消反凑” 的 “
algebraic telescope tool” (Zeitz 在 ACOPS 一书里的用词),就是同一回事吧。

:x: x * x^ 6 + x * x^5 + ..... x* 1
:1: - x^6 - ..... -x - 1
k**n
发帖数: 6198
326
你好牛。我家去年k我完全没管,就担心怎么给她playdate了。今年貌似交朋友好多了
, 这才留心学校的教学大纲。 将来我没准备全都读一遍,就准备以后把english and
social study和她过一遍。
昨天问了一年级娃,你们数学在教什么?她说老师在教五。
每天回来屁颠屁颠的就是说她们老师玩science.就是厨房里的那些玩艺,然后信誓旦旦
要当marine scientist 或者zoo scientist. 俺这不打击积极性吗,我告诉她咱周围只
有一个动物园, 如果你们全班都要当zoo scientists,没那么多工作给你们的.更何况
你们全年级还有那么多班:))

【在 f*******e 的大作中提到】
: 小娃读K,我也跟着读了一遍。希望他上高中时我不用跟着重读一遍高中。LOL
t******l
发帖数: 10908
327
sorry 我刚才水车回帖没看 id,刚才把你跟前面的另一个 id 混淆了。。。//

:x: x * x^ 6 + x * x^5 + ..... x* 1
:1: - x^6 - ..... -x - 1
t******l
发帖数: 10908
328
我家大娃在四年级以前我没咋管。。。其实也无所谓,最多就是四年级的时候忙一点。
。。

:你好牛。我家去年k我完全没管,就担心怎么给她playdate了。今年貌似交朋友好多了
:, 这才留心学校的教学大纲。 将来我没准备全都读一遍,就准备以后把english
and social study和她过一遍。
f*******e
发帖数: 3433
329
我的重点也是social, 所以每天数学最多教他6-7分钟:-) 他summer camp那6个星期我
一点数学都没教, 就指望着他和summer camp小朋友一起玩。暑假坚持每天读一本小娃
的故事书,问他问题,目的在于提高他听力理解。summer camp结束后发现他数学开始
生疏了,才又开始每天6-7分钟巩固他以前学的或者教他新的。我之所以琢磨教学方法
,是希望达到事半功倍的效果,这样娃也好教点,学得轻松点,不至于磨灭对数学兴趣
。而且学这些对我心算也有帮助:-P

and

【在 k**n 的大作中提到】
: 你好牛。我家去年k我完全没管,就担心怎么给她playdate了。今年貌似交朋友好多了
: , 这才留心学校的教学大纲。 将来我没准备全都读一遍,就准备以后把english and
: social study和她过一遍。
: 昨天问了一年级娃,你们数学在教什么?她说老师在教五。
: 每天回来屁颠屁颠的就是说她们老师玩science.就是厨房里的那些玩艺,然后信誓旦旦
: 要当marine scientist 或者zoo scientist. 俺这不打击积极性吗,我告诉她咱周围只
: 有一个动物园, 如果你们全班都要当zoo scientists,没那么多工作给你们的.更何况
: 你们全年级还有那么多班:))

t******l
发帖数: 10908
330
你可以用 AIME 十五道心算填空题来练心算。// run

:我的重点也是social, 所以每天数学最多教他6-7分钟:-) 他summer camp那6个星期我
:一点数学都没教, 就指望着他和summer camp小朋友一起玩。暑假坚持每天读一本小娃
相关主题
今天和幼儿园老师开会review娃的progress,非常upsetExplaining Your Math: Unnecessary at Best, Encumbering at W
好学生去普通公立高中真的会被耽误吗?是不是任我儿子继续迷象棋
陶天才论天才 (转载)9岁, 如何报名考amc10
进入Parenting版参与讨论
f**********g
发帖数: 4709
331
计算器也很好玩的,我家丫头也能玩很久很久。
不过赞同你的分类,玩计算器需要自己动脑筋思考,主动探索验证,跟电脑游戏的被动
接受确实不一样

HGT的

【在 t******l 的大作中提到】
: 计算器,平板电脑 chess,娃版电脑游戏,这三个对专注力本来就不能相提并论。。。
: 或者说在这三者之间的相对专注力的大小,本身可能正常范围 vs 超出正常范围的指标
: 之一。
:
: :
: :那些研究教育,心理,自闭多动的应该已经把专注力量化了吧。所以才有结论,HGT的

t******l
发帖数: 10908
332
说的也是,普通娃就是最喜欢玩不动脑子的电脑游戏,我家小娃就喜欢 temple run。
。。chess 现在得我陪着才能玩,计算器一般小娃很少玩。。。当然小娃花时间最多的
还是看 PBS KIDS VIDEO,倒不是玩游戏。
所以普通娃的专注兴趣跟 super number-sensr / supernumeracy 和 hypernumeracy
非常不一样,也是事实。

:计算器也很好玩的,我家丫头也能玩很久很久。
:不过赞同你的分类,玩计算器需要自己动脑筋思考,主动探索验证,跟电脑游戏的被
动接受确实不一样
f**********g
发帖数: 4709
333
这些个东东,难道不是让娃自己学了想出来的。你都这么一条条给她分析了还有什么意思

【在 t******l 的大作中提到】
: 比如另一个解法里,需要用到 x^3 - 1 = (x-1) *(x^2 + x + 1)。。。这公式娃不要
: 说背不出来,连听都没听说过(play with algebraic cubes)。。。“急用” 了不是
: ?。。。
: 所以就先观察该多项式对应的方程 x^3 - 1 = 0,可知有一个根为 x=1。。。根据多项
: 式理论,该多项式必然能被多项式 x-1 整除。。。那剩下的咋办?。。。又急用了不
: 是?。。。办法一,多项式长除法,正好复习一遍。。。这长除法做完了,但是好像有
: 点烦。。。OK 还有诡异的办法二:利用指数的 “移位对称性”(algebraic
: telescope tool) 凑正负抵消,反凑一个多项式 x^2 + x + 1 出来。
: 这样通过 “急用”,把枯燥变得稍微有趣一些,降低 normalnumeracy 娃学数学的最
: 大阻力 -- 枯燥。

t******l
发帖数: 10908
334
你这个似乎又一个支持小学数学牛蛙一定是 number sense / numeracy 方面的牛蛙。
高中数学先进蛙的话,我目测可能有一小部分是以 arithmetic-property sense 见长
的,number sense 一般,用 propertiracy 来弥补 numeracy。。。而这一部分可能就
是大伙儿看到的屌丝逆袭型。

:计算器也很好玩的,我家丫头也能玩很久很久。
:不过赞同你的分类,玩计算器需要自己动脑筋思考,主动探索验证,跟电脑游戏的被
动接受确实不一样
f**********g
发帖数: 4709
335
我怎么觉得中学背的出,大学背不出挺正常的啊
中学的时候正在用嘛,也不用专门背,熟练了自然可以直接写出来,用处嘛其实也就是
解题可以快一点儿
大学了,理解了背后的基础,知道的更多了,背不出来可以推导出来,可以计算器算出
来,也可以写程序run出来,反正如果需要知道答案可以有很多办法得到
什么年龄就该做什么事儿,拔苗助长要不得

【在 t******l 的大作中提到】
: 你这么说也对。。。但如果目的不是大学入学高分,而是人生相对轻松的话。。。那中
: 学背得出,大学背不出,其实反而不好。
: 另外 x^7 - 1 总是背不出来的。。。当然其实数学竞赛那波确实背诵很多分解和级数
: 求和公式。。。这也算很多人放弃数学竞技,把数学竞赛就是当初学数学的工具的原因
: 之一。。。大部分普通娃确实不是 supernumeracy,对背公式兴趣不大。
:
: :你背不出那是因为你已经不是中学生了。

t******l
发帖数: 10908
336
一条条分析确实没什么意思,但是能推个混州立。。。总比 pushy mama 微信群的 “
鸡兔同笼三十六计招招制敌” 要好上一万倍。。。因为那 “鸡兔同笼三十六计” 对
混州立
我觉得没啥帮助,对卖鸡卖兔子的小贩倒是帮助很大!!

:这些个东东,难道不是让娃自己学了想出来的。你都这么一条条给她分析了还有什么
意思
f*******e
发帖数: 3433
337
推个州立还是很容易的吧,不用AIME的。:-P

【在 t******l 的大作中提到】
: 一条条分析确实没什么意思,但是能推个混州立。。。总比 pushy mama 微信群的 “
: 鸡兔同笼三十六计招招制敌” 要好上一万倍。。。因为那 “鸡兔同笼三十六计” 对
: 混州立
: 我觉得没啥帮助,对卖鸡卖兔子的小贩倒是帮助很大!!
:
: :这些个东东,难道不是让娃自己学了想出来的。你都这么一条条给她分析了还有什么
: 意思

t******l
发帖数: 10908
338
因为你记忆力强,所以背得出来。。。我高中就不喜欢背公式,能不背就不背。
这就好比要让我娃学体操的话,她绝对不肯干。。。你娃就相反不是?
因材施教还是需要的,拔苗当然不可取。

:我怎么觉得中学背的出,大学背不出挺正常的啊
t******l
发帖数: 10908
339
我没推 AIME,我就找道简单的 AIME 前一两题当例题教数学而已,跟看数学书没区别
。。。而且这种对你们就是送分题。
另外你娃要是 GPA 有 C,你总是会长板补一下短板。

:推个州立还是很容易的吧,不用AIME的。:-P
f**********g
发帖数: 4709
340
就是就是,我都说了好几次了,感觉自己祥林嫂了都
不过反正潮水闲着也是闲着,貌似他家姑娘也挺闲的,做点儿数学题玩玩也挺好的。
我都羡慕死了,今天刚拿到丫头的shedule,目测连写作业的时间都不够了……

【在 f*******e 的大作中提到】
: 推个州立还是很容易的吧,不用AIME的。:-P
相关主题
9岁, 如何报名考amc10二年级女儿的report
有明天考AMC 8的吗?觉得美帝数学进度慢的进来看一下
[转载] 为什么说usamo 简单我招,我是猪
进入Parenting版参与讨论
s**********y
发帖数: 509
341
赞耐心。 高中你想更,也更不上, 娃也不让你更。 哈哈

【在 f*******e 的大作中提到】
: 小娃读K,我也跟着读了一遍。希望他上高中时我不用跟着重读一遍高中。LOL
s**********y
发帖数: 509
342
你强。 貌似 unique prime factorization 对以后影响不大哈

trick

【在 t******l 的大作中提到】
: 我看了前面帖子,我觉得如果娃不是 supernumeracy 的话(也就是反正将来计算不行
: ,MOEMS 奇偶性也就一般般),number sense 可以主要留给公立学校和课后班大路货
: 教就好了。。。我打算主攻 arithmetic-property sense,争取小娃能够直接跑步掌握
: unique prime factorization。。。// run
:
: :你这是登堂了。 你前面加法的帖子总结的很好, 这个说的也很对。 还有好多trick
: , 可以慢慢总结。

t******l
发帖数: 10908
343
我觉得初中娃就不行了,早就开始嫌父母烦了。。。所以我现在给娃讲数学,一般都用
那种看上去特简单,但娃一想就直接 totally clueless 的那种,这样首先能镇住娃。
。。而 AIME 送分题的好处也是送分题也给顶着个 AIME 的高帽子,首先能唬住娃坐下
来听数学理论。。。

:赞耐心。 高中你想更,也更不上, 娃也不让你更。 哈哈
s**********y
发帖数: 509
344
经验之谈, 不可不听哈

【在 t******l 的大作中提到】
: 我觉得初中娃就不行了,早就开始嫌父母烦了。。。所以我现在给娃讲数学,一般都用
: 那种看上去特简单,但娃一想就直接 totally clueless 的那种,这样首先能镇住娃。
: 。。而 AIME 送分题的好处也是送分题也给顶着个 AIME 的高帽子,首先能唬住娃坐下
: 来听数学理论。。。
:
: :赞耐心。 高中你想更,也更不上, 娃也不让你更。 哈哈

t******l
发帖数: 10908
345
应该是整套完整的 arithmetic property。。。单个 property 没办法单干确实是真的。
http://www.coolmath.com/prealgebra/06-properties


:你强。 貌似 unique prime factorization 对以后影响不大哈
n****f
发帖数: 3580
346
我们这SWC (stong-willed child), 还没上小学就不行了,不可能强迫他干任何自己没
兴趣的事。 5岁这一年,不肯上任何课外班, 除了在学校after school里上了两个,
art和吉他,都是打酱油的那种。
也好,也不用想着怎么推了。

【在 t******l 的大作中提到】
: 我觉得初中娃就不行了,早就开始嫌父母烦了。。。所以我现在给娃讲数学,一般都用
: 那种看上去特简单,但娃一想就直接 totally clueless 的那种,这样首先能镇住娃。
: 。。而 AIME 送分题的好处也是送分题也给顶着个 AIME 的高帽子,首先能唬住娃坐下
: 来听数学理论。。。
:
: :赞耐心。 高中你想更,也更不上, 娃也不让你更。 哈哈

t******l
发帖数: 10908
347
另外其实这里面很不全,只有基本的从算术到 algebra 1 代数的 properties。。。离
散的,几何的,排列组合的,等等等等,都没有。

:应该是整套完整的 arithmetic property。。。单个 property 没办法单干确实是真
的。
k**n
发帖数: 6198
348
娃不理咱,咱自娱自乐不行吗?我看看他们的social study,实话说很多是我想补课的
。 尤其是reading,最近在补,可是真的已经过了初中高中的记忆方式了。
不管怎么加大reading,直接上来是收获作者思维,和批判或接受其观点,没法记住作
者怎样遣词造句。除了写得特别差的有感觉

【在 t******l 的大作中提到】
: 我觉得初中娃就不行了,早就开始嫌父母烦了。。。所以我现在给娃讲数学,一般都用
: 那种看上去特简单,但娃一想就直接 totally clueless 的那种,这样首先能镇住娃。
: 。。而 AIME 送分题的好处也是送分题也给顶着个 AIME 的高帽子,首先能唬住娃坐下
: 来听数学理论。。。
:
: :赞耐心。 高中你想更,也更不上, 娃也不让你更。 哈哈

t******l
发帖数: 10908
349
反正再推几年数学就没的推了,我先保证把娃推进州大再说别的。。。实话实说。。。

:娃不理咱,咱自娱自乐不行吗?我看看他们的social study,实话说很多是我想补课
的。 尤其是reading,最近在补,可是真的已经过了初中高中的记忆方式了。
n****f
发帖数: 3580
350
加州竞争真的很激烈吗?
其他州没听说推娃进州大的, 不是自动进的,就是被动进的。

【在 t******l 的大作中提到】
: 反正再推几年数学就没的推了,我先保证把娃推进州大再说别的。。。实话实说。。。
:
: :娃不理咱,咱自娱自乐不行吗?我看看他们的social study,实话说很多是我想补课
: 的。 尤其是reading,最近在补,可是真的已经过了初中高中的记忆方式了。

相关主题
大学录取率来点美好的时光
学校早晚要教的东西,早早学会了又如何?有些小学四年级的数学题不太会做
amc 10 的A, B卷有啥区别转载: 从首个IMO季军谈起 by 付云皓
进入Parenting版参与讨论
t******l
发帖数: 10908
351
我娃 GPA 一般 B 比 A 多,偶尔有 C。。。她的朋友们基本以 A 为主,偶尔有 B。
当然我现在也不推娃拿 A,在 numeracy 和 literacy 方面,比不过那些娃,我觉得是
先天差别。。。所以我现在唯一的办法就是推点 propertiracy,虽然也不期望将来高
中能搞个平手,但至少能找回点场子排在队伍后边进州大。

:加州竞争真的很激烈吗?
k**n
发帖数: 6198
352
的确。
所以老加州移民,主要是娃娃都大学毕业了的,支持aa,新移民,娃娃没上大学的,都
反aa。
我一朋友从北卡搬去加州,告诉我他娃还在北卡,闭着眼睛上unc, 现在,光着膀子准
备去外州念州大。
不知道我解释清楚了没有。

【在 n****f 的大作中提到】
: 加州竞争真的很激烈吗?
: 其他州没听说推娃进州大的, 不是自动进的,就是被动进的。

s**********y
发帖数: 509
353
他要进博客来, 你也自动进/被动进?

【在 n****f 的大作中提到】
: 加州竞争真的很激烈吗?
: 其他州没听说推娃进州大的, 不是自动进的,就是被动进的。

f*******e
发帖数: 3433
354
UC难进一点。CalState应该好进吧。

【在 k**n 的大作中提到】
: 的确。
: 所以老加州移民,主要是娃娃都大学毕业了的,支持aa,新移民,娃娃没上大学的,都
: 反aa。
: 我一朋友从北卡搬去加州,告诉我他娃还在北卡,闭着眼睛上unc, 现在,光着膀子准
: 备去外州念州大。
: 不知道我解释清楚了没有。

t******l
发帖数: 10908
355
加州的 “推州大” 的意思,进 UCB / UCLA 属于中大奖,运气。。。当然能轻松读下
来就不怕,反正学费没差别。。。苦鼻的就算了,听说那两所里的非坑爹专业对 GPA
那可不是一般的抠门。坊间流传属于累S人都不带眨眼的那种。。。没办法,州大要是
不搞两只小鸡敬敬猴,阿猫阿狗都硬挤的话,招生办也很累的。。。


:他要进博客来, 你也自动进/被动进?
n****f
发帖数: 3580
356
他的目标是当码工吧

【在 s**********y 的大作中提到】
: 他要进博客来, 你也自动进/被动进?
n****f
发帖数: 3580
357
进alState学计算机出来,据说也能找到工作, 近水楼台先得月,其实也是条路。
不过推孩子上好点的学校,也是人之常情。

【在 f*******e 的大作中提到】
: UC难进一点。CalState应该好进吧。
t******l
发帖数: 10908
358
我觉得这确实是个例子,回答以前黄小木同学问的 “从抽象直接到抽象” 的问题。
我觉得至少对于俺,不存在 “从抽象直接到抽象” 的情况。。。黄小木同学以前看到
的差别,可能是 numeracy vs propertiracy 的差别。。。具体可能是这样,比如对于
这题(硬掰到 USAMO 证明题)的两个变形虫:
16*p + 1 = n^3 或者 18*P + 1 = n^3
numeracy 最先看到的是 16 和 18 是偶数,1 是奇数,n^3 的奇偶性,然后顺藤摸瓜。
<先发出来,省得万一白写>

【在 t******l 的大作中提到】
: 干了一下体力活,发现这 18p 问题确实有解。
: n-1 = 2*3
: n = 7
: (n^2 + n + 1) / 3 = 57/3 = 19,是质数
: 所以 18p 问题的答案是 019。
:
: 所以同理,对于
: 18*p = n^3 - 1
: 18*p = (n-1)*(n^2 + n + 1)
: 根据 unique prime factorization,n - 1 必然是 2*3*3*p 的一个因数(质因数或

f*******e
发帖数: 3433
359
查了一下公立大学usnews排名. 前50 加州就占了7个,有的州还没有前50州立。所以上
UC不应该难啊。 PS。我认得的那个华裔的精神出了问题的弟弟也是UC读书的。
1. UCB
2. UCLA
3. U virginia
4. U michigan
5. U north carolina
6. william mary
7. GIT
8. UCSB
9. UCI
10. UCSD
11. UCD
12. UIUC
13. U wisconsin
14. Penn state
15 U florida
16 Ohio state
17 texas austin
18 U washington
19 U connecticut
20 U maryland
21 Clemson
22 Purdue
23 U Georgia
24 U pittsburg
25 U Minnesota
26 texas AM
27 Virgina tech
28 Rutgers
29 Colorado school of mine
30 Indiana U
31 Michigan State
32 U delaware
33 M mass
34 Miami
35 UCSC
36 U Iowa
37 SUNY Binghamton
38 North Carolina
39 SUNY Stony Brook
40 SUNY college of environment ...
41 U colorado
42 U Vermont
43 Florida state
44 U alabama
45 SUNY Buffalo
46 Auburn
47 U Missouri
48 U Nebraska
49 U New Hampshire
50 U Oregon
t******l
发帖数: 10908
360
(接上文)。。。我觉得 propertiracy 首先第一步看到的其实是(这里不是说
propertiracy 其他的不看,而是大脑首先要对输入信息剪枝。这个剪枝过程实际上也
决定了 wishful thinking 的先后次序):
c*p + 1 = n^3
其中 c 是已知整数,p 是未知素数,n 是未知整数。(也就是结构和属性,这里的属
性是 已知 vs 未知,整数 vs 素数)。
(BTW: 这里并不是说 propertiracy 的大脑会把已知整数换成字母去思考,这是给自
己找麻烦。。。我这么写的意思是,我觉得 propertiracy 可能在大脑里 attach 一个
property,比如说 “已知” 这个 property,在那个比如 16 或 18 的数字图景上,
从而影响下一步 wishful thinking 对图景/结构进行剪枝的次序。。。当然人不可能
对自己大脑进行 scan,我这么写下来的是 reasonable guess 大脑的 information
process)。
propertiracy 第二步是意识到这是一个 factorization problem(factorization 也
是 properties 的一种,invariant (on structure),其实就是 Piaget 水杯
conservation task),于是把 1 移动到右边:
c*p = n^3 - 1
然后 propertiracy 首先注意到右边,wishful thinking 顺着 factorization 的
instinct 摸瓜,在等式右边尝试 polynomial factorization w/ integer solution,
得到:
c*p = (n-1)*(n^2 + n + 1)
如果是 AIME 填空题,快糙猛直接猜一个,猜中就收拾书包走人做下一题。。。但如果
不幸没猜中,或者不幸考试一半老师进来宣布临时把 AIME 心算填空题给改成 USAMO
证明作文题。。。那样的话 propertiracy 就把注意力移动到等式左边,因为左边的 c
是已知整数,p 是未知素数(虽然未知但不可分割),而这个大问题是
factorization 问题,所以就 wishful thinking 在左边尝试 进行 prime
factorization,得到:
( cp_1 * cp_2 * ... * cp_k ) * p = (n-1)*(n^2 + n + 1)
[ 上面的 cp_1 cp_2 ... cp_k 指已知素数(已知且不可分割)]
然后观察左右两边和等号,根据 unique prime factorization 和 commutative and
associative property of multiplication,然后 wishful thinking 一下觉得应该试
试 combinatorics 寻找所有的 partition 方式。。。这时把数字放进去,得到:
[16p 题目] : 2*2*2*2*p = (n-1)*(n^2 + n + 1)
[18p 题目] : 2*3*3*p = (n-1)*(n^2 + n + 1)
当然,实践上 propertiracy 对于常数,脑子里其实是直接用数字的写法和图景,而不
会用 ( cp_1 * cp_2 * ... * cp_k ) 来自己给自己找麻烦。。。但这时虽然是实际数
字,但因为大脑把实际数字挂钩在 unique prime factorization 这个 property 上,
所以我觉得在剪枝次序上,奇偶性变成第二位的时,严重时可能会视而不见。
当然,这也会有另一个问题,就像 AlphaGo 那种下棋基本靠蒙的,总是有很大的猜出
臭子的可能,或者半当中直接卡壳的可能。。。所以同样的环境下,正确率会比
numeracy 要低很多。

瓜。

【在 t******l 的大作中提到】
: 我觉得这确实是个例子,回答以前黄小木同学问的 “从抽象直接到抽象” 的问题。
: 我觉得至少对于俺,不存在 “从抽象直接到抽象” 的情况。。。黄小木同学以前看到
: 的差别,可能是 numeracy vs propertiracy 的差别。。。具体可能是这样,比如对于
: 这题(硬掰到 USAMO 证明题)的两个变形虫:
: 16*p + 1 = n^3 或者 18*P + 1 = n^3
: numeracy 最先看到的是 16 和 18 是偶数,1 是奇数,n^3 的奇偶性,然后顺藤摸瓜。
: <先发出来,省得万一白写>

相关主题
新加坡数学对小学生帮助大吗?好学生去普通公立高中真的会被耽误吗?
WISC-V 智商测试陶天才论天才 (转载)
今天和幼儿园老师开会review娃的progress,非常upsetExplaining Your Math: Unnecessary at Best, Encumbering at W
进入Parenting版参与讨论
t******l
发帖数: 10908
361
BTW: 我写这段的目的是讨论因菜施浇,因为我自己小学数学被红叉追屁股的经历。。。

【在 t******l 的大作中提到】
: (接上文)。。。我觉得 propertiracy 首先第一步看到的其实是(这里不是说
: propertiracy 其他的不看,而是大脑首先要对输入信息剪枝。这个剪枝过程实际上也
: 决定了 wishful thinking 的先后次序):
: c*p + 1 = n^3
: 其中 c 是已知整数,p 是未知素数,n 是未知整数。(也就是结构和属性,这里的属
: 性是 已知 vs 未知,整数 vs 素数)。
: (BTW: 这里并不是说 propertiracy 的大脑会把已知整数换成字母去思考,这是给自
: 己找麻烦。。。我这么写的意思是,我觉得 propertiracy 可能在大脑里 attach 一个
: property,比如说 “已知” 这个 property,在那个比如 16 或 18 的数字图景上,
: 从而影响下一步 wishful thinking 对图景/结构进行剪枝的次序。。。当然人不可能

s**********y
发帖数: 509
362
我看这楼映像最深的是“事后猪哥亮”一词。 愣了半天, 过气的猪哥亮何时变得如此
有名?

。。

【在 t******l 的大作中提到】
: BTW: 我写这段的目的是讨论因菜施浇,因为我自己小学数学被红叉追屁股的经历。。。
t******l
发帖数: 10908
363
其实我没有看过任何猪哥亮的节目,就记得以前曾瞥过一眼猪哥亮的镜头,记得那个好
像是西装短裤的样子?还有说到躲债还是躲黑老大人间蒸发了好久?

:我看这楼映像最深的是“事后猪哥亮”一词。 愣了半天, 过气的猪哥亮何时变得如
此有名?
t******l
发帖数: 10908
364
我刚才想了一下,对于这道题的上下文是整数/素数问题的话,guess an (integer)
root 应该是正确的 instinct,如果等式右边出现一个一般的高次多项式,不是这题简
单的立方差或多少方差。
详细点说,这个问题是右边不管是 x^3 - 1 还是 x^7 - 1 都是特殊情况。。。而对于
更一般的情况,也就是右边如果是一个一般的任意高次的多项式,通常的套公式因式分
解就可能根本没有公式可套,高次方程也无一般求根公式。
但这题前面的解法中,在等式右边的策略是多项式整数因式分解,而我们有 rational
root theorem 可以保证找出任意高次方程所有的有理数根。这样就可以从所有有理数
根里选出所有的整数根,然后多项式除法求余下的部分,然后用 unique prime
factorization,加上集合/组合理论遍历所有组合。
这样等式右边是任意的多项式都可至少有降低复杂度求解的办法,而不仅仅限制在立方
差因式分解。
所以从这个角度看,guess a root 并不是 “事后猪哥亮”,而死套公式才保证随着题
目难度上升而最终会达到 “事前猪一样” 的境界。。。// run

:你说的这些,基本都是“事后诸葛亮”。我觉得,如果一个学生知道x^3-1有一个根
:是x=1,从而x-1必然是一个因子,那么十有八九这个学生知道立方差公式。后面那
t******l
发帖数: 10908
365
详细一点说明上面的帖子,就是如果进一步讨论任意多项式的情况,也就是第二步的时
候改成:
c*p = Q(n)
其中:c 是已知整数;p 是未知素数;n 是未知整数,Q() 是已知最高次项系数为 1
的多项式。
那么使用 rational root theorem (这里是 integer root),只要 Q(x) 存在至少一个
整数根 a,那么 Q(x) 就可以整数因式分解成 (x-a)*S(x),(分解更多降低计算复杂
度,但只要能分解成两个,就能保证能用下面的办法解得出来解),这样式子就变成:
c*p = (n-a)*S(n)
其中:c 是已知整数;p 是未知素数;n 是未知整数,a 是已知整数,S() 是已知最高
次项系数为 1 的多项式。
然后如法炮制对左边进行 prime factorization:
( cp_1 * cp_2 * ... * cp_k ) * p = (n-a)*S(n)
[ 上面的 cp_1 cp_2 ... cp_k 指已知素数(已知且不可分割)]
因为未知素数只有一个,该未知素数不可分割。。。依据 unique prime
factorization,所以可以把左边 bi-partition 成 u*(v*p) 时,未知素数只会存在一
边,总存在一边都是已知数(抽屉原理最最简单的情况)。。。所以从 bi-partition
的已知数 u 的那一边,得知要么 u=n-a,要么 u=S(n),用 rational root theorem
可以解出所有整数根 n,然后解出来的 n 代入剩下的那一边,算出 p,然后验证 p 是
不是素数。。。遍历所有 bi-partition 就可以找到所有解,或者无解。当然不要忘了
特殊的 partition 1*(c*p) (multiplicative identity 的特例情况)。
所以从以上可知,只要用 rational root theorem 证明 Q(x) 存在至少一个整数根,就
可以用以上办法找出所有的素数 p,或者证明不存在该素数。。。当然如果 Q(x) 不存
在任何整数根,那就比较麻烦。。。

【在 t******l 的大作中提到】
: (接上文)。。。我觉得 propertiracy 首先第一步看到的其实是(这里不是说
: propertiracy 其他的不看,而是大脑首先要对输入信息剪枝。这个剪枝过程实际上也
: 决定了 wishful thinking 的先后次序):
: c*p + 1 = n^3
: 其中 c 是已知整数,p 是未知素数,n 是未知整数。(也就是结构和属性,这里的属
: 性是 已知 vs 未知,整数 vs 素数)。
: (BTW: 这里并不是说 propertiracy 的大脑会把已知整数换成字母去思考,这是给自
: 己找麻烦。。。我这么写的意思是,我觉得 propertiracy 可能在大脑里 attach 一个
: property,比如说 “已知” 这个 property,在那个比如 16 或 18 的数字图景上,
: 从而影响下一步 wishful thinking 对图景/结构进行剪枝的次序。。。当然人不可能

l*****c
发帖数: 1153
366
奥赛当然拿奖了,当然后来大学以后没有专攻数学。这种天赋我觉得不是说以后一定要
搞数学作为一生的职业。关键是锻炼思维模式,是个方法论和世界观的问题。

【在 f*******e 的大作中提到】
: 那你长大后数学一直名列前茅吗?数学奥赛拿奖了吗?问这些就是想知道那些小时候是
: “天才”的长大后怎样了。

l*****c
发帖数: 1153
367
这事情我小时候特别有印象。比如5+9吧,我记得当时我就知道应该是14,但是我不知
道怎么表示14. 忘了谁跟我说了两位数的写法,那就o了,再往后都可以自己推导了。

【在 f*******e 的大作中提到】
: 简单的说个教法吧:
: 9+5:
: -典型K解法:画9个圈再画5个圈,数一共多少
: -改进一点解法:画5个圈,从9开始往上数:10,11,12,13,14
: -快一点的解法: 脑子里,从9往上数: 10, 11, 12, 13, 14
: -make 10 (快速解法,需要背哪两个单词maket 10): 9+5 = 9 + 1 + 4 = 14
: - 纯记忆背法:9+5=14,

l*****c
发帖数: 1153
368
其实大部分家长从来就没有理解过什么是数学,所以最好不要去“教”小孩数学。 数
学是自然科学的哲学,这句话不是说说的。 算术本身只是一种基础的数学工具而已,
家长教一些算术的技术和开发小孩的数学能力完全是两码事。数学是思维,家长可以为
小孩创造条件,但是绝大多数人根本教不了。

【在 s**********y 的大作中提到】
: 点赞。 听起来比几岁会什么靠谱啊。
t******l
发帖数: 10908
369
赞同你说的 “关键是锻炼思维模式,是个方法论和世界观的问题。”。
大部分不走理论数学专业的,死记的数学公式真心用处不大,特别是当今 GooG + wiki
+ iPhone 10 一键三击搜索出黎曼猜想证明的信息社会。
而你说的正确的“思维模式”,干 STEM 任何行当都有用。。。第一步思维模式错了,
直接浪费掉仨月其实太容易了。。。当然了,小保芳的生物专业,不好说。。。// run

:奥赛当然拿奖了,当然后来大学以后没有专攻数学。这种天赋我觉得不是说以后一定
要搞数学作为一生的职业。关键是锻炼思维模式,是个方法论和世界观的问题。
P******e
发帖数: 1325
370
她的意思是问你有没有拿IMO金牌?
天分这么高,至少也得进个集训队吧?

【在 l*****c 的大作中提到】
: 奥赛当然拿奖了,当然后来大学以后没有专攻数学。这种天赋我觉得不是说以后一定要
: 搞数学作为一生的职业。关键是锻炼思维模式,是个方法论和世界观的问题。

相关主题
Explaining Your Math: Unnecessary at Best, Encumbering at W有明天考AMC 8的吗?
是不是任我儿子继续迷象棋[转载] 为什么说usamo 简单
9岁, 如何报名考amc10二年级女儿的report
进入Parenting版参与讨论
P******e
发帖数: 1325
371
你觉得现在的common core那些东西,比家长自己“教”的数学更能开发数学能力么?

【在 l*****c 的大作中提到】
: 其实大部分家长从来就没有理解过什么是数学,所以最好不要去“教”小孩数学。 数
: 学是自然科学的哲学,这句话不是说说的。 算术本身只是一种基础的数学工具而已,
: 家长教一些算术的技术和开发小孩的数学能力完全是两码事。数学是思维,家长可以为
: 小孩创造条件,但是绝大多数人根本教不了。

l*****c
发帖数: 1153
372
yeah, 没拿金牌。不过其实好玩的思维方法,不是死记硬背。这个真的各位家长如果不
懂数学的,最好不要以自己的想法去教小孩。

【在 P******e 的大作中提到】
: 她的意思是问你有没有拿IMO金牌?
: 天分这么高,至少也得进个集训队吧?

l*****c
发帖数: 1153
373
没研究过美国的数学教育啊。不过我觉得,能让小孩体会到美,激发热情的教学方法就
是好的方法,不管是数学还是别的学科。为了得奖去学习,去参加比赛就算了。

【在 P******e 的大作中提到】
: 你觉得现在的common core那些东西,比家长自己“教”的数学更能开发数学能力么?
P******e
发帖数: 1325
374
理论数学专业的,也不用死记数学公式。听说都是需要用的时候自己推一推。

wiki
run

【在 t******l 的大作中提到】
: 赞同你说的 “关键是锻炼思维模式,是个方法论和世界观的问题。”。
: 大部分不走理论数学专业的,死记的数学公式真心用处不大,特别是当今 GooG + wiki
: + iPhone 10 一键三击搜索出黎曼猜想证明的信息社会。
: 而你说的正确的“思维模式”,干 STEM 任何行当都有用。。。第一步思维模式错了,
: 直接浪费掉仨月其实太容易了。。。当然了,小保芳的生物专业,不好说。。。// run
:
: :奥赛当然拿奖了,当然后来大学以后没有专攻数学。这种天赋我觉得不是说以后一定
: 要搞数学作为一生的职业。关键是锻炼思维模式,是个方法论和世界观的问题。

P******e
发帖数: 1325
375
照你这个说法,lz的数学教的挺好的,人家的娃不是很有学习热情么?

【在 l*****c 的大作中提到】
: 没研究过美国的数学教育啊。不过我觉得,能让小孩体会到美,激发热情的教学方法就
: 是好的方法,不管是数学还是别的学科。为了得奖去学习,去参加比赛就算了。

t******l
发帖数: 10908
376
数学“美”对普通娃没用吧,普通娃学数学本质上是为了挣支票我觉得。

:没研究过美国的数学教育啊。不过我觉得,能让小孩体会到美,激发热情的教学方法
就是好的方法,不管是数学还是别的学科。为了得奖去学习,去参加比赛就算了。
s**********y
发帖数: 509
377
教育学当然是一门科学。 举例来说, 现在一些州的test, 可以produce a
personalized report to indicate the weaknesses of a kid, 而不是一个简单的分
数。

【在 l*****c 的大作中提到】
: yeah, 没拿金牌。不过其实好玩的思维方法,不是死记硬背。这个真的各位家长如果不
: 懂数学的,最好不要以自己的想法去教小孩。

t******l
发帖数: 10908
378
属实。。。而且会数学也不一定就会教育。

:教育学当然是一门科学。 举例来说, 现在一些州的test, 可以produce a
t******l
发帖数: 10908
379
这确实是一个问题。。。就拿上面的例子,如果要帮助娃建立 rational root theorem
的 instinct 的话,首先在 n 年前就需要建立 “有理数 vs 无理数” 的概念,打上
概念基础是不是?。。。因为这不仅仅是探讨 “divisible 与否” 的问题,而是探讨
“divisibility” 其概念本身是否总是存在的问题,也就是该 “数” 本身能不能基
于有限次整数除法而产生的,那个 “娃版 foundation of mathematics” 的基本层面
的问题。
但这又让我想起了当年乔装改扮深入 pushy mama 微信群敌后的日子。。。我就是某天
不小心嘀咕一句说 “帮娃娃概念证明一下无理数 vs 有理数”,就立即被微信群里一
片 “为啥不能像鲁迅一样把别家娃花在无理数概念上的时间,统统都花在刷 Kumon 计
算题上” 的声音给围歼消灭了。。。还好我当时反应快,当机立断立刻用卫星电话紧
急呼叫陆战一师的黑鹰直升机机群 air-lift,才得以用海空直撤的姿势,终于保住了
一条 BBS ID 的小命而从此离开敌后,之后转战抗战正面战场。。。

:你觉得现在的common core那些东西,比家长自己“教”的数学更能开发数学能力么
l*****c
发帖数: 1153
380
不可与夏虫语冰。淡定

theorem

【在 t******l 的大作中提到】
: 这确实是一个问题。。。就拿上面的例子,如果要帮助娃建立 rational root theorem
: 的 instinct 的话,首先在 n 年前就需要建立 “有理数 vs 无理数” 的概念,打上
: 概念基础是不是?。。。因为这不仅仅是探讨 “divisible 与否” 的问题,而是探讨
: “divisibility” 其概念本身是否总是存在的问题,也就是该 “数” 本身能不能基
: 于有限次整数除法而产生的,那个 “娃版 foundation of mathematics” 的基本层面
: 的问题。
: 但这又让我想起了当年乔装改扮深入 pushy mama 微信群敌后的日子。。。我就是某天
: 不小心嘀咕一句说 “帮娃娃概念证明一下无理数 vs 有理数”,就立即被微信群里一
: 片 “为啥不能像鲁迅一样把别家娃花在无理数概念上的时间,统统都花在刷 Kumon 计
: 算题上” 的声音给围歼消灭了。。。还好我当时反应快,当机立断立刻用卫星电话紧

相关主题
觉得美帝数学进度慢的进来看一下学校早晚要教的东西,早早学会了又如何?
我招,我是猪amc 10 的A, B卷有啥区别
大学录取率来点美好的时光
进入Parenting版参与讨论
t******l
发帖数: 10908
381
对于 twice exceptional,我觉得大多数娃本质上都是 twice exceptional,只是其
weaken 的方面不造成问题而已。。。就好比 propertiracy,为了建立对 property 的
intuition 也需要花费大脑硬盘空间,所以大脑就拉掉了相当部分的对 numeracy 的支
持,计算正确率降低,所以总的数学分数保持不变。。。但好处是总数学占脑力部分也
不变,这样就不挪用 gross motor 和 social 占脑力的空间,这样就不影响去 track
& field club 短跑队玩。
所以我觉得人人本质上都是 twice exceptional。。。所谓学术上的 twice
exceptional,就是发生了跨系挪用资金而被 FBI 盯上了的那些。。。系内调剂资金
FBI 就不管,因为最小审计单位是一个整系。。。// run

:我同意你说的"GIFTED和special education并不矛盾。gifted kids可能没有
:aspergeror hypernumeracy等, 也可能有。但有asperger等的并不一定是HGT"
教育方
t******l
发帖数: 10908
382
如果大藤自杀率高于 Highly Gifted,而 Highly Gifted 自杀率高于 Aspergers 的话
。。。那我觉得看起来自杀率更像是跟 “完美主义 + 期望值高”(Perfectionism +
High Expectations)的相关性更大,跟 Aspergers 相关性可能反而不是那么大。
这也可能可以解释为啥社交需求对自杀率反而可能是双刃剑。。。因为 “高期望值”
(High Expectations)的来源,一个是自身期望值,另一个是他人/社会期望值。。。
而后者反而跟社交需求正相关。。。而轻微自闭不鸟社会的,反而可能没事。
总结以上大胆假设就是,“沟命海心”跟自杀率的相关性,可能比 Aspergers 更大。
。。而心理学研究的 Highly Gifted,实际上是指 identified Highly Gifted,也就
是说被戴上了 Highly Gifted 高帽子的群体(当然这是废话,没戴高帽子的心理学老
军医也不可能知道,知道也不能发 paper),所以不排除有 “海命洋心” 的可能性。

:HGT自杀的肯定有些是Twice Exceptional,但多大比例很难说。因为一些藤校,特别
是大藤,自杀率甚至高于Asperger娃的自杀率,所以把HGT高自杀率完全归结于
Asperger
c********5
发帖数: 207
383
第一部分 说说他的短处,请帮我看看。 详见第124贴
第二部分 他还有几个我们都很郁闷的事情。详见第128贴
第三部分 继续我的龟速回贴
我最发愁的是娃的表达能力,在K开始的时候,retell故事的本事几乎为零,这个故事
的主人公是兔子,他很可能回答是小鸡。问故事讲什么了,就一句话。说的是一个相对
比较细节的描述。
我周围好多4岁小朋友都能很清楚滴描述preschool里发生的事情了。
喜欢silly的故事,喜欢nonfiction的书,有些喜欢的书,我给读过50遍以上,甚至过
百。他很少自己看书,但喜欢我给讲。至少半年前(K的中后期)我发现能
说出地球到太阳的距离(93 million mi),一天是23。9小时。知道1mile=1.6km。昨
天看书,读到1916年, 马上说100年前。
娃作raz kids,开始错误率极高,经过艰苦卓绝滴解释,现在平均一篇有1或2个错误。
他做razkids的时候,只盯着字,不看图的。问图里的问题,他100%会答错的,说文章
没讲呀。现在看来razkids对他的阅读理解应该是有训练作用的。
t******l
发帖数: 10908
384
我觉得普通娃很多有 “偶发暂时轻微 twice exceptional” 的一个例子是,很多大人
在着急或者思考的时候,会呈现 “偶发暂时轻微表达语言障碍”(spontaneous mild
anomic aphasia?)。。。每个人具体表现不一样,我自己的表现是快速敲 email 的时
候偶
尔漏词不算,关键漏掉的还是 not 之类的关键词。。。还有就是给娃讲有难度的数学
题时,娃说我不止一次指着减法说加法。。。还有就是快速滔滔不绝时突然摔倒在某个
发音上。。。当然每个人的症状不一样,有的人的症状是 “赶快把冰箱扔到洗碗机里
去洗一洗”,或者就是 “把肉放进冰箱里热一下”(稍微夸张一点哈)。。。

:对于 twice exceptional,我觉得大多数娃本质上都是 twice exceptional,只是其
:weaken 的方面不造成问题而已。。。就好比 propertiracy,为了建立对 property
c********5
发帖数: 207
385
在他考上天才班之前,我和老公都不同程度的担心他智商有问题。天才班考上之后,我
们心里才踏实些,觉得娃的智商应该是中等,或之上的水平。
他不知道单词什么意思,但是能读出来,我们在家没教,我一直认为是得益于4岁的蒙
校教育,他当时花了2个多月的时间熟悉26个首字母的发音,例如把以L和M开头的字母
的教具分类,教具不是单词,而是仿真的模型。例如蜥蜴,梯子,蘑菇。
然后就能读了。比较神的地方是,同样不认识的单词,他的发音往往是对的,我们的发
音是错误的。比较长的词,他还是不会读的。
不过因为暑假回国,现在娃中文进步很多,可惜英文退步也很多。
t******l
发帖数: 10908
386
其实我觉得这是 “大脑没有用到 100% 也用到了 95%” 之理论的又一个证明,也就是
学院总资金有限,所以同一学院的两个系临时按需把资金划来划去时,磁带 tar-in
tar-out 一下子没周转过来的情况。
但我觉得这不一定是坏事,spatial thinking 临时挪用 linguistics module 的内存
的话,虽然比较 annoying,但至少不挪用大脑 emotion / empathy / social /
executive function 这些虽然不那么直观,但出错损害更大的,那种高级模块的内存
不是?。。。从这个角度说,数学理论 instinct 学多了,反而解题时计算正确率降低
了,并且跟人解释的时候把减法说成加法,把 linear-ordered 说成 well-ordered,
这算是最小代价了,保底就是数学总成绩不变,但耗费总脑力也不变,不影响大脑其他
功能就不错了。。。至于心理学家苦思妄想的 Highly Gifted Kids,跟错误百出的普
通娃基本没啥关系就是了。。。

:我觉得普通娃很多有 “偶发暂时轻微 twice exceptional” 的一个例子是,很多大
人在着急或者思考的时候,会呈现 “偶发暂时轻微表达语言障碍”(spontaneous mild
:anomic aphasia?)。。。每个人具体表现不一样,我自己的表现是快速敲 email 的
时候偶
c********5
发帖数: 207
387
方向感我和老公都没感觉娃有什么特殊的,但是我倒是记得有几次他说对了,我错了。
现在我不行了。但是年轻的时候,只要我去过地方,我都会有一定的印象。很复杂的情
况下,我绝对能做到原路返回。
能给评说一下趣味数学,新加坡数学优缺点吗?
再有为啥大家都不用国内的教材?虽然简单粗暴一些,但物美价廉呀。而且是实用主义
论调呀。

【在 d******e 的大作中提到】
: 拼出单词和数学都是简化和推理能力强。
: 你家孩子方向感如何。
: 感觉学代数,物理时可能有优势吧。
: 不过这个也好像没大用。
: 做个马公,物理学家,和数学家也不算好出路。
: 等8,9岁丢一本趣味数学吧。
:
: 3

t******l
发帖数: 10908
388
国内教材跟美帝这边用词就不太一样。。。

:方向感我和老公都没感觉娃有什么特殊的,但是我倒是记得有几次他说对了,我错了
c********5
发帖数: 207
389
请问你家6岁普通娃phonics啥水平,读什么样的书,写作什么样?描述事情如何。阅读
理解能力啥样?retell故事什么水平?

【在 t******l 的大作中提到】
: 楼上 flyingpie 回答了你的问题。。。而我回答不了你的问题,因为我家小娃虽然在
: 幼儿园里的 comments 算是比较先进的娃,但她是个普通娃,不符合任何这些指标。
: 阅读方面:
: 我家小娃现在 6 岁,她的 phonics 差不多就是该年级的正常水平。虽然小娃她对
: 阅读有通常小孩的兴趣,但跟通常小孩一样,她对娃版教育节目兴趣更大。。。但
: 另一方面,而娃版教育 video 的 non-interactive 的方式,对通常小娃影响不大
: 。。。因为通常小娃还有 attention span 调节,或者 “一边XX一边YY”,也不太
: 容易到过犹不及的程度。
: 记忆方面:
: 我家小娃虽然在幼儿园里被夸记忆力很好,但实际上她还是经常忘记。。。这并不

t******l
发帖数: 10908
390
另一个猜测角度就是,就好比前面 AIME 题目,大脑在利用有限的数学脑力解题的时候
,因为脑力不够,所以必须对输入信息用 wishful thinking 连蒙带猜剪枝。。。但这
剪枝不保证总是剪对,剪错是常有的事儿,表现就是漏词用错词、把冰箱扔进洗衣机、
或者半道干脆来个断句啥的。。。当然把更多脑力划给数学的话,有可能就不需要
wishful thinking 连蒙带猜对输入信息进行不完全靠谱的信息剪枝了。。。但普通娃
的总脑力就这么多,都划给数学了,那 empathy 的脑力就可能都被挪用了而远远不够
用了。。。

:其实我觉得这是 “大脑没有用到 100% 也用到了 95%” 之理论的又一个证明,也就
是学院总资金有限,所以同一学院的两个系临时按需把资金划来划去时,磁带 tar-in
tar-out 一下子没周转过来的情况。
相关主题
有些小学四年级的数学题不太会做WISC-V 智商测试
转载: 从首个IMO季军谈起 by 付云皓今天和幼儿园老师开会review娃的progress,非常upset
新加坡数学对小学生帮助大吗?好学生去普通公立高中真的会被耽误吗?
进入Parenting版参与讨论
t******l
发帖数: 10908
391
我家就是普通娃普通指标,目前 picture book 读得磕磕碰碰,字都不认多少就不要谈
阅读理解,写作基本不会。。。描述事情问题不大,能 retell 故事大概,但丢失细节。

:请问你家6岁普通娃phonics啥水平,读什么样的书,写作什么样?描述事情如何。阅
读理解能力啥样?retell故事什么水平?
t******l
发帖数: 10908
392
另外前面你提到的 Raz-kids,小娃 K 班的时候学校提供了 Kids A-Z,也算是 Raz-
kids 的。。。小娃她靠听录音看图片当听力理解做,一般做对率还是可以的,挣
points 换里面的游戏装备。。。但小娃她当时大字更不识几个,没法做阅读,一旦阅
读就不行了。。。也还是普通娃普通指标。

:请问你家6岁普通娃phonics啥水平,读什么样的书,写作什么样?描述事情如何。阅
读理解能力啥样?retell故事什么水平?
s**********y
发帖数: 509
393
坡数学好像没有特色,大量练习倒是真的。

【在 c********5 的大作中提到】
: 方向感我和老公都没感觉娃有什么特殊的,但是我倒是记得有几次他说对了,我错了。
: 现在我不行了。但是年轻的时候,只要我去过地方,我都会有一定的印象。很复杂的情
: 况下,我绝对能做到原路返回。
: 能给评说一下趣味数学,新加坡数学优缺点吗?
: 再有为啥大家都不用国内的教材?虽然简单粗暴一些,但物美价廉呀。而且是实用主义
: 论调呀。

t******l
发帖数: 10908
394
灌到这里,我觉得可以做一个段落小结。。。也就是我认为对普通娃教育来说,Piaget
老先生的理论,不管存在多少缺陷,总体上还是最最合理的。
我觉得其中一个最最重要的原因,是 Piaget 被很多研究者诟病的研究手段 -- 观察
Piaget 自家几个娃 -- selection bias。
于是为了修正 Piaget 的这个问题,一些研究者去研究全球人类统计平均。。。黄小木
同学早就说了,人类早就在 regression toward sea sponge 的路上一路向西不回头啦
啦啦。。。研究全球人类平均,本质上是在研究猴子 -- 8 billion 猴子。
然后另一些研究者意识到上面这个问题。。。为了修正这个问题,另一些研究者以走极
端的姿势,去研究 CalTech 疯人院里的天才娃、HGT 娃、PG 娃。。。我就一句话,以
后研究疯子的 paper,在 Nature 上应该单独另起一页注明 “CalTech 疯人院研究报
告汇总”,跟人类认知学研究关系不大!!
总结就是,自 Piaget 以后,我看大部分研究者,不是在研究猴子、就是在研究疯子,
研究普通人类娃的没几个!!以后谁再说 Piaget 老先生研究方法的 selection bias
,我就跟谁急!!
t******l
发帖数: 10908
395
简单总结就是,Piaget 在家里观察自家娃,其后的哥们觉得这有 selection bias 倒
是也没错。。。但解决办法是这样的:一拨哥们去圣地亚哥动物园看猴子,另一拨哥们
去加州理工疯人院见疯子。最后两拨人马在环球影城好莱坞会合蒙太奇,Nature 大片
出炉。。。我都不希得说这些哥们了,当然为了短平快挣支票的心情也是可以理解的。
t******l
发帖数: 10908
396
当然刚才那些哥们发电报回来说,我们一半人马去加州理工看牛比,另一半人马去圣地
亚哥动物园看沙比,最后会师在环球影城好莱坞像你们马工一样,给加起来除以二给出
货,也不犯大错不是?
但我觉得问题是在于对“大脑可塑性”(cognition-level neuroplasticity),疯子
和猴子可能都不咋地,所以加起来除以二可能也不照。
<先发出来>

【在 t******l 的大作中提到】
: 简单总结就是,Piaget 在家里观察自家娃,其后的哥们觉得这有 selection bias 倒
: 是也没错。。。但解决办法是这样的:一拨哥们去圣地亚哥动物园看猴子,另一拨哥们
: 去加州理工疯人院见疯子。最后两拨人马在环球影城好莱坞会合蒙太奇,Nature 大片
: 出炉。。。我都不希得说这些哥们了,当然为了短平快挣支票的心情也是可以理解的。

t******l
发帖数: 10908
397
这个“大脑可塑性”(cognition-level neuroplasticity),有可能就是 Piaget 所
看到的冰山的水下部分之一。。。而从进化选择的角度看,合适程度 “大脑可塑性”
(cognition-level neuroplasticity),可能比仅仅基因重组,有更大的适应不断变
化的环境的优势。
当然,这里的关键是 “合适程度”。。。如果极端到 100% 的可塑性,没有任何预装
软件的话,那我们还要 TSMC 和三星干嘛,摩托罗拉直接卖你一块高纯硅(俗称:玻璃
)就好了。
但另一个极端,如果是 0% 的可塑性的话,我就问买个 TI-82 能不能在 SAT 考场里玩
Pokemon Go 吧。。。当然,刚才监考老师笑了。。。

【在 t******l 的大作中提到】
: 当然刚才那些哥们发电报回来说,我们一半人马去加州理工看牛比,另一半人马去圣地
: 亚哥动物园看沙比,最后会师在环球影城好莱坞像你们马工一样,给加起来除以二给出
: 货,也不犯大错不是?
: 但我觉得问题是在于对“大脑可塑性”(cognition-level neuroplasticity),疯子
: 和猴子可能都不咋地,所以加起来除以二可能也不照。
: <先发出来>

c********5
发帖数: 207
398
以前跟在虎妈牛娃后面推球类,娃和我都很崩溃。放手后整个世界都清净了。
所以现在我是做各种功课,但是不敢轻易在娃身上实践。惨痛的经验呀。

【在 t******l 的大作中提到】
: :以至于上K,我就放羊。
: 这个问题对于通常娃(也就是普通娃)而言,放不放都是羊,因为上帝放了一条细细
: 的红线,普通父母普通鸡飞狗跳是根本跨不过那条红线的。。。比如一条红线就是
: attention span,另一条红线就是 “扔 Kumon 习题集”。。。当然我家领导老是梦想
: 跨过那条看不见的红线,我都不希得说你特么这是跟上帝过不去啊。。。至于一脚绊倒
: 上帝的成功率为多少吧,自己看!
:
: 3

t******l
发帖数: 10908
399
所以在 “大脑可塑性”(cognition-level neuroplasticity)方面,猴子和疯子可能
都不照。
当然机理可能不太一样,猴子的问题,多半是是在 286 上装 Windows 10,用上 Kumon
压缩安装器才勉强装上,结果一旦 Windows 半夜 dreaming 时联网自动下载微软补丁
,就 disk full 从此补丁装不上。
而疯子的故事,可能是索南在 ebay 上下单买 Core i7 的时候,点名要求硬盘上预装
海量岛国爱情动作片,放在地下室里舍不得删,结果也是一旦 Windows 半夜 dreaming
时联网自动下载微软补丁时,这种就算 Core i7 也不行,一样也 disk full 从此补
丁也装不上。

【在 t******l 的大作中提到】
: 这个“大脑可塑性”(cognition-level neuroplasticity),有可能就是 Piaget 所
: 看到的冰山的水下部分之一。。。而从进化选择的角度看,合适程度 “大脑可塑性”
: (cognition-level neuroplasticity),可能比仅仅基因重组,有更大的适应不断变
: 化的环境的优势。
: 当然,这里的关键是 “合适程度”。。。如果极端到 100% 的可塑性,没有任何预装
: 软件的话,那我们还要 TSMC 和三星干嘛,摩托罗拉直接卖你一块高纯硅(俗称:玻璃
: )就好了。
: 但另一个极端,如果是 0% 的可塑性的话,我就问买个 TI-82 能不能在 SAT 考场里玩
: Pokemon Go 吧。。。当然,刚才监考老师笑了。。。

c********5
发帖数: 207
400
大人冤枉呀,我也没正儿八经滴教呀。我就是口头说了一下。我连笔纸都没动一下。
他按计算器的时候,问我这几个键什么意思呀。按不同的键,数不一样呀。
我想的是,学英语的机会来了,就告诉他oct是10的词根,就像october表示的是10月,
dec是12的词根,就像december表示的是12月。然后我就根本没走心的说:十二进制是
每满12进一位,就像十进制一样,每满10进1位一样。十二进制的一零,表示的是10进
制的12,十六进制是每满16进一位,16进制的一零,表示的是十进制的16.
我根本就没期望他能理解进制,我希望的是他能记住oct,dec是词根。
和娃爸确认了,他没给娃讲进制的问题。我应该是第一人,如果我没记错,以前他问过
同样的问题,我回答了同样的话。

【在 f*******e 的大作中提到】
: 6岁小孩教二进制,八进制,十六进制。。。我都懒得说你这当妈的了。
相关主题
陶天才论天才 (转载)9岁, 如何报名考amc10
Explaining Your Math: Unnecessary at Best, Encumbering at W有明天考AMC 8的吗?
是不是任我儿子继续迷象棋[转载] 为什么说usamo 简单
进入Parenting版参与讨论
t******l
发帖数: 10908
401
另外我大胆猜测,这个“大脑可塑性”(cognition-level neuroplasticity),有可
能就是一波人认为 “大脑只用了 10%”,而另一波人认为 “普通(正常被推的)娃的
大脑用了没有 100% 也有 95%” 的原因。
也就是说,存在一种可能,就是大脑把 80% 的脑力用在 “大脑可塑性”(cognition-
level neuroplasticity)。。。毕竟 “适者生存” (而不是 “靠举国体制拿藤校金
牌”)才是达尔文主义的精髓。而大脑不惜为此花费 80% 的脑力,在进化优势上也说
得过去(进化优势,首先是 “超长待机” + “任何情况都不当机”,半途挂掉咋都不
算进化优势)。。。至于这 80% 归成潜意识还是暗物质,有虎肉和李将军在,弗洛伊
德和霍金同志在 BBS 上永远都不会争论出个结果的。。。

Kumon
dreaming

【在 t******l 的大作中提到】
: 所以在 “大脑可塑性”(cognition-level neuroplasticity)方面,猴子和疯子可能
: 都不照。
: 当然机理可能不太一样,猴子的问题,多半是是在 286 上装 Windows 10,用上 Kumon
: 压缩安装器才勉强装上,结果一旦 Windows 半夜 dreaming 时联网自动下载微软补丁
: ,就 disk full 从此补丁装不上。
: 而疯子的故事,可能是索南在 ebay 上下单买 Core i7 的时候,点名要求硬盘上预装
: 海量岛国爱情动作片,放在地下室里舍不得删,结果也是一旦 Windows 半夜 dreaming
: 时联网自动下载微软补丁时,这种就算 Core i7 也不行,一样也 disk full 从此补
: 丁也装不上。

c********5
发帖数: 207
402
你突然让我想起北大的芙蓉姐姐。
报纸上好多10几岁天才娃,在老爹的实验室挂名,然后就出成果的。听过,一笑而过就
可以了。不必较真。

【在 f*******e 的大作中提到】
: “ 12岁上大学,绝大部分是自推的聪明娃。推基本是推不出来滴。当然极端情况,家
: 长放
: 弃一切,专心推也不是不可能滴。”
: 美国报道了很多这样的,都是home school, 上了个没听过的大学,就上报纸了,印度
: 裔居多。还有个吹自己小孩上了斯坦福的,结果发现是斯坦福online education. 这
: 年头BSO的太多了。你也别老想自己小孩是不是数学天才了。如果是数学天才,自己会
: 自推,不会被埋没的。你这样无来由的教6岁小孩高斯定律,教2进制,8进制,就算是
: 天才也会被教没的。你小孩是不是天才我不知道,但这当妈的绝对有问题。

t******l
发帖数: 10908
403
我个人觉得 Piaget 老先生的理论,对于普通娃最成功的地方,是 focus 在 conserve
/ reverse / inductive-thinking 等等这种 “intuition” 上,而不是具体的知识
或者 IQ 记忆力啥的(这种明显是 INTEL 硬件工程师出身)。。。这样 Piaget 理论
看起来更接近真正的 “认知心理学” -- teachable。
但另一方面,我个人觉得 Piaget 的 Formal Operation 其实也栽在这边,也就是背离
了 intuition。
<先发出来>

cognition-

【在 t******l 的大作中提到】
: 另外我大胆猜测,这个“大脑可塑性”(cognition-level neuroplasticity),有可
: 能就是一波人认为 “大脑只用了 10%”,而另一波人认为 “普通(正常被推的)娃的
: 大脑用了没有 100% 也有 95%” 的原因。
: 也就是说,存在一种可能,就是大脑把 80% 的脑力用在 “大脑可塑性”(cognition-
: level neuroplasticity)。。。毕竟 “适者生存” (而不是 “靠举国体制拿藤校金
: 牌”)才是达尔文主义的精髓。而大脑不惜为此花费 80% 的脑力,在进化优势上也说
: 得过去(进化优势,首先是 “超长待机” + “任何情况都不当机”,半途挂掉咋都不
: 算进化优势)。。。至于这 80% 归成潜意识还是暗物质,有虎肉和李将军在,弗洛伊
: 德和霍金同志在 BBS 上永远都不会争论出个结果的。。。
:

t******l
发帖数: 10908
404
我个人觉得 Piaget 的 Formal Operation,部分栽在对 deduction thinking 的理解
上。。。从上下文看,Piaget 的 deductive thinking,认为是 100% assertive 的。
。。而这个 100% assertive 就是问题。。。有争议的说,100% assertive 不是
deductive thinking,而是 “人肉计算器 thinking”。。。或者说,因为无法真正理
解 deductive thinking 而只能回到死记硬背的 belief,这样本质上是回到了
supernatural 的阶段。
<先发出来>

conserve

【在 t******l 的大作中提到】
: 我个人觉得 Piaget 老先生的理论,对于普通娃最成功的地方,是 focus 在 conserve
: / reverse / inductive-thinking 等等这种 “intuition” 上,而不是具体的知识
: 或者 IQ 记忆力啥的(这种明显是 INTEL 硬件工程师出身)。。。这样 Piaget 理论
: 看起来更接近真正的 “认知心理学” -- teachable。
: 但另一方面,我个人觉得 Piaget 的 Formal Operation 其实也栽在这边,也就是背离
: 了 intuition。
: <先发出来>
:
: cognition-

t******l
发帖数: 10908
405
这个 deductive thinking 不是 100% assertive 的,有三个证据:
(1)日常范畴的。。。如果做过 AIME-level & above 的,并且不是单纯靠背题型的
,就知道 wishful thinking “乱中猜测” 的重要性。。。而即使是 deductive-
thinking,也需要在这种 “乱中猜测” 的框架下。
(2)如果 deductive thinking 是 100% assertive 的,那么罗素就不可能推翻前人
而开创 formal logics 的纪元。。。而就算是今天,deductive thinking 的 Axiom
of Choice 还是有 Costco 开西瓜悖论。
(3)如果 deductive thinking 是 100% asssertive 的,那么 Autism /
Hypernumeracy 应该永远是 deductive thinking 最最好的了(个别天才除外,脑力强
大的缘故)。。。但实际上人不是计算机,就算是 deductive thinking 也需要被 “
大脑可塑性” (cognition-level neuroplasticity) 所调节,才能正常不断学习新的
deductive thinking。

【在 t******l 的大作中提到】
: 我个人觉得 Piaget 的 Formal Operation,部分栽在对 deduction thinking 的理解
: 上。。。从上下文看,Piaget 的 deductive thinking,认为是 100% assertive 的。
: 。。而这个 100% assertive 就是问题。。。有争议的说,100% assertive 不是
: deductive thinking,而是 “人肉计算器 thinking”。。。或者说,因为无法真正理
: 解 deductive thinking 而只能回到死记硬背的 belief,这样本质上是回到了
: supernatural 的阶段。
: <先发出来>
:
: conserve

t******l
发帖数: 10908
406
所以 deductive thinking 相对而言难以掌握、需要更多发展时间的原因,是因为
deductive thinking 要 very highly assertive,但同时也还是要 changeable。。。
其实是比 inductive thinking 更高一层的 intuition。
而有争议的说,intuition 跟 mechanic information processing 的最大区别是,
intuition 至少是理论上能改变其自身,往前发展的,就算该 intuition 是 highly
assertive 的。。。而 mechanic information processing 则是完全死板的。

【在 t******l 的大作中提到】
: 这个 deductive thinking 不是 100% assertive 的,有三个证据:
: (1)日常范畴的。。。如果做过 AIME-level & above 的,并且不是单纯靠背题型的
: ,就知道 wishful thinking “乱中猜测” 的重要性。。。而即使是 deductive-
: thinking,也需要在这种 “乱中猜测” 的框架下。
: (2)如果 deductive thinking 是 100% assertive 的,那么罗素就不可能推翻前人
: 而开创 formal logics 的纪元。。。而就算是今天,deductive thinking 的 Axiom
: of Choice 还是有 Costco 开西瓜悖论。
: (3)如果 deductive thinking 是 100% asssertive 的,那么 Autism /
: Hypernumeracy 应该永远是 deductive thinking 最最好的了(个别天才除外,脑力强
: 大的缘故)。。。但实际上人不是计算机,就算是 deductive thinking 也需要被 “

c***x
发帖数: 1826
407

conserve
倒是觉得研究普通娃和不普通娃的认知规律都很有意义,只是相对于我们复杂的大脑而
言,当下的神经认知科学还很初级。
After all, intelligence is our ultimate curiosity on our curiosity :-)

【在 t******l 的大作中提到】
: 我个人觉得 Piaget 的 Formal Operation,部分栽在对 deduction thinking 的理解
: 上。。。从上下文看,Piaget 的 deductive thinking,认为是 100% assertive 的。
: 。。而这个 100% assertive 就是问题。。。有争议的说,100% assertive 不是
: deductive thinking,而是 “人肉计算器 thinking”。。。或者说,因为无法真正理
: 解 deductive thinking 而只能回到死记硬背的 belief,这样本质上是回到了
: supernatural 的阶段。
: <先发出来>
:
: conserve

t******l
发帖数: 10908
408
而反过来从这个角度看,Piaget 老先生的 staged development,普通娃不能随便无限
制超前的原因之一。。。就是对普通娃而言,保持合理程度的大脑可塑性(cognition-
level neuroplasticity)永远是大脑认知发展最重要的几个目标之一。。。而从这个
角度,大脑就要防止自身过早进入 too much assertive (古人云:太早 jump into
conclusion 是一种危险),让大脑自身有足够时间完善到能接受那种程度的
assertive。
而从上面这个角度看,这可能就是普通娃不会过早成为 hypernumeracy /
hyperlexical 的原因。。。因为大脑的可塑性机制说了,请让理解和 intuition 先走
一步。

【在 t******l 的大作中提到】
: 所以 deductive thinking 相对而言难以掌握、需要更多发展时间的原因,是因为
: deductive thinking 要 very highly assertive,但同时也还是要 changeable。。。
: 其实是比 inductive thinking 更高一层的 intuition。
: 而有争议的说,intuition 跟 mechanic information processing 的最大区别是,
: intuition 至少是理论上能改变其自身,往前发展的,就算该 intuition 是 highly
: assertive 的。。。而 mechanic information processing 则是完全死板的。

t******l
发帖数: 10908
409
本来我敲累了想休息一下。。。看到你来了说一个有点意思的观察。。。就是 Piaget
的 reverse。
首先对小娃来说,减法是加法的 “reverse”,小娃能理解。。。但除法是乘法的 “
reverse”,小娃总是说她 “not really understand it”。
有一个有趣的是,我跟小娃在上学的路上说这个,也就是环境是没有草稿纸,没有实体
,都是 mental object 想象。。。然后这个有趣的地方是,what is 2*3。。。小娃可
以做 2+2+2,但是我要讲 reverse,所以我要求小娃用实体想象,然后我发现小娃最喜
欢的想象是:each stem has 2 bead, 3 stems, how many total beads。。。小娃想
象的是她的玩具算盘。
现在有趣的是,如果我把这个反过来,6/3。。。这是小娃最喜欢的想象不再是 stem /
bead,而是 6 birds, landing on 3 trees, each tree has same number of birds,
how many birds on each tree? 。。。一个完全不同的想象。。。用玩具算盘的
stem / bead 想象,小娃会 stuck 在那边不一定想通。。。而且目前小娃不用想象无
法解这个除法。。。
<先发出来>

【在 c***x 的大作中提到】
:
: conserve
: 倒是觉得研究普通娃和不普通娃的认知规律都很有意义,只是相对于我们复杂的大脑而
: 言,当下的神经认知科学还很初级。
: After all, intelligence is our ultimate curiosity on our curiosity :-)

t******l
发帖数: 10908
410
现在来了个更有趣的二义性。。。我想要么乘法也用 bird / tree 模型。。。于是我
问小娃 “each tree has 2 birds, total 3 trees”。。。然后我多说了一句 “all
birds fly to sky”,然后我说了一句 “how many birds”,忘了加上 “in the sky
”。。。小娃回答 “no birds on the tree”。。。OK,you are correct。。。
然后我问了 how many birds in the sky。。。and surprise to me。。。小娃没能问
答出来,她 stuck on the counting (in her mental picture)。
我个人觉得这个问题是在于,小娃的 modelling 还是非常 specific model,而不是一
个 generalized model (比如 array)。。。小娃看过 array 的 model,但还不能掌
握(mentally manipulate)。。。所以看起来小娃的这个乘法和除法 reverse 的问题
,并不在于 operation 本身,而是大脑为 operation 所建立的 model,或者说
intuition,不容易 reverse(stem/bead vs tree/bird 这俩不一样的玩意儿当然不太
容易 reverse)。
还有一个奇数加奇数等于偶数的形象证明,累了不敲字了。。。

Piaget
/

【在 t******l 的大作中提到】
: 本来我敲累了想休息一下。。。看到你来了说一个有点意思的观察。。。就是 Piaget
: 的 reverse。
: 首先对小娃来说,减法是加法的 “reverse”,小娃能理解。。。但除法是乘法的 “
: reverse”,小娃总是说她 “not really understand it”。
: 有一个有趣的是,我跟小娃在上学的路上说这个,也就是环境是没有草稿纸,没有实体
: ,都是 mental object 想象。。。然后这个有趣的地方是,what is 2*3。。。小娃可
: 以做 2+2+2,但是我要讲 reverse,所以我要求小娃用实体想象,然后我发现小娃最喜
: 欢的想象是:each stem has 2 bead, 3 stems, how many total beads。。。小娃想
: 象的是她的玩具算盘。
: 现在有趣的是,如果我把这个反过来,6/3。。。这是小娃最喜欢的想象不再是 stem /

相关主题
[转载] 为什么说usamo 简单我招,我是猪
二年级女儿的report大学录取率
觉得美帝数学进度慢的进来看一下学校早晚要教的东西,早早学会了又如何?
进入Parenting版参与讨论
c***x
发帖数: 1826
411

Piaget
/
赞生动事例。6岁小娃有这个想象力已经很棒了。

【在 t******l 的大作中提到】
: 本来我敲累了想休息一下。。。看到你来了说一个有点意思的观察。。。就是 Piaget
: 的 reverse。
: 首先对小娃来说,减法是加法的 “reverse”,小娃能理解。。。但除法是乘法的 “
: reverse”,小娃总是说她 “not really understand it”。
: 有一个有趣的是,我跟小娃在上学的路上说这个,也就是环境是没有草稿纸,没有实体
: ,都是 mental object 想象。。。然后这个有趣的地方是,what is 2*3。。。小娃可
: 以做 2+2+2,但是我要讲 reverse,所以我要求小娃用实体想象,然后我发现小娃最喜
: 欢的想象是:each stem has 2 bead, 3 stems, how many total beads。。。小娃想
: 象的是她的玩具算盘。
: 现在有趣的是,如果我把这个反过来,6/3。。。这是小娃最喜欢的想象不再是 stem /

c***x
发帖数: 1826
412

all
sky
modelling属于抽象思维,属于formal operation,你娃才6岁啊!
不过,你还是挺循循善诱的 :-)

【在 t******l 的大作中提到】
: 现在来了个更有趣的二义性。。。我想要么乘法也用 bird / tree 模型。。。于是我
: 问小娃 “each tree has 2 birds, total 3 trees”。。。然后我多说了一句 “all
: birds fly to sky”,然后我说了一句 “how many birds”,忘了加上 “in the sky
: ”。。。小娃回答 “no birds on the tree”。。。OK,you are correct。。。
: 然后我问了 how many birds in the sky。。。and surprise to me。。。小娃没能问
: 答出来,她 stuck on the counting (in her mental picture)。
: 我个人觉得这个问题是在于,小娃的 modelling 还是非常 specific model,而不是一
: 个 generalized model (比如 array)。。。小娃看过 array 的 model,但还不能掌
: 握(mentally manipulate)。。。所以看起来小娃的这个乘法和除法 reverse 的问题
: ,并不在于 operation 本身,而是大脑为 operation 所建立的 model,或者说

t******l
发帖数: 10908
413
今天码字码累了,我下次码奇数加奇数等于偶数的小娃版证明,我用的是两幅不同颜色
的纸牌背面,左右轮流发牌,实际上创建了证明所需的 “any” 算子。。。小娃当时
认可是 always true。。。但一天后还是回到 mostly true。。。当然后来我没有再去
给小娃用纸牌证明,生怕过犹不及反而搞成机械记忆,以后再说。
另一角度,这个帮助判断是不是过犹不及,我觉得确实是 Piaget 理论对实践的一个重
要指导意义。

c***x
发帖数: 1826
414

Piaget
/
生活中也这么多话吗?也这么好玩吗?也这么头头是道,歪理真理各半吗?
累了就早点休息吧。老刑又不付你工资,娃还要靠你指导,不管上藤校还是州大,还要
靠你的支票。
我下了,干活的去了。

【在 t******l 的大作中提到】
: 本来我敲累了想休息一下。。。看到你来了说一个有点意思的观察。。。就是 Piaget
: 的 reverse。
: 首先对小娃来说,减法是加法的 “reverse”,小娃能理解。。。但除法是乘法的 “
: reverse”,小娃总是说她 “not really understand it”。
: 有一个有趣的是,我跟小娃在上学的路上说这个,也就是环境是没有草稿纸,没有实体
: ,都是 mental object 想象。。。然后这个有趣的地方是,what is 2*3。。。小娃可
: 以做 2+2+2,但是我要讲 reverse,所以我要求小娃用实体想象,然后我发现小娃最喜
: 欢的想象是:each stem has 2 bead, 3 stems, how many total beads。。。小娃想
: 象的是她的玩具算盘。
: 现在有趣的是,如果我把这个反过来,6/3。。。这是小娃最喜欢的想象不再是 stem /

t******l
发帖数: 10908
415
我今天确实码字比较多,要先休息一下,周末找个时候查邮件吧。


:【 在 timefall (时光崩塌) 的大作中提到: 】
s**********y
发帖数: 509
416
小蛙从初二开始, 一般都能够自己做出来。

Piaget
/

【在 t******l 的大作中提到】
: 本来我敲累了想休息一下。。。看到你来了说一个有点意思的观察。。。就是 Piaget
: 的 reverse。
: 首先对小娃来说,减法是加法的 “reverse”,小娃能理解。。。但除法是乘法的 “
: reverse”,小娃总是说她 “not really understand it”。
: 有一个有趣的是,我跟小娃在上学的路上说这个,也就是环境是没有草稿纸,没有实体
: ,都是 mental object 想象。。。然后这个有趣的地方是,what is 2*3。。。小娃可
: 以做 2+2+2,但是我要讲 reverse,所以我要求小娃用实体想象,然后我发现小娃最喜
: 欢的想象是:each stem has 2 bead, 3 stems, how many total beads。。。小娃想
: 象的是她的玩具算盘。
: 现在有趣的是,如果我把这个反过来,6/3。。。这是小娃最喜欢的想象不再是 stem /

s**********y
发帖数: 509
417
赞创造力。 娃觉得需要知道奇偶哈?

【在 t******l 的大作中提到】
: 今天码字码累了,我下次码奇数加奇数等于偶数的小娃版证明,我用的是两幅不同颜色
: 的纸牌背面,左右轮流发牌,实际上创建了证明所需的 “any” 算子。。。小娃当时
: 认可是 always true。。。但一天后还是回到 mostly true。。。当然后来我没有再去
: 给小娃用纸牌证明,生怕过犹不及反而搞成机械记忆,以后再说。
: 另一角度,这个帮助判断是不是过犹不及,我觉得确实是 Piaget 理论对实践的一个重
: 要指导意义。
:
: :

t******l
发帖数: 10908
418
娃的 学校/after-school 里给娃提及的。。。普通娃没那么多创造力,但也不排斥。
。。这就好比给娃买个新玩具,普通娃总是喜欢新玩具,但玩了一段时间也就差不多了
,也不会太着迷。
我觉得趣味数学对普通娃就是两面,一面就是有用;另一面就是个玩具,这个玩具老实
说也不太好玩,但比没有玩具强。

:赞创造力。 娃觉得需要知道奇偶哈?
t******l
发帖数: 10908
419
小学二年级吧。。。初二很多是 AMC 8 了。。。

:小蛙从初二开始, 一般都能够自己做出来。
t******l
发帖数: 10908
420
其实这个问题是我多此一举加了句 “all birds fly to sky“。。。如果不加那句,
也就是隐含 “all birds stay on the trees“,那娃用 bird / tree 做(10 或 20
以内的)乘法除法就没有问题,不会 stuck on counting。。。所以可以用同样的模型
做乘法和除法,但我觉得乘法除法时,小鸟飞还是不飞情景有可能不完全一样。
总是普通小娃乘除 reverse 的概念,需要时间慢慢2玩。。。总之第一罗马不是一日建
成。第二不为竞赛藤校的话,特别对普通小娃,学个趣味数学也不用太认真太苦鼻,我
觉得保着玩玩的心态,能加料就喂一口。娃吃了一口继续去看 Martha Speaks ,那就
随着娃的天性。普通娃不值得去强扭。想 “一脚绊倒上帝” 不是那么容易的。

:现在来了个更有趣的二义性。。。我想要么乘法也用 bird / tree 模型。。。于是我
:问小娃 “each tree has 2 birds, total 3 trees”。。。然后我多说了一句 “
all birds fly to sky”,然后我说了一句 “how many birds”,忘了加上 “in the
sky”。。。小娃回答 “no birds on the tree”。。。OK,you are correct。。。
相关主题
amc 10 的A, B卷有啥区别转载: 从首个IMO季军谈起 by 付云皓
来点美好的时光新加坡数学对小学生帮助大吗?
有些小学四年级的数学题不太会做WISC-V 智商测试
进入Parenting版参与讨论
s**********y
发帖数: 509
421
打快了, 除二小蛙自己可以做 。要不先从除二开始。 三可以慢慢加入。

【在 t******l 的大作中提到】
: 小学二年级吧。。。初二很多是 AMC 8 了。。。
:
: :小蛙从初二开始, 一般都能够自己做出来。

t******l
发帖数: 10908
422
我已经教了除二了,除二没有问题,所以加上除三和除一,以及 6/6 结果应该是 1 这
种。

:打快了, 除二小蛙自己可以做 。要不先从除二开始。 三可以慢慢加入。
t******l
发帖数: 10908
423
其实我还教了一次用 skip counting 做除法,用扑克牌反面来形象化表示的。。。但
我转天测试一下,娃还是不用那个 skip counting 做除法(偶尔会用来做乘法倒是)
。。。所以不强扭。
我的个人理念是 “一脚绊倒上帝” 的难度很大。。。我觉得普通娃可能要先建立对除
法的 “图景” / intuition,然后才能谈 skip counting 之类的然后。。。我觉得父
母的任务主要是 “师傅领到门前”,之后让娃的思维慢火自然熟,至少对父母比较容
易。

:我已经教了除二了,除二没有问题,所以加上除三和除一,以及 6/6 结果应该是 1
这种。
t******l
发帖数: 10908
424
另外就是出现零和一,对我们大人是很容易的事,但对普通小娃似乎并不是那么容易。
。。这可能说明对普通小娃,additive identity 和 multiplicative identity 的概
念并不是那么容易的。。。推而广之,普通娃在认知 (mathematical) identity 的存
在和概念,建立图景和 intuition,可能需要一个过程,以及相应的时间。
不过话说回来,对我们大人而言,mathematical identity (比如零或一)的概念,之
所以觉得容易,也可能就是题目简单或者是已经记忆成定式。
一个例子就是如果是 AIME non-routine
problem,比如 prime factorization 完做 partition 的时候,1 (multiplicative
identity) 是不是要作为一个 partition 的情况,老实说漏掉的人也不少了。
就算不漏掉,大部分人还是会专门想一想出现 multiplicative identity 算不算对于
该题而言是一种 valid partition。
所以说明在实际新情况或者复杂情况下,处理 mathematical identity 的概念图景
intuition,可能对大人而言,也不是那么容易的。

:我已经教了除二了,除二没有问题,所以加上除三和除一,以及 6/6 结果应该是 1
这种。
1 (共1页)
进入Parenting版参与讨论
相关主题
新加坡数学对小学生帮助大吗?有明天考AMC 8的吗?
WISC-V 智商测试[转载] 为什么说usamo 简单
今天和幼儿园老师开会review娃的progress,非常upset二年级女儿的report
好学生去普通公立高中真的会被耽误吗?觉得美帝数学进度慢的进来看一下
陶天才论天才 (转载)我招,我是猪
Explaining Your Math: Unnecessary at Best, Encumbering at W大学录取率
是不是任我儿子继续迷象棋学校早晚要教的东西,早早学会了又如何?
9岁, 如何报名考amc10amc 10 的A, B卷有啥区别
相关话题的讨论汇总
话题: 数学话题: asperger话题: aime话题: 普通话题: 小娃